Sie sind auf Seite 1von 144

Table of Contents

GENERAL KNOWLEDGE ................................................................................................................... 2


NLSIU ................................................................................................................................................... 2
1998............................................................................................................................................... 24
1999............................................................................................................................................... 27
2000............................................................................................................................................... 29
2001............................................................................................................................................... 30
2002............................................................................................................................................... 32
2003............................................................................................................................................... 34
2004............................................................................................................................................... 36
2005............................................................................................................................................... 37
2006............................................................................................................................................... 39
2007............................................................................................................................................... 40
NALSAR.............................................................................................................................................. 41
1998............................................................................................................................................... 41
1999............................................................................................................................................... 44
2000............................................................................................................................................... 47
2001............................................................................................................................................... 52
2002............................................................................................................................................... 56
2003............................................................................................................................................... 60
2004............................................................................................................................................... 63
2005............................................................................................................................................... 65
2006............................................................................................................................................... 68
2007............................................................................................................................................... 73
NUJS .................................................................................................................................................. 76
2000............................................................................................................................................... 76
2001............................................................................................................................................... 77
2002............................................................................................................................................... 79
2003............................................................................................................................................... 81
2004............................................................................................................................................... 84
2005............................................................................................................................................... 85
2006............................................................................................................................................... 87
2007............................................................................................................................................... 89

© Clat Possible. All rights reserved. Unauthorized copying, sale, distribution or circulation
of any of the contents of this work is a punishable offence under the laws of India.
www.clatpossible.com
A Team Satyam Offering
CLAT .................................................................................................................................................. 91
2010............................................................................................................................................... 98
2011............................................................................................................................................. 103
2012............................................................................................................................................. 105
2013............................................................................................................................................. 107
2014............................................................................................................................................. 114
2015............................................................................................................................................. 117
2016 ................................................................................................................................................ 122
NLU-D .............................................................................................................................................. 126
2015............................................................................................................................................. 139
OPERATION DECISIVE STORM ......................................................................................................... 141
Afghanistan has become the 164th WTO member and the 36th least developed country (LDC) to
join the global trade body.The World Trade Organisation (WTO) on Thursday formally approved
Afghanistan’s membership at its 10th ministerial conference in the Kenyan capital Nairobi. ....... 144

GENERAL KNOWLEDGE
NLSIU
1988
Question Answer Explanation
Number
1. A Inflation. Inflation is when incomes do not increase along with the prices
of the good thereby, leaving an effectively reduced purchasing power.
2. B Inverse. The general concept is that as goods become more expensive,
people tend to demand less of them. E.g. If the cost of a shirt doubles,
consumers buy fewer shirts, all else being equal. If the shirts go on sale,
consumers tend to buy more.
3. C President. As per article 217, the chief Justice of the high court is
appointed by the President in consultation with the Chief justice of India
as well as the Governor of the state in question.
4. D Non-interference of State. The dictionary meaning of the word
‘secularism’ is skepticism in matters of religion. But we, in India, use the
work in a broader sense. We use the word to mean impartiality or non-
interference by the Government of the country in matters of religion.
5.
6. C Badminton. Thomas Cup, sometimes called the World Men’s Team
Championships, is an international badminton competition among teams
representing member nations of the Badminton World Federation (BWF),
the sport’s global governing body. The Uber Cup is sometimes called the
World Team Championships for Women.
7. C Gunnar Myrdal
8. D Bahadur Shah Zafar. He was the son of Mirza Akbar Shah II and Lalbai,

2
who was a Hindu Rajput, Zafar became Mughal Emperor when his father
died on 28 September 1837.
9. B Constitution of India was adopted by the Constituent Assembly on 26th
November, 1949 and came into force on 26th January, 1950.
10. C Both Houses of Parliament. The Vice-President is elected by an Electoral
College, which consists of the members of the Lok Sabha and Rajya Sabha
(both elected and nominated members)
11. C Sex of fetus.
12. B Reservation provisions in public services are intended to ensure that the
vast gap in educational attainments and economic status does not stand in
the way of occupying decision making positions.
13. C Lok Adalat is one of the alternative dispute redressal mechanisms, it is a
forum where disputes/cases pending in the court of law or at pre-litigation
stage are settled/ compromised amicably. Lok Adalats have been given
statutory status under the Legal Services Authorities Act, 1987. Please see:
http://www.legalserviceindia.com/articles/lok_a.htm
14. C Enquiring corruption in government.
15. D Quit India Movement. This resolution was ratified in the All India
Congress Committee at Bombay on August 7, 1942. Here a nonviolent
mass struggle under the leadership of Gandhi was sanctioned in the
“August Kranti Maidan”.
16. D Inflation.
17. C/D Inflation can benefit either the lender or the borrower, depending on the
circumstances. If wages increase with inflation, and if the borrower
already owed the money before the inflation occurred, the inflation
benefits the borrower. When there is fresh credit.
Please visit: http://www.investopedia.com/ask/answers/111414/does-
inflation-favor-lenders-or-borrowers.asp#ixzz4TMhi2KBi
18. C Solar energy
19. A 100. According to the National Institute of Health, the average resting
heart rate: for children 10 years and older, and adults (including seniors) is
60 - 100 beats per minute
20. None 29 States.

1989
Question Answer Explanation
Number
1. C Seismology
2. C Under Article 61, the President of India can be removed from the office
by a process of impeachment for the violation of the Constitution. The
impeachment is to be initiated by either House of Parliament. The
charges are to be framed in the form of resolution, signed at least by 1/4
th members of the total members of the House. The President has to be
given a notice of 14 days in advance. The resolution is to be passed by
2/3 rd majority of the total members of the House and then it is to be
sent to other House for investigation and decision. If the other House
after investigation sustains the charges and passes the identical
resolution with 2/3 rd majority of the total membership, the President

3
ceases to hold office from the date such resolution is passed.
3. B Prithvi. Prithvi missiles are tactical short range surface-to-surface
ballistic missiles. There are 3 variants of it. Prithvi-I, an Army Version,
Prithvi-II, an Air Force Version, and Prithvi-III, a Naval Version
4. C Commonwealth
5. C 685m.
6. NOTA According to WHO's report, as of 2015, the life expectancy in India is:
Male- 66.9, female- 69.9 and total-68.3. India is ranked 123 of World
Life Expectancy.
7. B Advisory body. It is not created by an act of Parliament and is not
provided for within the Constitution.
8. B Balance of Trade. The balance of trade (BOT) is the difference between
a country's imports and its exports for a given time period. The balance
of trade is the largest component of the country's balance of
payments (BOP). Economists use the BOT as a statistical tool to help
them understand the relative strength of a country's economy versus
other countries' economies and the flow of trade between nations. The
balance of trade is also referred to as the trade balance or the
international trade balance.
9 B Inverse relation with price. The general concept is that as goods become
more expensive, people tend to demand less of them. E.g. If the cost of
a shirt doubles, consumers buy fewer shirts, all else being equal. If the
shirts go on sale, consumers tend to buy more
10 D Public and private sectors. An economic system in which both the
private enterprise and a degree of state monopoly (usually in public
services, defense, infrastructure, and basic industries) coexist.
11 B Decrease in value of money. Inflation is when incomes do not increase
along with the prices of the good thereby, leaving an effectively
reduced purchasing power. This leads to decrease in value of money.
12 C Budget
13. http://economictimes.indiatimes.com/news/economy/policy/indira-
gandhis-rupee-devaluation-master-stroke-the-forgotten-legacy-of-
6/6/66/articleshow/52631073.cms
14. D -40
15. D Light years
16. B More than the speed of sound. Vehicles that fly at supersonic speeds
are flying faster than the speed of sound. The speed of sound is about
768 miles per hour (1,236 kilometers per hour) at sea level.
17. A Someone with short-sightedness can see near objects clearly, but cannot
focus properly on distant objects. This is caused by the eyeball being
elongated, so that the distance between the lens and the retina is too
great. It can be corrected by placing a concave lens in front of the eye.
18. B Nigeria. Wole Soyenka won the Nobel Prize in Literature in 1986. In
the subsequent years, Nadine Gordimer (South Africa), John Maxwell
and Doris Lessing also received the prestigious Honour.
http://www.cp-africa.com/2012/05/27/list-of-african-nobel-prize-
winners/
19. C Col. Gaddafi
20. D The Jaguar Smile

4
21. C A.O. Hume
22. NOTA Chaudhary Rehmat Ali. He coined the word “Pakistan” for 30 million
Muslims who live in the five northern units of India; Punjab, North
West Frontier (Afghan) Province, Kashmir, Sindh and Baluchistan.
http://storyofpakistan.com/chaudhry-rehmat-ali
23. A Dominion.
24. C Buddha's previous births.
25. C She was elected PM of Srilanka who retired at the age of 84.
26. C Tiger.
27. D The state emblem is an adaptation from the Sarnath Lion Capital of
Ashoka. In the original, there are four lions, standing back to back,
mounted on an abacus with a frieze carrying sculptures in high relief of
an elephant, a galloping horse, a bull and a lion separated by
intervening wheels over a bell-shaped lotus. Carved out of a single
block of polished sandstone, the Capital is crowned by the Wheel of the
Law (Dharma Chakra).
In the state emblem, adopted by the Government of India on 26 January
1950, only three lions are visible, the fourth being hidden from view.
The wheel appears in relief in the centre of the abacus with a bull on
right and a horse on left and the outlines of other wheels on extreme
right and left. The bell-shaped lotus has been omitted. The
words Satyameva Jayate from Mundaka Upanishad, meaning 'Truth
Alone Triumphs', are inscribed below the abacus in Devanagari script.
28. C The term Torah is also used to designate the entire Hebrew Bible. Since
for some Jews the laws and customs passed down through oral
traditions are part and parcel of God’s revelation to Moses
and constitute the “oral Torah,” Torah is also understood to include
both the Oral Law and the Written Law.
29. D Sriharikota
30. D Narora Atomic Power Station is located in Narora, Bulandshahar
District in Uttar Pradesh
31. D Dalal Street in downtown Mumbai, India is the address of the Bombay
Stock Exchange and several related financial firms and institutions.
32. C India Standard Time ( IST ) is 5:30 hours (5 hours 30 minutes) ahead
of Greenwich Mean Time(GMT+5.5).
33. C Kavaratti. Kavaratti serves as the capital of the Union Territory and the
region comes under the jurisdiction of Kerala High Court. The islands
are the northernmost of the Lakshadweep-Maldives-Chagos group of
islands, which are the tops of a vast undersea mountain range, the
Chagos-Laccadive Ridge.
34. D The administrative head of the Ministry is the Secretary. The Secretary
also acts as the Director General
35. D Apsara is the oldest of India's research reactors. The reactor was
designed by the Bhabha Atomic Research Center (BARC) and built
with assistance from the United Kingdom. Apsara is a light water
swimming pool-type reactor with a maximum power output of one
megawatt thermal (MWt).
36. A Reita Faria. Priyanka Chopra(2000), Aishwarya Rai (1994), Diana
Hayden (1997); and Yukta Mookhey (1999) are others who have won

5
this Title.
37. D
38. D The Shining Path began in the late 1960s as a small communist
revolutionary group led by a philosophy professor named Abimael
Guzmán. Guzmán opposed Peru’s prevailing political elites. His
followers drew on Marxism and the example of Cuba’s Fidel Castro,
and coalesced into a significant and violent guerrilla army which
regularly used terrorist tactics in their effort to destabilize and
overthrow the Peruvian government
39. B
40. D Alexei Leonov opened the airlock of his tiny space capsule, which was
orbiting the Earth, and stepped into the void. For 12 minutes the Soviet
cosmonaut floated above our planet, tethered to his ship by a 16ft cable.
In doing so, Leonov became the first person to walk in space.

1990
Question Answer Explanation
Number
1. C The Sea of Tranquility is the landing site of Apollo 11, the mission that
gave mankind its first ever walk on the Moon.
2. B Centaur. "The logo of the new airline is a red coloured flying swan with
the 'Konark Chakra' in orange, placed inside it. The flying swan has
been morphed from Air India's characteristic logo, 'The Centaur',
whereas the 'Konark Chakra' is reminiscent of Indian's logo".
3. C Barcelona (Summer Olympics in 1992) & Albertville(Winter Olympics
in 1992)
4. The Nobel Peace Prize 1989 was awarded to The 14th Dalai
Lama(Tenzin Gyatso).
5. C The Periyar Tiger Reserve, named after the Periyar river, is one of
Kerala's prestigious possessions on the High Ranges of the Western
Ghats.
6. B
7. B Clement Attlee. The legislation was formulated by the government of
Prime Minister Clement Attlee and the Governor General of India Lord
Mountbatten, after representatives of the Indian National Congress, the
Muslim League, and the Sikh community came to an agreement with
the Viceroy of India, Lord Mountbatten of Burma.
8. A The Hague
9. - Question is wrong spelled
10. C
11. B The Aurora is an incredible light show caused by collisions between
electrically charged particles released from the sun that enter the earth's
atmosphere and collide with gases such as oxygen and nitrogen. The
lights are seen around the magnetic poles of the northern and southern
hemispheres.
12. C
13. A In 1976 in Montreal, Romanian athlete Nadia Comaneci became
the first gymnast in Olympic history to be awarded the perfect score of

6
10.0 for her performance on the uneven bars. She went on to record
the perfect 10.0 six more times and became the youngest all-around
Olympic gold medallist ever.
14. C
15. D The Amazon is the greatest river in the world by so many measures; the
volume of water it carries to the sea (approximately 20% of all the
freshwater discharge into the oceans), the area of land that drains into it,
and its length and width.
16. D
17. A
18. C
19. B 1556. The battle was fought between Akbar (Ruler of Mughal Dynasty)
and Muhammad Adil Shah (ruler of Pashtan Suri Dynasty), along with
his Prime Minister Hemu.
20. B Panchajanya. Lord Vishnu is said to hold a special conch, Panchajanya ,
that represents life as it has come out of life-giving waters. It is believed
that Paanchajanya emerged during the churning of the Ksheerasagara
by the devas and the asuras. As it rose out of the ocean, its tremendous
decibel frightened the asuras who appealed to Vishnu to save them.
Lord Vishnu obliged, taking charge of the conch shell . The primordial
sound of creation, that is the ‘ Omkar' or ‘ Pranavanadham' , was
thereby controlled.
21. C
22. B The Islamic calendar is based on the year prophet Muhammad and his
fellow Muslims (known as Sahabah, the Companions) emigrated to
Madinah in the year 622 C.E. (Christian Era). The emigration took
place after thirteen years of persecutions by the disbelievers of Makkah.
By the command of God, the Prophet left the city with his companion
Abu Bakr Siddique (R.A.) and escaped a death threat by the
disbelievers. The event marks the beginning of a second phase of the
Islamic movement. It is the phase when Madinah became the center of
an Islamic state.
23. B
24. C
25. B Geiger counters are used to detect radioactive emissions, most
commonly beta particles and gamma rays.
26. A Mount Sagarmatha (Everest; 8,848 m) and an elevation range of 6,000
m Sagarmatha National Park (SNP) covers an area of 124,400 hectares
in the Solu-Khumbu district of Nepal.
27. B On 13 November 1989, the Sri Lankan government announced that
Rohana Wijeweera, an extreme-left Sinhalese nationalist leader of two
failed insurrections, had died in police custody - in unclear
circumstances.
28. A An economic system in which both the private enterprise and a degree
of state monopoly (usually in public services, defense, infrastructure,
and basic industries) coexist.
29. A Liver is the largest gland in human body. It is also the largest (internal)
organ in our body and can weigh up to 1.5-1.6 kg for a human adult.

7
30. B Dadasaheb Phalke Award is India’s highest award in cinema. This
award was established in 1969 to commemorate the birth centenary
year of father of Indian cinema Dadasaheb Phalke. The award is
conferred annually by the Union Government to person for his lifetime
contribution to Indian cinema.
31. C

32. NOTA Yasser Arafat


Please visit: http://www.iccr.gov.in/content/nehru-award-recipients

33. C In 2002 the lira ceased to be legal tender in Italy after the euro,
the European Union’s monetary unit, became the country’s sole
currency.

34. A

35. B Gold Medal in Men's 100m.

36. A Rover

37. A

38. B

39. B Myth: Ferdinand Magellan (Fernão de Magalhães) was the first person
to circumnavigate the globe. There's no doubt that Magellan intended to
have a successful journey when his expedition set off from Spain on
September 20, 1519.
40. B

1990(2)
Question Answer Explanation
Number
1. B
2. D A nuncio is appointed by and represents the Holy See, and is the head

8
of the diplomatic mission, called an Apostolic Nunciature, which is the
equivalent of an embassy.
3. B Fungi are nonmotile, filamentous eukaryotes that lack plastids and
photosynthetic pigments. The majority of fungi are saprophytes; they
obtain nutrients from dead organic matter. Other fungi survive as
parasitic decomposers, absorbing their food, in solution, through their
cell walls.
4. B Johannes Gutenberg. The German goldsmith's 15th-century
contribution to the technology was revolutionary — enabling the mass
production of books and the rapid dissemination of knowledge
throughout Europe.
5. B It is celebrated as the Declaration of Independence, adopted on 4 July,
1776. The Thirteen Colonies of America declared themselves to be
states and no longer part of the British Empire, though the
revolutionary war continued for some time after.
6. D Russian Revolution of 1917, two revolutions, the first of which, in
February (March, New Style), overthrew the imperial government and
the second of which, in October (November), placed the Bolsheviks in
power.
7. C Manuel Antonio Noriega Morena, Panamanian military leader,
commander of the Panamanian Defense Forces (1983–89), who, for
the years of his command, was the actual power behind the civilian
president.
8. A One of the great Dutch painters and printmakers of the 17th
century, Rembrandt van Rijn is best known for his expressive use of
light and shadow (also called chiaroscuro) in his many portraits.
9. A ornithologist studies birds. Ornithologists may study the behavior,
physiology, and conservation of birds and bird habitats.
10. D Paris
11. C Taka is currency of Bangladesh.
12. A An avalanche is a rapid flow of snow down a hill or mountainside.
Although avalanches can occur on any slope given the right
conditions, certain times of the year and certain locations are naturally
more dangerous than others.
13. C Fiduciary relationsips are relationships based on trust. E.g. Lawyer-
client, Doctor- patient.
14. A Prior to 1982, the Jnanpith Award were given for a single work of
Author, but after that, the Jnanpith Award is being given for the
lifetime work.
15. C
16. C On 8th August 1942, Mahatma Gandhi launched the Quit India
Movement for freedom from British rule in Mumbai (then
Bombay). The Quit India Movement, also known as the August
Movement was a Civil Disobedience Movement launched by Gandhi
for Satyagraha (independence)
17. C Yakshagana is a traditional theatre form that combines dance, music,
dialogue, costume, make-up, and stage techniques with a unique style
and form. This theatre style is mainly found in the coastal districts and
the Malenadu region of Karnataka, India.

9
18. B A barometer is a scientific instrument used in meteorology to measure
atmospheric pressure. A simple barometer consists of a long glass tube
(closed at one end, open at the other) filled with mercury and turned
upside down into a container of mercury.
19. A
20. D
21. C The Gulf Stream is a powerful current in the Atlantic Ocean. It starts
in the Gulf of Mexico and flows into the Atlantic at the tip of Florida,
accelerating along the eastern coastlines of the United States and
Newfoundland.
22. D Sri Krishna Deva Raya was the most famous king of Vijayanagara
Empire. He belonged to the tuluva bunt community. Presiding over the
empire at its zenith, he is regarded as a hero by Tuluvas, Kannadigas
and Telugus, and one of the great kings of India.
23. D Canada
24. NOTA Justice Khehar
25. NOTA Mukul Rohatgi
26. A
27 NOTA
28. A
29. Options? Assam
30. B Pilgrimage to a holy site is a core principle of almost all faiths. The
Kaaba, meaning cube in Arabic, is a square building elegantly draped
in a silk and cotton veil. Located in Mecca, Saudi Arabia, it is the
holiest shrine in Islam.
31. B As per Hindu tradition, it was King Bhagiratha's austerities and desire
to free the souls of his ancestors that brought Holy River Ganga
32.
33. B
34. A Kidneys
35. C Thomas Cup, sometimes called the World Men’s Team
Championships, is an international badminton competition among
teams representing member nations of the Badminton World
Federation (BWF), the sport’s global governing body. The Uber Cup is
sometimes called the World Team Championships for Women.
36. D The term sub judice literally means "under judicial consideration".
The sub judice rule is part of the law relating to contempt of court. The
rule governs what public statements can be made about ongoing legal
proceedings before, principally, the courts.
37. C A volcanic crater is a funnel shaped hollow at the top of the vent. It is
formed as lava, gas and ash are blasted upwards from the main vent.
Material falls back down to earth around the vent and slowly piles up
forming a rim around it. The inside of the crater is kept clear by the
force of upward moving material constantly removing any debris
which falls there.
38. D
39. B As per the data from the Annual statements received, the highest
number of newspapers were published in Hindi (7910), followed by

10
English (1406), Urdu (938), Gujarati (761), Telugu (603), Marathi
(521), Bengali (472), Tamil (272), Oriya (245), Kannada (200) and
Malayalam (192).
40. C

1991
Question Answer Explanation
Number
1. B By virtue of Art.63, the Vice President acts as the Chairman of rajya
Sabha.
2. C Hockey.
3. C Painter. He is often referred as 'Picasso of India'.
4. A Hydrochloric acid in the stomach lowers the pH to the ideal
environment for enzymes to digest proteins into units that the
body can use. This acidic environment creates an antibacterial
environment that protects the body from disease.
5. B The Salar Jung Museum is an art museum located at Darushifa, on the
southern bank of the Musi river in the city of Hyderabad, Telangana,.
6. A
7. B Dadasaheb Phalke Award is India’s highest award in cinema. This
award was established in 1969 to commemorate the birth centenary
year of father of Indian cinema Dadasaheb Phalke. The award is
conferred annually by the Union Government to person for his
lifetime contribution to Indian cinema.
8. A
9. D
10. B Diet.
11. A Nek Chand Saini was a self-taught Indian artist, known for building
the Rock Garden of Chandigarh, an eighteen-acre sculpture garden in
the city of Chandigarh, India.
12. D Halley's Comet orbits the sun every 76 years. The comet travels in an
elliptical path, with the sun at one of the foci.
13. A Victor Hugo was a French poet, novelist and a leading figure of the
Romantic Movement in France.
14. B
15. C Refraction refers to how light rays "bend" at an interface between to
media with different indices of refraction. The rainbow is created
because the index of refraction of water droplets changes as a
function of wavelength.
16. D Barcelona(Spain). Winter in France.
17. C
18. D Common Agro forestry species. http://www.narendramodi.in/social-
forestry-a-model-sector-in-gujarat-3154
19. C
20. C Pandit Jasraj is an Indian classical vocalist. He belongs to the Mewati
gharana of Hindustani classical music. Others are writers.
21. C
22. A

11
23. A V. S. Naipaul was knighted in 1989. He was awarded the David
Cohen British Literature Prize by the Arts Council of England in 1993
and the Nobel Prize for Literature in 2001. He holds honorary
doctorates from Cambridge University and Columbia University in
New York, and honorary degrees from the universities of Cambridge,
London and Oxford.
24. C Prasar Bharati is a statutory autonomous body established under the
Prasar Bharati Act and came into existence on 23.11.1997. It is the
Public Service Broadcaster of the country. The objectives of public
service broadcasting are achieved in terms of Prasar Bharati Act
through All India Radio and Doordarshan, which earlier were
working as media units under the Ministry of I&B and since the above
said date became constituents of Prasar Bharati.
25. B (Under the given options, this has a capacity of 80000 spectators).
26. C
27. A
28. A
29. B
30. A
31. C
32. A Amritsar was established by the Fourth Guru Guru Ramdas ji in 1589
A.D. There is an evidence that he bought a land around a tank
(sarovar) from the Zamindars of Tung Villages in Jhabal District for
Rs. 700. There is another version which goes like this that Emperor
Akbar was so impressed by the work being done by third Guru
Amardas ji that he donated the land at Amritsar to the Guru ka
Langar.
33. D The National Policy on Education was framed in 1986 and modified
in 1992. Since then several changes have taken place that calls for a
revision of the Policy.
34. B As a tribute to the memory of the late Prime Minister Shri Jawaharlal
Nehru and to his life-long dedication to the cause of world peace and
international understanding, the Government of India instituted the
Jawaharlal Nehru Award for International Understanding in 1965.
35. A Mradanga. It is the most highly developed Avanadha Vadya of
Ancient India. It is barrel shaped, bulging in the middle and tapering
towards the ends. It's etymological meaning is 'earth body'.
36. C Jaiprakash Narain. Please note that A Prison Diary is also a series by
Jeffery Archer. On July 19, 2001, following a conviction for perjury,
this international bestselling author was sentenced to four years in
prison. A Prison Diary, Purgatory and Heaven are the memoirs of his
life as Prisoner FF8282--from the first three weeks of his
incarceration,
37. B Sarod
38. D Nukkad (1986-87)
39. A
40. A

1992

12
Question Answer Explanation
Number
1. C Government for the people, by the people.
2. B
3. B On 29 August, 1947, the Constituent Assembly set up a Drafting
Committee under the Chairmanship of Dr. B.R. Ambedkar to prepare
a Draft Constitution for India. While deliberating upon the draft
Constitution, the Assembly moved, discussed and disposed of as
many as 2,473 amendments out of a total of 7,635 tabled.
4. B The President is the formal head of
the executive, legislature and judiciary of India and is also
the commander-in-chief of the Indian Armed Forces.
5. C The Governor holds office during the pleasure of the President. He
can be removed by the President at any time. .
6. D The tenure of the ministry or a minister is not fixed. A
ministry/each minister remains in office so long as it enjoys the
confidence of the majority in Lok Sabha, or so long as the
Prime Minister does not resign.
7. B
8. C Public and private sectors. An economic system in which both the
private enterprise and a degree of state monopoly (usually in public
services, defense, infrastructure, and basic industries) coexist.
9. C Black money is money which is earned through any illegal activity
controlled by country regulations. Black money proceeds are usually
received in cash from underground economic activity and, as such, is
not taxed.
10. D Real income refers to the income of an individual or group after
taking into consideration the effects of inflation on purchasing power.
For example, if you receive a 2% salary increase over the previous
year and inflation for the year is 1%, then your real income only
increases by 1%. Conversely, if you receive a 2% raise in salary and
inflation is at 3%, then your real income shrinks by 1%. real income
refers to the amount of goods and services you can buy today
compared to the price of the same goods and services you could have
purchased in another time period.
11. C Between 1980-1993.
12. C Journalism, Literature, and Creative Communication Arts. Kulandie
Francis is the most recent Indian to receive it in 2012.
13. B V.K. Gokak was a major writer in the Kannada language and a
scholar of English and Kannada literatures. He was the fifth writer to
be honoured with the Jnanpith Award in 1990 for Kannada language,
for his epic Bharatha Sindhu Rashmi.
14. A Glimpses of the World History' is an account of the progress of the
world through centuries and ages. This book is a collection of letters
that Jawaharlal Nehru wrote to his daughter Indira when he was in
various Indian prisons for three years.
15. C Ban Ki-moon is the second Asian.
16. D 1952-53
17. C

13
18. C
19. B
20 B
21 A The Inca civilization flourished in ancient Peru between c. 1400 and
1533 CE, and their empire eventually extended across western South
America from Quito in the north to Santiago in the south, making it
the largest empire ever seen in the Americas and the largest in the
world at that time.
22 C Church of the Nativity and the Pilgrimage Route, Bethlehem. The
inscribed property is situated 10 km south of Jerusalem on the site
identified by Christian tradition as the birthplace of Jesus since the
2nd century.
23. C 599 BC, Vaishali
24 A
25. C The Gandhara School of Art is also known as the Graeco-Buddhist
School of Art since Greek techniques of Art were applied to
Buddhist subjects.
26. B
27. C
28. C It is a mixture.
29. B the branch of biology that deals with the relations of organisms to one
another and to their physical surroundings.
30 D Physiology is the study of normal function within living creatures. It
is a sub-section of biology, covering a range of topics that include
organs, anatomy, cells, biological compounds, and how they all
interact to make life possible.
31. C Humans have 46 chromosomes in every normal cell of the body. Sex
cells (sperms and eggs) have exactly half this number (23).
32. A Genes are probably responsible for most of an
organism's characteristics - visible and non-visible. It must be said
that the environment will also have an influence on this. Genes
effectively control everything that a cell does, and this is because
they contain the specification for proteins that cells produce - for
example enzymes, hormones and the very proteins that cells are built
from.
33. C The small particles (molecules, tiny water droplets and dust
particles) scatter photons the more, the shorter their wavelength is.
Therefore, in thescattered light, the short wavelengths predominate,
the sky appears blue, while direct sunlight is somewhat yellowish, or
even reddish when the sun is very low.
34. C
35. A
36. Not Mukul Rohatgi
applicable
(A)
37. B the policy or practice of restructuring or reforming the economic and
political system. First proposed by Leonid Brezhnev in 1979 and
actively promoted by Mikhail Gorbachev, perestroika originally
referred to increased automation and labour efficiency, but came to

14
entail greater awareness of economic markets and the ending of
central planning.
38. A
39. B
40. C

1993
Question Answer Explanation
Number
1. A
2. B Mr.Laurie Baker was a British-born Indian Architect renowned for his
initiatives in cost-effective Architecture which was very much
thoughtful on energy consumption and efficient use of energy. All his
designs stressed an emphasis on usage of local materials and the
quantity of materials he'd use in his projects were less than the
average materials consumed during the construction of the same
project by another Architect/Builder.
3. B (n.a.) Mukul Rohatgi
4. Missing
5. A
6. A
7. C On January 26, 1950, the Indian constitution takes effect, making the
Republic of India the most populous democracy in the world.
http://www.history.com/this-day-in-history/republic-of-india-born
8. C Lord Dhanvantari is the Hindu God of medicine and the doctor of
the Gods. He is also believed to be the founder of Ayurveda,
9. B
10. C Sangam literature is believed to have been authored by the poets who
were attached to the third academy which flourished in Madurai from
300 B. C to 300 A. D. The poems were composed by poets drawn
from almost all the sections of the ancient Tamil society and
belonging to different villages and towns.
11 B The Dhammapada is the best known and most widely esteemed text in
the Pali Tipitaka, the sacred scriptures of Theravada Buddhism. The
work is included in the Khuddaka Nikaya ("Minor Collection") of the
Sutta Pitaka
12. C The first version of the book was compiled by the 5th Sikh
Guru, Arjun, at Amritsar in 1604 CE. He included his own hymns and
those of his predecessors, the Gurus Nanak, Angad, Amar Das,
and Ram Das, and a selection of devotional songs of both Hindu and
Islamic saints (notably the poet Kabīr).
13. A The trinity composers lived in an area of India called Tamilnadu in the
city of Thanjavur which is considered to be the Carnatic
Music Capital.
14. B
15. B Report submitted in 1955. Rejected in 1961. Then Mandal
Commission submitted its report.
16. C for all practical purposes the term backward class is used

15
for backward castes, but the Constitution recognises special
provisions for advancement of any socially and
economically backward class.
17. C
18. C On 17th August 1947 the Radcliffe Line was declared as the boundary
between India and Pakistan, following the Partition of India. The line
is named after Sir Cyril Radcliffe
19. A The Sabarmati Ashram (also known as Harijan Ashram) was home to
Mohandas Gandhi from 1917 until 1930 and served as one of the main
centres of the Indian freedom struggle.
20. C The term tripartite system is ascribed to French Enlightenment
political philosopher Baron de Montesquieu. In The Spirit of the Laws
(1748), Montesquieu described the separation of
political power among a legislature, an executive, and a judiciary.
21 C
22 B
23 A
24. A Aga Khan founded it in 1906.
25. C
26. B Ignorance of the cultivators was one of the main reasons why it was
possible for the European planters to repress them. Gandhi therefore
set up voluntary organizations to improve the economic and
educational conditions of the people. They opened schools and also
taught the people how to improve sanitation.
The government realized Gandhi’s strength and his devotion to
causes. They themselves then set upon a committee to enquire into the
grievances of the cultivators. They invited Gandhi to serve on that
committee, and he agreed. The result was that within a few months the
Champaran Agrarian Bill was passed. It gave great relief to the
cultivators and land tenants.
27 B 1947-48, 1965, 1971 were the 3 wars.
28 B
29. C
30. B
31. C Egyptian Politician
32 B The zaire traded against the United States dollar at a seemingly
ridiculous rate of 8 million to one in October 1993; by December the
rate was a patently absurd Z110 million to the dollar and still rising.
Once again some rioting and looting occurred when opposition forces
promoted a boycott of the new currency.
33 B The worst atrocity of the Balkan Civil War took place in July of 1995
when Srebrenica, a UN-declared safe area, came under siege by forces
led by the Bosnian Serb commander Ratko Mladić. Serb forces, in an
act of genocide, executed more than 8,000 Bosnian Muslim males.
The remaining women and children were expelled from their homes.
34 C Civil war and starvation in Somalia eventually led to the deployment
of 1,700 German troops forhumanitarian relief operations as part
of UNO- SOM II
35 A

16
36 D Effective demand refers to the willingness and ability of consumers to
purchase goods at different prices.
37 A. Inflation (2016) the world economy in 2016 will continue to be
characterized by a New Abnormal in terms of output, economic
policies, inflation, and the behavior of key asset prices and financial
markets.
https://www.weforum.org/agenda/2016/02/what-is-making-today-s-
global-economy-so-abnormal
38. D He enjoys office till the pleasure of the President until a new Lok
Sabha is elected.
39. B
40 B Mohammed Iqbal.

1994
Question Answer Explanation
Number
1 B Lord Bahubali
2 A In the present day, the Gomateshwara Statue is considered as the
world’s biggest monolithic statue. The Gomateshwara Statue is a
famous pilgrim place for the Jains.
3 B Construction of the 30 Buddhist cave-shrines at Ajanta, many of
which display features of Gupta architecture.
4 A The term Nalanda means “giver of knowledge”. It was founded by
Kumaragupta I during the Gupta period. It was patronised by his
successors and later by Harsha.
5 B Kashmir
6 C
7 C Reservation of legislative seats for Scheduled Castes and Tribes.
8. B On 29 August, 1947, the Constituent Assembly set up a Drafting
Committee under the Chairmanship of Dr. B.R. Ambedkar to prepare
a Draft Constitution for India. While deliberating upon the draft
Constitution, the Assembly moved, discussed and disposed of as
many as 2,473 amendments out of a total of 7,635 tabled.
9 C
10 B The Constitution of India does not define Scheduled Tribes as such.
However, Article 366 (25) refers to Scheduled Tribes as, such tribes
or tribal communities or groups as are deemed under Article 342 of
the Constitution. According to Article 342 of the Constitution, the
Scheduled Tribes are the tribes or tribal communities that have been
declared as such by the President through a public notification.
11 B R K Shanmukhan Chetty served as the finance minister in Jawaharlal
Nehru'sCabinet between 1947-49. On November 26,
1947 Chetty presented the first budget of independent India.
12 A
13 C The Uruguay Round was the 8th round of multilateral trade
negotiations (MTN) conducted within the framework of the General
Agreement on Tariffs and Trade (GATT), spanning from 1986 to
1994 and embracing 123 countries as "contracting parties".

17
14 C Peter Sutherland, United Nations Special Representative of the
Secretary-General for International Migration and Development, is
former Director General of the World Trade Organization, EU
Commissioner for Competition, and Attorney General of Ireland.
15 B
16 A
17 B
18 B The system has three levels: gram panchayat (village level), mandal
parishad or block samiti or panchayat samiti (block level) and zila
parishad (district level).
19 B
20 A
21 B A pressure group can be described as an organised group that does
not put up candidates for election, but seeks to influence government
policy or legislation. They can also be described as 'interest groups',
'lobby groups' or 'protest groups'.
22 C economic development that is conducted without depletion of natural
resources.
23 B
24 B After the monsoons, in the winter. October.
25 C
26 D All of these.
27. C
28. B Only persons who are of unsound mind and have been declared so by
a competent court or disqualified due to ‘Corrupt Practices’ or
offences relating to elections are not entitled to be registered in the
electoral rolls.

visit: http://eci.nic.in/eci_main/ECI_voters_guideline_2006.pdf
29 C The SC is barred from deciding water disputes. For every particular
water dispute, a tribunal is set up, the decision of which is final and
binding. Only the enforcement of the award of the tribunal can be
challenged before the Court of Law.
30 B Not binding. It is an advisory body.
31 C S.R. Bommai v UOI laid down few guidelines where the President
can invoke Art 356 in a state. Failure of constitutional machinery in a
state can attract such Presidential Rule.
32 A The President can invoke Presidents rule only after a report from the
Governor stating that such Rule is the only measure that can be taken.
33 B
34 A
35 B
36 B His radical humanistic revolution achieved, not by violence or armed
insurrection, but throug process of education. M.N.Roy was very
much critical of western democracy, especially parliamentary
democracy.
37 A
38 C M.S Subbulakshmi is a name that is synonymous with the world of
Carnatic music. This flawless singer whose voice had a divine power

18
is the first singer in India to be presented with the Bharat Ratna, the
highest award given to any civilian.
39 C
40 None Goa

1995
Question Answer Explanation
Number
1 B Gommateshwara Statue is a 57-foot (17 m) high
monolithic statue located at Shravanbelagola in the Indian state of
Karnataka.
2 B
3 A
4 C Annie Besant was a prominent theosophist, women's rights activist,
writer, orator and supporter of Irish and Indian self rule
5 A
6 A
7 A
8 B
9 B
10 A
11 B A fiscal deficit occurs when a government's total expenditures exceed
the revenue that it generates, excluding money from
borrowings. Deficit differs from debt, which is an accumulation of
yearly deficits.
12 C
13 B reduction of the general level of prices in an economy.
14 A
15 D
16 C Art. 370 talks about attribution of special status to Jammu and
Kashmir
17 B
18 C But now there are 193 countries.
19 B
20 A
21 B
22 C
23 B
24 A For over nine years until 2003, Dr. Kasturirangan steered the Indian
Space programme as Chairman of the Indian Space Research
Organisation (ISRO).
25 C Art 1 of the Indian Constitution describes India as a Union of States.
26 B This doctrine, as proposed by Montesque, divides the powers into 3
parts: Judiciary, Legislative and Executive Powers.
27 C Art 22 talks about such Preventive detention
28 B The President of India (based on a recommendation from incumbent
Govt of India) appoints the Chief Election Commissioner.
Conventionally, senior-most Election Commissioner is appointed as

19
CEC. He has tenure of six years, or up to the age of 65 years,
whichever is earlier
29 C Legislative determination of guilt. E.g. in case of impeachment of
President, both the Houses vote.
30 C(n.a.) In 2014, Sikkim Chief Minister Pawan Chamling's consecutive fifth
term beat the record set by Jyoti Basu in West Bengal.
31 B
32 C
33 A
34 A
35 B the procedure by which a state or nation, upon receipt of a formal
request by another state or nation, turns over to that second
jurisdiction an individual charged with or convicted of a crime in that
jurisdiction.
36 B
37 C By way of Notification
38 B
39 D Optional and open to only member states
40 B

1996
Question Answer Explanation
Number
1 B
2 C Mount of the dead
3 B Rajatarangini, historical chronicle of early India, written in Sanskrit
verse by the Kashmiri Brahman Kalhana in 1148, that is justifiably
considered to be the best and most authentic work of its kind. It covers
the entire span of history in the Kashmir region from the earliest times
to the date of its composition.
4 B In 1996, Kolkata Mint releasd commemorative of S.C. Bose, birth
centenary until they realised they were a year earlier. The actual
cetenary was in 1997.
5 B Government of India Act 1935
6 C members were chosen by indirect election by the members of the
Provincial Legislative Assemblies, according to the scheme
recommended by the Cabinet Mission. The arrangement was: (i) 292
members were elected through the Provincial Legislative Assemblies;
(ii) 93 members represented the Indian Princely States; and (iii) 4
members represented the Chief Commissioners' Provinces.
7 B
8 C Right to property 19(1)(f) was removed from fundamental freedoms.
9 C He can send it back but must give assent to it when it is subsequently
resent.
10 B
11 A
12 C Entry 46, List 2
13 C Art 123 allows the President to promulgate an Ordinance when either

20
or both the Houses are not in session.
14 C Division of power between central and state government and
providing state autonomy is federalism.
15 C Indian Medical Association v V.P. Santha
16 D
17 n.a. Robert Azevado
18 B Dayton Accords, peace agreement reached on Nov. 21, 1995, by the
presidents of Bosnia, Croatia, and Serbia, ending the war in Bosnia
and outlining a General Framework Agreement for Peace in Bosnia
and Herzegovina. It preserved Bosnia as a single state made up of two
parts, the Bosniak-Croat federation and the Bosnian Serb Republic,
with Sarajevo remaining as the undivided capital city.
19 B
20 C In the summer of 1995, six foreign tourists were kidnapped in a
meadow in Kashmir by a group calling itself Al Faran. One, an
American, escaped. A second, a Norwegian, was found with his head
severed from his body.

21 C
22. A
23 C
24 B
25 B
26 B
27 C Nepal's Supreme Court ordered in August, 1995 the nation's
Parliament reinstated, calling on the government and the speaker of
the House of Representatives to convene a special session of to
discuss a no-confidence motion against the government. Regular
sessions of Parliament were also ordered. King Birendra, on the
recommendation of Nepal's first elected Communist government.
28 A
29 B
30 C COBOL (Common Business Oriented Language) was the first widely-
used high-level programming language for business applications.
31 A The standard meridian of India is 82.5 degrees east of the Greenwich
Meridian. This meridian passes through the city of Allahabad.
32 D 1995-1999.
33 C South Korea. Masterminding a 'creeping coup'.
34 C
35 B Garry Kasparov, the defending champion, played Viswanathan
Anand, the challenger, in a twenty-game match. Kasparov won the
match after eighteen games with four wins, one loss, and thirteen
draws.
36 B A cryogenic engine/ cryogenic stage is the last stage of space launch
vehicles which makes use of Cryogenics.
37 A(n.a.) 2018: Jakarta`
38 C
39 A

21
40 B

1997
Question Answer Explanation
Number
1 A Chandragupta Maurya defeated the Macedonian satrapies in the
northwest of the Indian subcontinent in 317 BC. Thereafter, he
turned his attention towards Northwestern India. Chandragupta
allied with the Himalayan king Parvatka and launched an attack
against Dhana Nanda of the Nanda Empire. The battle ended
around 321 BC, with the siege of the capital city of Kusumapura
and the conquest of the Nanda Empire. Thus was born the
powerful Maurya Empire in Northern India.
2 D It is important to note that:
In April 2002, the Netherlands became the first country to legalise
euthanasia and assisted suicide. It imposed a strict set of
conditions: the patient must be suffering unbearable pain, their
illness must be incurable, and the demand must be made in "full
consciousness" by the patient.
However, From the Voluntary Euthanasia Society of England and
WalesIn 1995, the Northern Territory of Australia became the
first legislature in the world to pass a law for voluntary
euthanasia. This came into effect on July 1st 1996.
3 A The Sardar Sarovar Dam is a gravity dam on the Narmada River
near Navagam,Gujarat in India. The construction and
development of Sardar Sarovar Dam lie between three states of
India. They are Gujarat, Maharashtra, and Madhya Pradesh.
4 C
5 B Kasuti is a traditional form of folk embroidery practised in the
state of Karnataka, India. Kasuti work which is very intricate
sometimes involves putting up to 5,000 stitches by hand and is
traditionally made on dresswear like Ilkal sarees, Ravike and Angi
or Kurta.
6 C
7 A The name of the book is 'The Degeneration of India'.
8 Non Galileo was ordered to turn himself in to the Holy Office to begin
determinative trial for holding the belief that the Earth revolves around the Sun,
options which was deemed heretical by the Catholic Church. Standard
practice demanded that the accused be imprisoned and secluded
during the trial.
9,10 Missing
11 D Maori are the indigenous people of Aotearoa New Zealand.
Contemporary Maori culture has been shaped by the traditions of
its rich cultural heritage. It is an integral part of kiwi life.
12 A Constitution of India was adopted by the Constituent Assembly
on 26th November, 1949 and came into force on 26th January,
1950.
13 A (in Japan) a conglomeration of businesses linked together by

22
cross-shareholdings to form a robust corporate structure.
14 B
15 D  Asynchronous Transfer Mode is a high speed network standard
designed to support voice and data communications. It is used in
computer network, mostly used for Wireless Area Network
(WAN). It splits the data into a packet of a fixed size and due to
its complex nature is able to guarantee more accurately regarding
data range, delays, jitters etc.
16 C  Mr. Annan joined the UN system in 1962 as an administrative and
budget officer with the World Health Organization in Geneva . He
later served with the Economic Commission for Africa in Addis
Ababa , the UN Emergency Force (UNEF II) in Ismailia , the
United Nations High Commissioner for Refugees (UNHCR) in
Geneva , and in various senior posts in New York dealing with
human resources, budget, finance and staff security. Immediately
before becoming Secretary-General, he was Under-Secretary-
General for Peacekeeping
17 C 
18 A  1997.
19 C  35th Amendment
20 C 
21 A 
22 n.a, 1570 as of 2014
23 B
24 C
25 A
26 D
27 B The three lists divide the subject matters on which State and Union
can make laws. These decide legislative competence of Centre/
State on a particular subject matter. The Concurrent List gives
subject maters on which both can make laws. Important Articles
246, 246 and 254.
28 C Arya Samaj is an Indian religious movement that promotes values
and practices based on the infallibility of the Vedas. The sect was
founded by Dayananda Saraswati, a sannyasi, on 7 April 1875.
29 A The Human Genome Project (HGP) was the international,
collaborative research program whose goal was the complete
mapping and understanding of all the genes of human beings. All
our genes together are known as our "genome."
30 D South Korea
31 B This Article bars discrimination but allows 'positive
discrimination'.
32 B
33 A This treatment is used to preserve food, reduce the risk of
food borne illness, prevent the spread of invasive pests, and delay
or eliminate sprouting or ripening. Irradiated food does not become
radioactive. The radiation can be emitted by a radioactive
substance or generated electrically.
34 C

23
35 C
36 D
37 C
38 n.a 56.6%
39 D Sarojini Naidu was a prolific writer and poet. The first volume of
her poetries The Golden Threshold was published in 1905, after
which two more collections The Bird of Time and The Broken
Wing arrived in 1912 and 1917 respectively.
40 B

1998
Question Answer Explanation
Number
01 B Sir Chandrasekhara Venkata Raman was an Indian physicist born in the
former Madras Province in India, who carried out ground-breaking
work in the field of light scattering, which earned him the 1930 Nobel
Prize for Physics.
02 C Mars Pathfinder is an American robotic spacecraft that landed a base
station with a roving probe on Mars in 1997.
03 D Sir Ian Wilmut, OBE FRS FMedSci FRSE is a British embryologist and
Chair of the Scottish Centre for Regenerative Medicine at the University
of Edinburgh.
04 C
05 B
06 A A common procedure for DNA fingerprinting is restriction fragment
length polymorphism (RFLP). In this method, DNA is extracted from a
sample and cut into segments using special restriction
enzymes. RFLP focuses on segments that contain sequences of repeated
DNA bases, which vary widely from person to person.
07 C
08 C Dario Fo is an Italian actor-playwright, comedian, singer, theatre
director, stage designer, songwriter, painter and political campaigner of
the italian extremist left-wing, recipient of the 1997 Nobel Prize in
Literature.
09 C Jody Williams (born 1950) is an American political activist known
around the world for her work in banning anti-personnel landmines,
her defense of human rights – especially those of women – and her
efforts to promote new understandings of security in today’s world. She
was awarded the Nobel Peace Prize in 1997 for her work toward the
banning and clearing of anti-personnel mines.
10 C
11 D
12 C
13 C The Maastricht Treaty (formally, the Treaty on European Union or
TEU) undertaken to integrate Europe was signed on 7 February 1992 by

24
the members of the European Community in Maastricht, Netherlands.
On 9–10 December 1991, the same city hosted the European Council
which drafted the treaty.
14 B The curve suggests that, as taxes increase from low levels, tax revenue
collected by the government also increases. It also shows that tax rates
increasing after a certain point (T*) would cause people not to work as
hard or not at all, thereby reducing tax revenue.
15 D The Bombay Plan is the name commonly given to a World War II-era
set of proposals the development of the post-independence economy of
India.
16 D
17 C
18 B
19 B
20 A
21 C
22 C The Kesavananda Bharathi judgement or His Holiness Kesavananda
Bharati Sripadagalvaru and Ors. v. State of Kerala and Anr. (case
citation: (1973) 4 SCC 225) is a landmark decision of the Supreme
Court of India that outlined the Basic Structure doctrine of
the Constitution.[2] Justice Hans Raj Khanna asserted through this
doctrine that the constitution possesses a basic structure of constitutional
principles and values. The Court cemented the prior
precedent Golaknath v. State of Punjab AIR 1967 SC 1643, which held
that constitutional amendments pursuant to Article 368 were subject to
fundamental rights review.
The Basic Structure doctrine forms the basis of power of
the Indian judiciary to review, and strike down amendments to the
Constitution of India enacted by the Indian parliament which conflict
with or seek to alter this basic structure of the Constitution.
The 13-judge Constitutional bench of the Supreme Court deliberated on
the limitations, if any, of the powers of the elected representatives of the
people and the nature of fundamental rights of an individual. In a
sharply divided verdict, by a margin of 7-6, the court held that while the
Parliament has "wide" powers, it did not have the power to destroy or
emasculate the basic elements or fundamental features of the
constitution.
Although the court upheld the basic structure doctrine by only the
narrowest of margins, it has since gained widespread acceptance and
legitimacy due to subsequent cases and judgments. Primary among
these was the imposition of the state of emergency by Indira
Gandhi in 1975, and the subsequent attempt to suppress her prosecution
through the 39th Amendment. When the Kesavananda case was
decided, the underlying apprehension of the majority bench that elected
representatives could not be trusted to act responsibly was perceived to
be unprecedented. However, the passage of the 39th Amendment proved

25
that in fact this apprehension was well-founded. In Indira Nehru Gandhi
v. Raj Narain, a Constitutional Bench of the Supreme Court used the
basic structure doctrine to strike down the 39th amendment and paved
the way for restoration of Indian democracy
The Kesavananda judgment also defined the extent to which Parliament
could restrict property rights, in pursuit of land reform and the
redistribution of large landholdings to cultivators, overruling previous
decisions that suggested that the right to property could not be
restricted. The case was a culmination of a series of cases relating to
limitations to the power to amend the Indian constitution.
23 C In the Republic of India, the phrase "President's rule" refers to the
imposition of Article 356 of the Constitution of India on a State whose
constitutional machinery has failed. In the event that a State
government is not able to function as per the Constitution, the State
comes under the direct control of the central government; in other
words, it is "under President's rule". Subsequently, executive authority
is exercised through the centrally appointed Governor, who has the
authority to appoint retired civil servants or other administrators to
assist him.
On the other hand, when the State government is functioning normally,
it is run by an elected Council of Ministers, who are collectively
responsible to State's legislative assembly (Vidhan Sabha). The Council
is headed by the Chief Minister who is the de facto chief executive of
the State; the Governor is only a de jure constitutional head. However,
during President's rule, the Council of Ministers stands dissolved, the
office of Chief Minister becomes vacant and the Vidhan Sabha is either
put in suspended animation or dissolved (necessitating a fresh election).
In the state of Jammu and Kashmir, failure of constitutional machinery
results in Governor's rule, imposed by invoking Section 92 of
the Constitution of Jammu and Kashmir. The proclamation is issued by
the state's Governor after obtaining the consent of the President of India.
If it is not possible to revoke Governor's rule before within six months
of imposition, President's Rule under Article 356 of the Indian
Constitution is imposed. There is little practical difference between the
two provisions.
Following its landmark judgment in the 1994 Bommai case,
the Supreme Court of India has clamped down on arbitrary impositions
of President's rule by central governments. Chhattisgarh and Telangana
are the only states where President's rule hasn't been imposed so far.
24 B
25 A
26 A
27 A
28 A Sarkaria Commission was set up in June 1983 by the central
government of India. The Sarkaria Commission's charter was to
examine the relationship and balance of power between state and central

26
governments in the country and suggest changes within the framework
of Constitution of India.
29 B
30 A
31 B
32 C The Ghauri–I (Urdu: ‫غوری‬- ; official codename: Hatf–5 Ghauri–I) is
a land-based surface-to-surface medium-range ballistic missile, in
current service with the Army's Strategic Forces Command— a
subordinate command of Strategic Plans Division.
33 C
34 B
35 B Nirad Chandra Chaudhuri was an Indian Bengali−English writer and
man of letters. He was born in a Hindu family in 1897 in Kishoreganj,
then part of Bengal in British India, now in Bangladesh. Chaudhuri
authored numerous works in English and Bengali.
36 C
37 C
38 A Murlidhar Devidas Amte, popularly known as Baba Amte was an Indian
social worker and social activist known particularly for his work for the
rehabilitation and empowerment of poor people suffering from leprosy.
39 B
40 C Lieutenant-General Mikhail Timofeyevich Kalashnikov was a Russian
general, inventor, military engineer, writer and small arms designer.

1999
Question Answer Explanation
number
01 A Justice M. Fathima Beevi was the first female judge to be appointed to
the Supreme Court of India (1989) and the first Muslim woman to be
appointed to any higher judiciary. She is the first woman judge of a
Supreme Court of a nation in Asia. On her retirement from the court she
served as a member of the National Human Rights Commission and
as Governor in Tamil Nadu (1997–2001).
02 B
03 C
04 B
05 B
06 B Verghese Kurien was an Indian social entrepreneur who is known as the
Father of White Revolution in India for his Operation Flood, the world's
largest agricultural development programme.
07 A
08 A V. Ramaswami was the Justice of the Supreme Court of India and the
first judge against whom impeachment proceedings were initiated in
independent India.

27
09 A
10 B
11 A
12 C
13 A
14 A
15 A
16 A Admiral Vishnu Bhagwat, PVSM , AVSM is a former Chief of the
Naval Staff of India. A talented and all-round officer, he had a brilliant
career in the Indian Navy, eventually achieving the highest rank.He is
the first and only chief of naval staff who was relieved from office while
still serving and stripped of Admiral rank as punitive measure.
17 B
18 A
19 D
20 B
21 A
22 A
23 D
24 C
25 D
26 A
27 A HQ’s at Kolkata.
28 A
29 C
30 A Homi Jehangir Bhabha was an Indian nuclear physicist, founding
director, and professor of physics at the Tata Institute of Fundamental
Research.
31 D
32 A
33 A
34 C
35 C
36 D
37 B
38 B
39 D
40 B
41 B
42 D
43 C
44 C
45 D

28
2000
Question Answer Explanation
number
01 D He/She only votes in case of a tie or else he/she doesn’t participate in
voting process.
02 A Bank Rate refers to the official interest rate at which RBI will provide
loans to the banking system which includes commercial / cooperative
banks, development banks etc. Such loans are given out either by direct
lending or by rediscounting (buying back) the bills of commercial banks
and treasury bills. Thus, bank rate is also known as discount rate.
03 D
04 C By virtue of Art 280, it is the duty of the Finance Commission to
distribute the net proceeds of taxes between the Centre and the States
and the proportions of the States.
05 A East Timor, or Timor-Leste, a Southeast Asian nation occupying half
the island of Timor, is ringed by coral reefs teeming with marine life.
Landmarks in the capital, Dili, speak to the country's struggles for
independence from Portugal and then Indonesia. The iconic 27m-tall
Cristo Rei de Dili statue sits on a hilltop high over the city, with
sweeping views of the surrounding bay.
06 C Where Art. 148-151 provide for the office of Comptroller and Auditor
General, Art. 76 provides for the Attorney General and Art 165 provides
for Advocate General, the Consitution does not provide expressly for
the Solicitor General. Solicitor General instead assists the Attorney
General.
07 D
08 D
09 C The party of the opposition always conducts parliamentary Accounts
Committee.
10 A In USA, the primary election is where the registered voters participate
in choosing the candidate for the party's nomination by voting. There
are two kinds of primaries, open and closed which determines who is
eligible to vote in the primary. In a closed primary, a registered voter
may vote only in the election party with which the voter is affiliated.
Visit the link to further understand the functioning of Primary:
https://votesmart.org/education/presidential-primary#.WDvTBLJ97IU
11 C The Indian constitution guarantees right to equality Art.14) and right to
personal liberty (Art.21) to aliens as well.
12 C Dyarchy was introduced as a constitutional reform by Edwin Samuel
Montagu(secretary of state for India, 1917–22) and Lord
Chelmsford (viceroy of India, 1916–21), through Government of India
Act, 1919. The principle of dyarchy was a division of the executive
branch of each provincial government into authoritarian and popularly
responsible sections. The first was composed of executive councillors,
appointed, as before, by the crown. The second was composed of
ministers who were chosen by the governor from the elected members

29
of the provincial legislature.
13 D India follows single citizenship borrowed from the UK Model.
14 D
15 A
16 B
17 A
18 C The National Renewal Fund was abolished by the government in 2000
and administration of the VRS scheme was shifted to Department of
Public Enterprises (DPE), instead of DIPP from fiscal year 2001-02.
19 D Dividend Tax, Capital Gains, Intrest Tax are all form of Indirect Tax.
20 D

2001
Question Answer Explanation
number
01 C India shares its boundary with 7 countries
Bangladesh, China, Pakistan, Nepal, Myanmar, Bhutan, Afganistan.
02 A In November 2000
Three states were added
26. Chattisgarh (1/11/2000)
27. Uttarakhand (09/11/2000)
28. Jharkhand (15/11/2000)
03 A
04 A
05 B
06 B
07 B
08 C
09 B
10 D
11 D As of 2016, 16 Lok Sabha elections have
12 C  First Plan (1951–1956)

 Second Plan (1956–1961)


 Third Plan (1961–1966)
 Fourth Plan (1969–1974)
 Fifth Plan (1974–1979)
 Rolling Plan (1978–1980)
 Sixth Plan (1980–1985)
 Seventh Plan (1985–1990)
 Annual Plans (1990–1992)

30
 Eighth Plan (1992–1997)
 Ninth Plan (1997–2002)
 Tenth Plan (2002–2007)
 Eleventh Plan (2007–2012)
 Twelfth Plan (2012–2017)
13 A The Sardar Sarovar Dam is a gravity dam on the Narmada
river near Navagam, Gujarat in India. It is the largest dam and part of
the Narmada Valley Project, a large hydraulic engineering project
involving the construction of a series of
large irrigation and hydroelectric multi-purpose dams on the Narmada
river. The project took form in 1979 as part of a development scheme to
increase irrigation and produce hydroelectricity.
14 A The Constitution was adopted on 26th November, 1949 but it came into
effect from 26 January 1950.
15 D
16 A The present President of Pakistan is Mamnoon Hussain.
17 C The planet with the most moons in the Solar System is Jupiter, with a
total of 63 confirmed moons (as of 2009), The planet with second
highest number of moons is Saturn.
18 A
19 C
20 A Human Rights Day is observed every year on 10 December. It
commemorates the day on which, in 1948, the United Nations General
Assembly adopted the Universal Declaration of Human Rights. In 1950,
the Assembly passed resolution 423 (V), inviting all States and
interested organizations to observe 10 December of each year as Human
Rights Day.
21 C
22 D
23 A
24 B
25 C She won the medal in weight lifting competition.
26 C
27 B First Batsmen to score more than 10000 runs
Test Cricket – Sunil Gavaskar (India)
One Day Cricket - Sachin Tendulkar (India)
28 D
29 C
30 A
31 B
32 C
The term is most associated with Patty Hearst, the Californian
newspaper heiress who was kidnapped by revolutionary militants in
1974. She appeared to develop sympathy with her captors and joined

31
them in a robbery. She was said to be suffering with"Stockholm
Syndrome" - a term that explains the apparently irrational feelings of
some captives for their captors.
33 B
34 C
35 D
36 C
37 A
38 A 52 Seconds
39 D
40 B
41 A
42 A Gujarat

2002
Question Answer Explanation
number
01 B Hind Swaraj or Indian Home Rule is a book written by Mohandas K.
Gandhi in 1909. In it he expresses his views on Swaraj, modern
civilization, mechanisation etc.
02 C
03 B
04 B
05 C Also called as Frontier Gandhi.
06 C
07 A
08 C Right to property has been removed as fundamental right and now is a
constitutional right by the 44rth Amendment 1978
09 C
10 D
11 A
12 A The meaning of the Rolling Plan was that now, every year the
performance of the plan will be assessed and a new plan will be made
next year based upon this assessment. In the rolling plans there are three
kind of plans. First is the plan for the current year which comprises the
annual budget. Second is a plan for a fixed number of years, which may
be 3, 4 or 5 years. This second plan is kept changing as per the
requirements of the economy (and politics). Third is a perspective plan
which is for 10, 15 or 20 years. Thus, there is no fixation of dates in
respect of commencement and end of the plan in the rolling plans.
13 D
14 D
15 A
16 D

32
17 B
18 A
19 B
20 B
21 C
22 B
23 B
24 B
25 B 2016: New York City declared this year’s April 14 as ‘Bindeshwar
Pathak Day’ in recognition of the contributions made by the Indian
social activist and ‘Sulabh International’ founder for improving the lives
of people engaged in the “most dehumanising situation”.
New York City Mayor Bill de Blasio honoured Pathak for his outstanding
work to improve health and hygiene and ‘moving the world forward’.
26 A
27 C
28 A
29 A Basketball
30 D
31 C The Copyright Act, 1957 came into effect from January 1958. This Act
has been amended five times since then, i.e., in 1983, 1984, 1992, 1994,
1999 and 2012. The Copyright (Amendment) Act, 2012 is the most
substantial. The main reasons for amendments to the Copyright Act,
1957 include to bring the Act in conformity with WCT and WPPT; to
protect the Music and Film Industry and address its concerns; to address
the concerns of the physically disabled and to protect the interests of the
author of any work; Incidental changes; to remove operational facilities;
and enforcement of rights. Some of the important amendments to the
Copyright Act in 2012 are extension of copyright protection in the
digital environment such as penalties for circumvention of technological
protection measures and rights management information, and liability of
internet service provider and introduction of statutory licences for cover
versions and broadcasting organizations; ensuring right to receive
royalties for authors, and music composers, exclusive economic and
moral rights to performers, equal membership rights in copyright
societies for authors and other right owners and exception of copyrights
for physically disabled to access any works.
32 C The Kyoto Protocol is an international agreement linked to the United
Nations Framework Convention on Climate Change, which commits its
Parties by setting internationally binding emission reduction targets.
33 C As of June 2016 , human euthanasia is legal in
the Netherlands, Belgium, Colombia, and Luxembourg. Assisted suicide
is legal in Switzerland, Germany, Japan, Canada, and in the US states of
Washington, Oregon, Vermont, Montana, and California.
34 C
35 D

33
36 B
37 A
38 B The Laffer Curve is a theory developed by supply-side economist
Arthur Laffer to show the relationship between tax rates and the amount
of tax revenue collected by governments. The curve is used to
illustrate Laffer's main premise that the more an activity such as
production is taxed, the less of it is generated.
39 A
40 A

2003
Question Answer Explanation
number
01 C El Niño is a climate cycle in the Pacific Ocean with a global impact on
weather patterns. The cycle begins when warm water in the western
tropical Pacific Ocean shifts eastward along the equator toward the
coast of South America. Normally, this warm water pools near
Indonesia and the Philippines.
02 B
03 B Constituent Assembly was formed in the year 1946, consisting of 389
members.
04 D
05 A
06 C
07 C
08 B
09 B
10 B
11 B
12 B
13 B Our Research team finds the question to be factually incorrect the Art.
21-A was added by the 86th amendment 2002.
14 C
15 C
16 B Netherlands became the first country to legalise Ethunasia.
17 A
18 B
19 D Genocide is the intentional action to systematically eliminate
an ethnic, national, racial, or religious group. The word is a combination
of "genos" (race, people) and "cide" (to kill). The United
Nations Genocide Convention defines it as "acts committed with intent
to destroy, in whole or in part, a national, ethnical, racial or religious
group"
20 A
21 D Indira Gandhi was the first female to receive Bharat Ratna in 1971

34
Mother Teresa received it in 1980
M.S. Subbalaxmi received it in 1998
Lata Mageskar in 2001.
22 B
23 B
24 B Racism is a belief or doctrine that inherent differences among the
various human racial groups determine cultural or individual
achievement, usually involving the idea that one's own race is superior
and has the right to dominate others or that a particular racial group is
inferior to the others.
25 C
26 B The first amendment to the Indian Constitution added the Ninth
Schedule to it. It was introduced by the Nehru Government, on 10 May
1951 to address judicial decisions and pronouncements especially about
the chapter on fundamental rights. Nehru was also very clear on the
purpose behind the first amendment. The state wanted to pursue
nationalisation, take away lands from the zamindars, re-distribute them,
and make special provisions for the socially and economically
backward.
27 D
28 D Justice Venkatchaliah
29 D
30 B
31 C Right to property was removed as a fundamental right and added as
legal right by the 44rth Amendment 1978.
32 B Palk Strait
33 C It’s a group of of mystic minstrels from Bengal which includes Indian
State of West Bengaland the country of Bangladesh. Bauls constitute
both that of a syncretic religious sect and a musical tradition. Bauls are a
very heterogeneous group, with many sects, but their membership
mainly consists of Vaishnava Hindus and sufimuslims. They can often
be identified by their distinctive clothes and musical instruments. Not
much is known of their origin. Lalon Fokir is regarded as the most
important poet-practitioner of the Baul tradition. Baul music had a great
influence on Rabindranath Tagore's poetry and on his music (Rabindra
Sangeet).

34 B
35 A
36 D Kindly refer to explanation no. 21 in this sheet.
37 B M. C. Mehta
38 B
39 D Montenegro
40 C Harvanshrai Bachhan father of famous actor Amitabh Bachhan.

35
2004
Question Answer Explanation
number
01 C South Korea
02 D
03 D The 2008 ICC Under-19 Cricket World Cup was held in Malaysia from
17 February 2008 to 2 March 2008. The opening ceremony took place
on 15 February 2008. The final was played between South Africa and
India, which India won by 12 runs by D/L Method.
04 A
05 B
06 B
07 B
08 C
09 B Justice M. Fathima Beevi was the first female judge to be appointed to
the Supreme Court of India (1989) and the first Muslim woman to be
appointed to any higher judiciary. She is the first woman judge of a
Supreme Court of a nation in Asia. On her retirement from the court she
served as a member of the National Human Rights Commission and
as Governor in Tamil Nadu (1997–2001).
10 A
11 C
12 D The final of the 2007 ICC Cricket World Cup was played on 28 April
2007 at the Kensington Oval, Barbados. Australia defeated Sri Lanka by
53 runs (under the Duckworth–Lewis method) to win their 4th World
Cup and 3rd in succession.
13 B Law day is celebrated on the event of our constitution being enacted on
26 November 1949.
14 C Law day is celebrated on the event of our constitution being enacted on
26 November 1949.
15 A
16 D
17 D Human Rights day is celebrated on 10 December every year.
18 B Our Research team finds the question to be factually incorrect.
19 B
20 B Sucheta Kriplani (25 June 1908 – 1 December 1974) was
an Indian freedom fighter and politician. She was India's first
woman Chief Minister, serving as the head of the Uttar Pradesh
government from 1963-67.
21 A
22 C
23 D SAARC Preferential Trading Arrangement (SAPTA)
In December 1991, the Sixth Summit held in Colombo approved the
establishment of an Inter-Governmental Group (IGG) to formulate an
agreement to establish a SAARC Preferential Trading Arrangement

36
(SAPTA) by 1997. Given the consensus within SAARC, the Agreement
on SAPTA was signed on 11 April 1993 and entered into force on 7
December 1995 well in advance of the date stipulated by the Colombo
Summit. The Agreement reflected the desire of the Member States to
promote and sustain mutual trade and economic cooperation within the
SAARC region through the exchange of concessions.
24 B
25 D Financial emergency (Art. 360) has never been imposed in India till
date.
26 C A.O. Hume
27 D
28 D 0 degrees Fahrenheit is equal to -17.77778 degrees Celsius: (0 °F = -
17.77778 °C)
The temperature T in degrees Celsius (°C) is equal to the
temperature T in degrees Fahrenheit (°F) minus 32, times 5/9
29 B
30 C
31 D
32 D The Ninety-second Amendment of the Constitution of India, officially
known as The Constitution (Ninety-second Amendment) Act,
2003, amended the Eighth Schedule to the Constitution so as to
include Bodo, Dogri, Santhali and Maithali languages
33 C
34 C
35 D
36 B
37 A Lord Mountbatten became the first Governor-General of Independent
India. Whereas C. Rajgopalachari became the first Indian Governor-
General of Independent India.
38 A RTE was added by 86th amendment 2002 as Fundamental right under
article 21-A.
39 A Jharkhand became the 28 state added in the Indian Union on 15
November 2000.
40 D

2005
Question Answer Explanation
number
01 B 1981
02 C A Labour below the age of 14 Years is considered to be child labour.
03 A Right to property was removed as a fundamental right and added as
legal right by the 44rth Amendment 1978.
04 B
05 D

37
06 A
07 C Saudi Arabia
08 A
09 D
10 C Multi stands for many hence national for nation hence Many-Nations
(Multi National).
11 B As per Archimedes’ Law the weight of the solid submerged in a liquid
is directly proportional to water displaced by it hence the level of water
shall not change.
12 D M. S. Subbalaxmi was conferred the Bharat Ratna in 1998 whereas Lata
Mageshkar it was in 2001.
13 C
14 A
15 C Termination of the session of the parliament.
16 D 1992
17 B SAARC South Asian Association for regional co-operation.
18 B
19 C
20 B Dr. Rajendra Prasad was the declared as the president of the constituent
assembly on 11 of December 1946.
21 D
22 C
23 Our research team finds the question to be factually in correct. The
correct answer is Padma Bhushan.
In January 2005, she declined the Padma Bhushan awarded by the
Indian Government. In a letter to President A P J Abdul Kalam, she said
she was "astonished to see her name in the list of awardees because
three months ago when I was contacted by the HRD ministry and asked
if I would accept an award, I made my position very clear and explained
my reason for declining it". Thapar had declined the Padma Bhushan on
an earlier occasion, in 1992. To the President, she explained the reason
for turning down the award thus: "I only accept awards from academic
institutions or those associated with my professional work, and not state
awards".
24 A
25 D March 8 and December 2 respectively.
26 A 10 November 1989.
27 B
28 A Physics
Chemistry
Literature
Medicine
World Peace
Economics
29 B

38
30 D 350th Anniversary of Taj Mahal at Agra.

2006
Question Answer Explanation
number
01 A Our Research team found the question to be factually incorrect the
Highest wicket taker for India is Anil Kumble with 619 wickets, Kapil
Dev has 434 wickets.
02 B
03 B USA controls the internet all over the world.
04 D
05 A
06 C A Member of Parliament cannot hold any office of profit. Hence Sonia
Gandhi resigned as M.P.
07 C Under Part IV-A, Article 51-A (e) fundamental duties of the
constitution.
08 C Under Part IV, Article 39-A (DPSP) of the constitution.
09 B
10 C
11 C
12 C Secular stands to say that state has no religion of its own all religion are
equal before it.
13 A
14 C
15 B
16 C
17 A
18 D
19 C
20 B
21 C
22 A
23 B
24 B
25 C
26 A Preamble was amended (only once) by 42 amendment 1976.
27 D
28 C
29 A
30 A
31 C
32 A
33 C
34 C

39
35 C
36 B
37 A
38 A
39 D
40 B

2007
Question Answer Explanation
number
01 B
02 A
03 C As per the year 2007 Jyoti basu shall be the correct option however now
its Sikkim Chief Minister Pawan Chamling has become the longest
serving chief minister in India, breaking Jyoti Basu's record of 23 years.
04 A Afghanistan became the 8rth member of SAARC along with
Bangladesh, Bhutan, India, Maldives, Nepal, Pakistan and Sri Lanka.
05 D
06 A
07 C
08 B
09 C
10 D
11 B
12 A
13 C
14 D
15 B Parliament expelled 11 members — 10 from the Lok Sabha and one
from the Rajya Sabha — whose conduct was found to be "unethical and
unbecoming" of Members of Parliament. For the first time in the annals
of Parliament, the membership of the 11 MPs was terminated by voice
vote, 11 days after the sting operation on the cash-for-questions scam hit
the headlines. The Rajya Sabha agreed with the recommendation of its
Ethics Committee while the Lok Sabha endorsed the report of the
Pawan Kumar Bansal Committee set up to go into the allegations.
16 A
17 B
18 B India is third largest producer after USA & China.
19 A
20 C Shilpa Shetty in 2007
21 B
22 A
23 A She was chief minister of UP
24 B She was Governor of UP

40
25 D
26 C Sania Mirza (Tennis) Padma Shri 2006 at the age of 19.
27 B
28 B As per the year 2007 its 94, Currently in 2016 its 100 amendments.
29 A A. B. Vajpayee was the PM for mere 13 days in 1996.
30 C
31 A
32 B As per the year 2007 the answer is 21 whereas now it’s 24 High Courts.
33 D
34 C
35 A
36 D
37 B
38 A
39 D
40 D
41 C
42 B
43 A
44 B
45 A
46 B
47 D
48 D
49 C
50 A Longest with Bangladesh and shortest with Afghanistan.

NALSAR

1998
Question Answer Explanation
number
21 D The answer might be correct for that year however by the Sixth pay
commission it has been further raised three times to 1,00,000.00 per
Month.
22 C DrRajendra Prasad was the first President of India and held this
office for two successive terms (1950-1957, 1957-1962,total 12
years). He was also the President of the Constituent Assembly (11
December 1946), President of Congress in 1939 , 1943 and 1947.He
was also awarded Bharat Ratna in 1962. He spent the last few
months of his life in retirement at the Sadaqat Ashram in Patna.
23 A According to Article 143, President can consult the Supreme

41
court about the constitutional validity of any issue.
24 A Rajasthan (Nagaur District on Oct 2, 1959).In the history of
Panchayati Raj, in India, on 24 April 1993, the Constitutional (73rd
Amendment) Act 1992 came into force to provide constitutional
status to the Panchayati Raj institutions It has 3 levels: village, block
and district.
25 D The concept of Eight-fold path forms the theme ofDhrama Chakra
PravartanaSutta.AsthagikaMarg consists Right
faith,Rightaction,Rightthought,Rightlivelihood,Righteffort,Rightspee
ch,Rightrememberence and Right concentration
26 A Alberuni was a great philosopher,mathematician,astrologist and
historian of Ghazni(Afganistan).In 1017 he traveled to the Indian
subcontinent and authored “Tarikh Al-Hind”orkitab-ul hind (History
of India) in Arabic language
27 B Taxila was a capital of Gandhar .Taxila was the famous site of
Gandhar art.Gandharart. Also known as Greco-Roman –Buddhist
art.Chanakyawas a teacher at Taxila University.
28 B DadabhaiNaoroji, known as the Grand Old Man of India, was a
moderate leader of INC. He was a Liberal Party member of
parliament (MP) in the United Kingdom House of
Commons between 1892 and 1895, and the first Asian to be a British
MP. His book POVERTY AND UN BRTISH RULE IN
INDIA brought attention to the draining of India's wealth into
Britain.
29 D This is known as the Northeast Monsoon or Retreating Monsoon
. While travelling towards the Indian Ocean, the dry cold wind picks
up some moisture from the Bay of Bengal and pours it over
peninsular India and parts of Sri Lanka.
30 A Atacama is a desert of Chile which is driest desert on the earth
31 D The Answer is right for the year 1998 however the answer now 2016
shall be 5 States.
32 A Uri Hydroelectric project is located in Baramula district of the
Jammu and Kashmir on the river of Jhelum.
33 C Abdusalam Abul Akbar became thePresdient of Nigeria the death of
GenSan Abacha in 1998
34 A Indonesia (Rupiah) in 1998
35 A Operation "Jayasikkurui" was a Sri Lankan military action launched
on 13 May,1997. Primary objective of this operation was to clear a
land route to the government-held Jaffna Peninsula through territory
held by the LITTE.
36 D Rajan Pillaiwas an Indian businessman, popularly known as the
'Biscuit Baron'. He died in custody four days after his arrest in 1995,
The Justice Leila Seth Commission under Leila Seth was constituted
to enquire into the conspiracy angle of his custodial death.
37 B Dominique Lapierre
38 B Richard Attenborough was an English actor, film director and film

42
producer. Attenborough won two Academy award for Gandhi film
in 1983
39 A Harper Collinsis the publisher of The God of Small Things. This
famous novel is written by Arundhati Roy
40 A English Patient
41 C Claude Cohen-Tannoudji is a French physicist and Nobel Laureate.
He shared the 1997 Nobel Prize in Physics with Steven Chu and
William Daniel Phillips for research in methods of laser cooling and
trapping atoms.
42 C Dileep Kumar has been awarded the Pakistan's highest civilian
medal, Nishan-i-Imtiaz in 1997
43 C Ismail Merchant was an Indian-born Hollywood film producer and
director.
44 B The Ramon Magsaysay award is an annual award established by
Philippine President Ramon Magsaysay in 1957.Vinoba bhave was
the first Indian who got Ramon Magsaysay Prize.

45 D Green house gas is a gas in an atmosphere that absorbs and emits


radiation within the thermal infrared range. This process is the
fundamental cause of the greenhouse effect. water vapour (H2O),
carbon dioxide gas is main responsible for green house effect
46 D Haemophiliaa isdisorder in which blood doesn't clot normally. Dr. J
Pool discovered hemophilia in 1960. Hemophilia is more common in
Male because Males have only one X
chromosome. HEMOPHILIA is usually not seen in females because
females have two X chromosomes and one X chromosome can mask
the problem of the other. This disorder is hereditary
47 A worms have no bones
48 B Jonas Edward Salk was an American medical researcher and
virologist. He discovered and developed the first successful polio
vaccine
49 B Tariff barriers to be brought down by the year 2003
50 A Foreign Investment Promotion Board is a national agency of Govt of
India, with the remit to consider and recommend FDI
51 C Dauor Suker (Croatia)
52 B The word 'Foot' comes from football and 'ix' from Asterix, the noted
French cartoon
53 B Viswanathan "Vishy" Anand is an Indian chess Grandmaster and a
former World Chess Champion. Viswanathan Anand became India's
first grandmaster in 1988
54 A& Grand Slam or Grand slam may refer to Autoracing, chess, bridge
B (cards),golf ,tennis ,baseball, judo

55 A According to Article 76, Attorney General of Indiais a legal advisor


of Govt of India.
56 A The South Asian Association for Regional Cooperation (SAARC) is

43
an economic and geopolitical organisation of eight countries that are
primarily located in South Asia or Indian subcontinent. The SAARC
Secretariat is based in Kathmandu, It is established in 1985.
57 D Shivkumar Sharma is the master instrumentalist of the Santoor. The
Santoor is a folk instrument of J & K.
58 C A self-fuelling reusable multi-role mini space plane capable of
putting Satellites in orbit which is being designed by Indian scientists
59 C Dame Rosalyn C. Higgins, Baroness Higgins, DBE, QC is the
former President of the International Court of Justice. She was the
first female judge elected to the ICJ, and was elected President in
2006. Her term of office expired on 6 February 2009.
60 D AT Present speaker of lok sabha is SumitraMahajan

1999
Question Answer Explanation
number
31 A The secretary general of Lok Sabha who is the chief of its secretariat, is
appointed by the speaker of the lok sabha
32 C According to article 156 The Governor shall hold office during the
pleasure of the President
33 A The constitution of India borrowed the concept of the directive
principles of state policy from the constitution of Ireland. The Directive
Principles of State Policy contained in Part IV, Articles 36-51 of the
Indian constitution
34 A The Indian National Union was formed in 1854 by A.O. Hume&in1885
Indian National union converted into INC
35 A Mundaka Upanishad. The Upanishad are a collection of texts that
contain some of the central philosophical concepts of Hinduism. There
are 108 upanishdas.
36 D Equal pay for equal work (article 39) Protection of the health of the
workers (article 42 ) and Prevention of concentration of wealth and the
means of production are socialist principle. The Directive Principles of
State Policy contained in Part IV, Articles 36-51 of the Indian
constitution .The Directive Principles may be classified into 3 broad
categories—
1. Socialistic
2. Gandhian and
3. Liberal
37 C The five types of writs issued are (a) Habeas Corpus (meaning - you
may have the body) (b)Mandamus, meaning we command (c)Certiorari,
meaning to be certified (d)Prohibition and (e) Quo warranto.
38 C Hawkins was an adventurous English Seaman and he arrived in India

44
in 1608 with a letter from James I to the Emperor of India (Jahangir)
39 B The Kailash nath temple is one of the largest rock cut ancient Hindu
temples located in Ellora, Maharashtra, constructed by
Rashtrakuta king Krishna I ( 8th century )
40 B Han dynasty. This wall built 220–206 bce by Quin Shi huang the first
Emperor of China.
41 C He was called so by Vincent Smith because of his great military
conquests known from the 'Prayag Prashati' written by his courtier and
poet Harisena, who also describes him as the hero of a hundred battles.
42 D Shivaji Bhonsle, also known as Chhatrapati Shivaji Maharaj, was an
Indian warrior king and a member of the Bhonsle Maratha dynasty.
43 B The Godavari River is the second longest river in India after the river
Ganges having its source at Tryambakeshwar, Maharashtra
44 A Equinox is where the day and night is equal.
45 D The period October to December is referred to as north east monsoon
season over peninsular region.Earliar this period was also known as post
monsoon or retreating southwest monsoon

46 B Karnataka
47 C Jog Falls, Gerosoppa Falls is a highest waterfall (255m) in India located
in Karnataka on the river of Sharawati.
48 B Shabana Azmi is an Indian actress of film, television and theatre. She is
the wife of poet and screenwriterJaved Akhtar and daughter of Kaifi
Azmi
49 C Bhupen Hazarika was an Indian lyricist, musician, singer, poet and film-
maker from Assam
50 A The Redcliff line became the international border between India and
Pakistan (which also included what is now Bangladesh) during the
partition of India. The line divided Bengal into Indian held West Bengal
and East Bengal which became East Pakistan in 1956.
51 C Walter Max Ulyate Sisulu was a South African anti-apartheid activist
and member of the African National Congress, serving at times as
Secretary-General and Deputy President of the organisation
52 D Methanol, also known as methyl alcohol, wood alcohol, wood naphtha,
methyl hydrate, or wood spirits, is a chemical with the formula CH₃OH
53 A Acetylene is the chemical compound with the formula C₂H₂. It is a
hydrocarbon and the simplest alkyne. This colorless gas is widely used
as a fuel and a chemical building block
54 C Dr. V. Mohini Giri is an Indian social worker and activist, who has been
Chairperson of the Guild of Service, a New Delhi-based social service
organization
55 D Indica
56 A Chamoli district is the second largest district of Uttarakhand state of
India
57 B Encourage the use of solar energy
58 C Swami Vivekananda Bengali, Shāmi Bibekānondo; 12 January 1863 – 4

45
July 1902, born Narendranath Datta, was an Indian Hindu monk, a chief
disciple of the 19th-century Indian mystic Ramakrishna. he introduced
Hinduism at the Parliament of the World's Religions in Chicago in
1893.the Ramkrishna mission was established by swami ji in1896 in
bellur
59 B 'Blue dwarfs' and 'Red giants' refer respectively to Young stars and old
stars
60 D he was a South African anti-apartheid revolutionary, politician,
and philanthropist, who served as President of South Africa from 1994
to 1999
61 A She was a middle distance runner and won the 800 metres at the 1995
Asian Athletics Championships. She went on to win the bronze medals
in 800 m and 1500 m events in the in 1998Asian Athletics
Championships Championships and gold medals in both the events in
theAsian Games in Bangkok in 1998.
62 A Linor Abargil is an Israeli lawyer, actress, model and beauty queen who
won the Miss World beauty pageant in 1998
63 B Indian Express Limited is an Indian news media publishing company. It
publishes several widely circulated dailies, including The Indian
Express and The Financial Express in English, the Loksatta in Marathi
and the Jansatta in Hindi
64 * The 2014 Commonwealth Games (officially the 20th Commonwealth
Games) were held in Glasgow, Scotland, from 23 July to 3 August
2014.
65 D He was awarded the Nobel Memorial Prize in Economic Sciences in
1998 and Bharat Ratna in 1999 for his work in welfare economics.
66 A Sir Ahmad Salman Rushdie, FRSL is a British Indian novelist and
essayist. His second novel, Midnight's Children, won the Booker Prize
in 1981
67 D Shakespeare in Love
68 C The shaheen, is a land based supersonic and short -to-intermediate
range surface to sueface guided ballistic missile developed by the
Pakistan

69 B Harinder Singh
70 C Serbia is a country on southeast Europe's Balkan peninsula .
71 C Silicon chips can contain computer processors, memory and special
devices
72 B A barometer is a scientific instrument used in meteorology to
measure atmospheric pressure,discovered by Evangelista Torricelli
73 B The recharging is done by passing a salt (brine) solution through the
resin. The sodium replaces the calcium and magnesium which are
discharged in the waste water. Haed water treated with an ion
exchange water softener has sodium added
74 C The answer is correct for the year however the answer now 2016 is 67.
75 B Ornithology

46
76 C The FEMUR is the only bone of the human thigh. It is both the longest
and the strongest bone in the human body,
77 A .Portuguese explorer Vasco de gama discovered a sea route to India in
1497-1498..
78 D Ports in India.
Indian coastline is about 7516.6 kilometers and it is one of the biggest
peninsulas in the world. 12 major ports service it, 200 notified minor
and intermediate ports. Maharashtra (48) has the maximum number of
non-major ports followed by Gujarat (42) and Andaman & Nicobar
Islands (23).
79 A Bank of Tokyo-Mistubishi Bank is the world's biggest bank
80 A The International Monetary Fund is an international organization
headquartered in Washington, D.C., of "189 countries working to foster
global monetary cooperation,at present IMF CEO IS Christine
Lagarde.

2000
Question Answer Explanation
number
52 A Prohibition of discrimination on the basis of religion, race, caste, sex or
place of birth(art-15),Equal opportunity(art16), Right to freedom(art-
19), Cultural and Educational Rights.(art-29,30) are enjoyed by citizens
only
53 A The G 7 is a group consisting of Canada, France, Germany, Italy, Japan,
the United Kingdom, and the United States. It was founded in 1975.
The 42nd G7 summit was held in Japan on May 2016
54 B It (26 Nov, 1949) is also called as Law day,constitution day
55 B Article 16 of the constitution lays down that the State cannot
discriminate against anyone in the matters of employment but he State
may also reserve posts for members of backward classes, scheduled
castes or scheduled tribes which are not adequately represented in the
services under the State to bring up the weaker sections of the society.
56 C A form of government which is headed by an elected representative and
not a hereditary monarch is known as Ripublican. The sources of
REPUBLICAN in Indian Constitution are French constitution.

57 A Sri Nagendra Singh was an Indian lawyer and administrator who served
as President of the International Court of Justice from 1985 to 1988
58 D Accoding to article 124 Judges of the Supreme Court of India serve till
they attain the age of 65.
59 C India has a single integrated judiciary unlike USA. The
Indian judiciary consists of a Supreme Court for the entire nation, High
Courts in the states, District Courts and the courts at local level. India
has an integrated judiciary. It means the Supreme Court controls
the judicial administration in the country.

47
60 D Perjury means the offence of wilfully telling an untruth or making a
misrepresentation under oath.
61 A Diplomatic immunity means the privilege of exemption from certain
laws and taxes granted to diplomats by the state in which they are
working.
62 D National commission to review the working of the constitution also
known as Justice Venkatachaliah commission was set by NDA Govt
led by Atal bihari Vajpayee on 22 February 2000 for suggesting
possible amendments to the Constitution of India

63 D The Liberhan Commission was a long-running inquiry commissioned


by the Government of India to investigate the destruction of the disputed
structure Babri Masjid in Ayodhya in 1992.
64 B The International Court of Justice is is the principal judicial organ of the
United Nations (UN). It was established in June 1945 by the Charter of
the United Nations and began work in April 1946. The seat of
the court is in The Hague (Netherlands).
65 C Inflation is a sustained increase in the general price level of goods and
services in an economy over a period of time
66 C In the case of indirect tax, the burden of tax can be shifted by the
taxpayer to someone else. Indirect tax has the effect to raising the price
of the products on which they are imposed. Customs duty, central
excise, service tax, GST and value added tax are examples of indirect
tax.
67 C The scheme of operation blackboard was launched in 1987 to provide
minimum essential facilities to all primary school in country

68 C
69 C Sex ratio is used to describe the number of females per 1000 of males.
In the Population Census of 2011 it was revealed that the population
ratio in India 2011 is 943 females per 1000 of males.
70 C The Khilafat movement (1919–22) was a pan-Islamic, political protest
campaign launched By Muslims in British India to influence the British
government. The movement collapsed by late 1922 when Turkey gained
a more favourable diplomatic position and moved toward secularism.
By 1924 Turkey simply abolished the roles of Sultan and Caliph.
71 A Blood may be classified as one of these four types:
 Type A

 Type B
 Type AB
 Type O
type O negative blood was considered the universal blood donor type
and type AB blood can receive blood from any group (with AB being
preferable), but cannot donate blood to any group other than AB. They

48
are known as universal recipients.
72 C Mosquitoes have become DDT resistant
73 B Mr. Subramaniam Swamy
74 A disallowed reservation in super speciality medical and engineering
courses
75 B The Madras high court is one of the three High Courts in India
established at the Presidency Towns byLetters patent granted by Her
Majesty Queen Victoria bearing date 26 June 1862
76 C stayed by the Supreme Court
77 B European Monetary Union is an group of policies aimed at converging
the economies of member states of the European Union at three stages.
The policies cover the 19 eurozone states, as well as non-euro European
Union states.
78 B Rajeev Gandhi the ex-Prime Minister of India, occurred as a result of
a suicide bombing in Sriperumbudr, nearChennai, in Tamil
Nadu, India on 21 May 1991
79 B is the scientific analysis and study of interactions among organisms and
their environment. It is an interdisciplinary field that
includes biology, geography, and Earth science
80 C special provisions for the state of Jammu and Kashmir
81 C In 1975 which sikkim merged with the Union of India by 36th
amendment
82 A HGB is an international scientific research project with the goal of
determining the sequence of chemical base pairs which make up
human DNA, and of identifying and mapping all of the genes of
the human genome from both a physical and a functional standpoint
83 B Permanent Settlement was introduced by Lord Cornwallis in 1790-93 in
Bengal, bihar, Orissa , Varanasi and northern karnataka
84 B Chauri-Chaura incident occurred at Chauri Chaura in the Gorakhpur
district of the United Province, British India on 4 February 1922, when
a large group of protesters participating in the Non-cooperation
movement turned violent, leading to police opening fire. In retaliation
the demonstrators attacked and set fire to a police station, killing all of
its occupants. The incident led to the deaths of three civilians and 22 or
23 policemen. The Indian National Congress halted the Non-
cooperation Movement on the national level as a direct result of this
incident
85 A The BCG is a vaccine primarily used against tuberculosis. The BCG
vaccine was first used medically in 1921
86 C The International Court of Justice is is the principal judicial organ of the
United Nations (UN). It was established in June 1945 by the Charter of
the United Nations and began work in April 1946. The seat of
the court is in The Hague (Netherlands).
87 B The North-South Dialogue refers to the process through which the
developing and newly independent nations of the “third world,”
predominantly in Asia, Africa, and Latin America, engaged the

49
industrialized countries of North America and Western Europe in
negotiations over changes to the international economic system during
the 1970s.
88 D NTPC (previously known as The National Thermal Power Corporation)
is an Indian PSU Public Sector Undertaking, engaged in the business
of generation of electricity and allied activities. It is a company
incorporated under the Companies Act 1956 and a Government
Company within the meaning of the act. The headquarters of the
company is situated at New Delhi.
89 C decreased from 21 to 18 by 61st amendment in 1989
90 C RSS is a National Volunteer Organisation,National Patriotic
Organisation, is a right-wing Hindu nationalist paramilitary volunteer
organisation in India that is widely regarded as the parent organisation
of the ruling party of India, the Bharatiya Janata Party, Founded on 27
September 1925.
91 A Sumit Sarkar & K.N. Pannickar
92 D rejected the petition
93 A
94 A Indian Airlines Flight 814 commonly known as IC 814 was an India
Airlines Airbus A300 en route from Tribhuvan International
Airport in Kathmandu, Nepal to Indira Gandhi International
Airport in Delhi, India on Friday, 24 December 1999, when it
was hijacked. Harkat-ul-Mujahideen, a Pakistan-based Islamic
extremist group, was accused of the hijacking. After touching down
in Amritsar, Lahore and Dubai, the hijackers finally forced the aircraft
to land in Kandahar, Afghanistan, which at the time was controlled by
the Taliban
95 A Aparna Popat is a former Indian badminton player. She was India's
national champion for a record equalling nine times when she won all
the senior national championships between 1997 and 2006
96 D Coastal India spans from the south west Indian coastline along
the Arabian sea from the coastline of the Gulf of Kutch in its western
most corner and stretches across the Gulf of Khambhat, and through
the Salsette Island of mumbai along the Konkan and southwards across
the Raigad region and through Kanara and further down through
Mangalapuram or Mangalore and along theMalabar through Cape
Comorin in the southernmost region of South India with coastline along
the Indian Ocean and through theCoromandal
Coast or Cholamandalam Coastline on the South Eastern Coastline of
the Indian Subcontinent along the Bay of Bengal through the
Utkala Kalinga region until the easternmost Corner of shoreline near
the Sunderbans in Coastal East India.
97 B Damodar valley lies in chotanagpur plateau of the state of Jharkhand
and west Bengal.Many people refer to the damodar valley as the Rhur of
India because of its similarities with the Rhur mining industrial area o
Germany

50
98 B The march on foot undertaken by Gandhi and seventy-eight Congress
volunteers was the most significant event in the history of the breach of
salt law in our country. march began on 12 March 1930 and As Gandhi
and the others continued on what would become a 24-day march
to Dandi to produce salt without paying the tax, growing numbers of
Indians joined them along the way. Gandhi broke the salt laws at 6:30
am on 6 April 1930 at Dandi
99 A According to Budhha THE SOUL IS A MYTH
100 D The three Round table conferences of 1930–32 were a series of
conferences organized by the British Government to discuss
constitutional reforms in India. They were conducted as per the
recommendation by the report submitted by the Simon Commission in
May 1930. Demands for swaraj, or self-rule, in India had been growing
increasingly strong.The first Round Table Conference was opened
officially by Lord Irwin on November 12, 1930 at London and chaired
by the British Prime Minister, Ramjay Macdonald.
The three British political parties were represented by sixteen delegates.
There were fifty-seven political leaders from British India and sixteen
delegates from the princely states. In total 89 delegates from India
attended the Conference. However, the INC, along with Indian business
leaders, kept away from the conference. Many of them were in jail for
their participation in CIVIL DISOBEDIENCE MOVEMENT
101 C The Bhoodan movement was a voluntary land reform movement in
India, started by Acharya Vinoba Bhave in 1951 at Pochampally village
in Telangana.
102 D
103 C Current may pass to the earth in case of short circuit without doing any
harm to
104 C Dialysis is a treatment that does some of the things done by healthy
kidneys. It is needed when your own kidneys can no longer take care of
your body's needs
105 B Jaundice is a yellowish pigmentation of the skin,
the conjunctival membranes over the sclerae (whites of the eyes), and
other mucous membranes caused by high blood bilirubin levels.
Jaundice is often seen in liver disease such as hepatitis.
106 D
107 A Graphite, archaically referred to as Plumbgo, is a crystalline form
of carbon, a semimetal, a native element mineral, and one of
the allotropes of carbon
108 A Silicon
109 C Azim Hashim Premji is an Indian business
tycoon, investor and philanthropist, who is the chairman of Wipro
Limited, informally known as the Czar of the Indian IT Industry.
110 C Ramanujan (22 December 1887 – 26 April 1920) was an Indian
mathematician and autodidact.
111 D at present the chairman of NHRC is HL Dattu

51
112 A was an Australian Christian missionary who, along with his two sons
Philip and Timothy , was burnt to death by a gang while sleeping in his
station wagon at Manoharpur village in Keonjhar district
in Odisha, India on 23 January 1999. In 2003, a Bajrang
Dal activist, Dara Singh, was convicted of leading the gang that
murdered Graham Staines and his sons, and was sentenced to life in
prison
113 C The minimum number of judges for setting up a constitution bench are 5
114 C The answer is correct for the year however the answer is incorrect now
as right to education is also a fundamental right now under article 21-A
(Added by 86th Amendment).
115 C

2001
Question Answer Explanation
number
33 B Chokila Iyer is India's first female Foreign secretary. She took charge on
14 March 2001 replacing Mr. Lalit Mansingh. She is a 1964 Batch
Officer
34 A The Servent of India Society was formed in Pune, Maharashtra, on June
12, 1905 by Gopal Krishna Gokhale.
35 B Viswanathan "Vishy" Anand is an Indian chess Grandmaster and a
former World Chess Champion. Viswanathan Anand became India's
first grandmaster in 1988
36 C Karnam Malleswari is a retired Indian weightlifter. She won the world
title in the 54 kg division in 1994 and 1995 and placed third in 1993 and
1996.
37 A The Census of India has been conducted 15 times, As of 2011. It has
been conducted every 10 years, beginning in 1871(Lord Meyo). All the
census since 1951 are conducted under 1948 Census of India Act.

38 D Our Research team finds the question to be factually incorrect. (Hence


answer shall be None of the above). The Bharat Ratna is the highest
civilian award of the Republic of India. Instituted in 1954
39 B Joseph Estrada is a Filipino politician who served as the 13th President
of the Philippines from 1998 to 2001 and as the 9th Vice President of
the Philippines from 1992 to 1998. Slobodan Milošević was a Serbian
and Yugoslav politician who was the President of Serbia from 1989 to
1997 and President of the Federal Republic of Yugoslavia from 1997 to
2000
40 B The National Association of Software and Services Companies is a
trade association of Indian Information Technology and Business
Process Outsourcing industry. Established in 1988, NASSCOM is a
non-profit organization.At present NASSCOM chairman is CP Gurnani.
Founders member of NASSCOM is Nandan Nilekani & Dewang Mehta

52
41 B But According to census 2011 the most populous states in the country
are UP,Maharashtra,Bihar& West Bengal
42 A Pullela Gopichand is a former Indian badminton player. Presently, he is
the Chief National Coach for the Indian Badminton team.He won the
All England Open Badminton Championships in 2001
43 C Montesquieu was a French lawyer, man of letters, and political
philosopher of France ,his famous book is The spirit of law.
44 A In 1946, the Labour Government dispatched an all-party parliamentary
delegation to India to meet Indian leaders and convince them of the
British desire for an early settlement of the Indian constitutional issue.
However, it was only in March 1946 that with the arrival of three
Cabinet Ministers, Lord Pethick-Lawrence, Sir Stafford Cripps and Mr.
A. V. Alexander, a crucial stage was reached in the negotiations
between the British Government and the Indian political parties.
45 B Bal Gangadhar Tilak, born as Keshav Gangadhar Tilak, was an Indian
nationalist, teacher, social reformer, lawyer and an independence
activist Considered as Father of Indian National Movement; Founded
“Deccan Education Society”(1867) ; formed Home Rule League
movement in 1916 in Belgaum to attain the goal of Swaraj.his famous
book is The Arctic Homes of the Arayans.MARATHA(English) and
KESHARI(Marathi) was his famous magzine
46 B In 1327, Tughluq passed an order to shift the capital from Delhi
to Daulatabad ( Maharashtra) in Deccan region of south India. Tughluq
said that it would help him to establish control over the fertile land of
the Deccan plateau. He also felt that it would make him safe from
the Mongol invasions which were mainly done on Delhi and regions
around in north India. in 1335, he decided to shift the capital back to
Delhi.
47 B During Camparan Satyagriha (1917), Gandhi was addressed by the
people as Bapu (Father) and Mahatma by Ravindra Nath Tagore.
48 A Anand Math is a Bengali novel, written by Bankim Chandra
Chattopadhyay and published in 1882. Set in the background of
the Sannyasi Rebellion in the late 18th century. The national song of
India, Vande Mataram, was first published in this novel.first sung at the
1896 session(CULCUTTA) of the Indian National Congress
49 B He was also called last vieroy of British India
50 D Dayanand Saraswati was a Hindu religious leader who founded the
Arya Samaj in 1875 in Bombay, a Hindu reform movement of the Vedic
tradition. His famous book is Satyarth Prakash
51 A According to article 76 Attorney General of India is appointed by the
president.
52 A According to article 75 Prime Minister is appointed by the President ,he
is chief of the Govt,leader of the majority party,head of council of
minister & chief advisor of the president.
53 B In case of conflict between central law and state law on a subject listed
in the Concurrent List, the central law prevails over the state. But it also

53
says executive power rests with states unless specified
54 D No such limit
55 A Accoding to article 60 the oath of the President is taken in the presence
of the Chief Justice of India, and in his absence, by the most senior
judge of the Supreme Court.
56 C According to article 108 Speaker of the Lok Sabha presides over the
joint sittings of both Houses of Parliament but summoned by the
President.There are three joint sitting held in Indian parliamentary
history-1960,1977,2000
57 C The Indian Union is Indestructible union of destructible
states.According to article 1 India that is Bharat shall be union of states.
58 C According to article 56 the resignation of the President should be
addressed to the Vice-President.
59 A The states concerned with the Sardar Sarovar Project are Gujarat,
Maharashtra, Madhya Pradesh and Rajasthan.It is a combined project of
Gujrat and Rajasthan on the river of Narmada.This project was opposed
by Medha Patekar due to environmental reason and restablshment of the
people.
60 A Isopeth is a line on a map connecting points having equal incidence of a
specified meteorological feature
61 A IST is the time observed throughout India , with a time
offset of UTC+05:30, Indian Standard Time is calculated on the basis of
82.58° E longitude, in , Mirzapur
62 C Paradip, Vizag, Madras, Tuticorin
63 B Gujarat forms the longest coastline of all Indian states. Coastle length
of Indian states in decreasing order are Gujrat (1600 km),Tamilnadu
(1076km),Andhara (974),Maharashtra (720 km),kerala (580 km),Odisha
(480 km).Karnataka (322 km).WB(201),goa (103)
64 A Haryana
65 A Gross domestic product at factor prices refers to Gross value of goods
and services produced in an year
66 C According to Law of, Demand is Price and quantity demanded are
inversely related
67 D The Rio Declaration on Environment and Development was a short
document produced at the 1992 United Nations , UNCED, informally
known as the Earth Summit, and signed by over 170 countries
68 A
69 B Some Direct taxes are-Income tax,banking cash transaction,corporate
tax,capital gain tax,fringe benife tax,securities tax and Tax incentive and
indirect taxes are-anti dumping duties,custom duty,,excise duty,sales
tax,service tax and VAT,GST
70 D Association of southeast association Nation,ASIAN is a political and
economic organisation of ten Southeast Asian countries. It was formed
on August 8, 1967 by Indonesia, Malaysia, the Philippines,Singapore,
and Thailand. Since then, membership has expanded to
include Brunei, Cambodia, Laos, Myanmar (Burma), and Vietnam

54
71 D Structural unemployment is is a form of unemployment caused by a
mismatch between the skills that workers in the economy can offer, and
the skills demanded of workers by employers.
72 C Oxysen is an important part of the atmosphere and diatomic oxygen gas
constitutes 20.8% of the Earth's atmosphere. Additionally, as oxides the
element also makes up almost half of the Earth's crust
73 B both the intensity and frequency of sound increases
74 D
75 B Seismology is the scientific study of earthquakes
76 B Always increase due to gravitational force
77 D Hypochondriasis usually develops during adulthood.
Symptoms include a long-term and intense fear of having a serious
condition and worry that minor symptoms indicate something serious

78 B The date was chosen to honour the United Nations General Assembly’s
adoption and proclamation, on 10 December 1948
79 B U.K., U.S.A., Russia, France and China
80 B The World Bank is an international financial institution that provides
loans to developing countries for capital programs,it’s heaquartes is in
Washingoton DC.FOUNDED IN 27 December 1945.
81 B On Indian Government's Service
82 D The term was introduced by the sattanathan commission in 1971and
Ramnandan committee(1993), which directed that the creamy layer
should be excluded from the reservations (quotas) of civil posts and
services granted to the OBCs. The creamy layer criteria was defined as
earning more than 100,000 rupees per annum in 1993, and revised to Rs
2.5 lakh (2004), then Rs 4.5 lakh (2008), and Rs 6 lakh (2013)
83 D Recall is basically a process whereby the electorate has the power to
remove the elected officials before the expiry of their usual term.
84 A The Kyoto Protocol; which was agreed upon on December 11, 1997, at
a meeting of the UNFCCC in kyoto, Japan; was created as an effort to
force action on the international community. Under the Kyoto Protocol,
industrialized nations agreed to cut their greenhouse gas emissions to a
certain percentage below 1990 levels
85 C TANSI means Tamilnadu Small Industries Corporation.Jaya
Publications and Shashi Enterprises in which Jayalalithaa and
Shashikalaa were partners. Purchase Tansi foundry land and Foundry
and Tansi Enamel wire unit and land in 1992.Charge sheet filed on
November 15, 1996 on the ground of loss to state exchequer of Rs 2.28
crores and 58.88 lakhs, Charges were framed against six accused,
including Jayalalithaa, Sasikala on June 10,1999.

86 B Tehelka is an Indian news magazine known for its investigative


journalism and sting operations. It was founded by Tarun
Tejpal and Aniruddha Bahal in 2000 as a website Tehelka's first sting

55
operation was on a cricket match fixing scandal in 2000 and the second,
which was its most widely known, was Operation West End for
corruption in various defence deals in 2001
87 A George Walker Bush is an American politician who served as the 43rd
President of the United States from 2001 to 2009 and 46th Governor of
Texas from 1995 to 2000
88 B The Lok pal bill is an anti-corruption bill drafted and drawn up by civil
society activists in India seeking the appointment of a Jan Lokpal, an
independent body to investigate corruption cases.

2002
Question Answer Explanation
number
41 D increase stability
42 A Smog is a kind of air pollution, originally named for the mixture of
smoke and fog in the air. Classic Smog results from large amounts
of coal burning in an area and is caused by a mixture of smoke and
sulfur dioxide. In the 1950s a new type of Smog, known as
Photochemical smog, was first described
43 C Sir Edward Richard Henry, a British official stationed in India,
began to develop a system of fingerprint identification for Indian
criminals.
44 D A mixture of two or more straight fertiliser material is referred to as
mixed fertiliser . Mixed fertilisers are physical mixtures of fertiliser
materials containing two or three major plant nutrients. Mixed
fertilisers are made by thoroughly mixing the ingredients either
mechanically or manually. If a proper mixture is used to suit a
particular soil type and crop, the used of a fertiliser mixture leads to
balanced manuring. This gives higher yield and more profit to the
cultivators.

45 A Chimical Reaction is a process that involves rearrangement of the


molecular or ionic structure of a substance, as distinct from a
change in physical form or a nuclear reaction
46 B Mixed economy is an economic system combining private and state
enterprise.
47 B First five year Plan was launched in 1951 to 1956 which mainly
focused in development of the primary sector. The First Five-Year
Plan was based on the Harrod–Domar model The target growth rate
of First plan was 2.1% and the achieved growth rate was 3.6%
48 D The economic policies of the British resulted in impoverishing
society.expect a handful owing the colonial policies of economic
exploitation,industry,trade-commerce and agriculture,India became
de-industrialized
49 A The Green revolution refers to a series of research, and

56
development, and technology transfer initiatives, occurring between
the 1940s and the late 1960s, that increased agricultural production
worldwide. The main development was higher-yielding varieties
of wheat, which were developed by many scientists, including
American agronomist Dr. Norman Borlaug, Indian geneticist M. S.
Swaminathan, and others
50 C The textile industry continues to be the second largest employment
generating sector in India. India is the second largest producer of
fibre in the world and the major fibre produced is cotton. Other
fibres produced in India include silk, jute, wool, and man-made
fibers. 60% of the Indian textile Industry is cotton based.
51 D The champaran movement was launched in 1917,it’s objective was
to create awakening against the European planters . after this
movement Mahatma title was given by Ravindra Nath Tagore to
Gandhi ji.
52 D Motilal Nehru and C R Das was the founder member of Swaraj
party (congress khilafat swaraj party in 1992),it is an integral part
of the congress.it decided to take part in election of the council.
53 A The Jallianwala Bagh massacre took place on 13 April 1919 when
a crowd of nonviolent protesters, along with Baishakhi pilgrims,
were fired upon by troops of theBritish Indian Army under the
command of Colonel Reginald Dyer
54 B Dayanand Saraswati was a famous Hindu religious leader who
founded the Arya Samaj in 1875 in Bombay, a Hindu reform
movement of the Vedic tradition. His famous book is Satyarth
Prakash
55 D The cabinet mission plan (1946) aimed to discuss and plan for the
transfer of power from the British government to Indian leadership
to provide India with independence. Formulated at the initiative
of Clement Attlee, the Prime Minister of the United Kingdom, the
mission had Lord Pethick-Lawrence, the Secretary of State for
India,Sir Stafford Cripps, President of the Board of Trade, and A.
V. Alexander, the First Lord of the Admiralty
56 A a political and economic theory of social organization which
advocates that the means of production, distribution, and exchange
should be owned or regulated by the community as a whole.
57 C The Security Council held its first session on 17 January 1946.The
Security Council consists of fifteen members- Russia, the United
Kingdom, France, China and the United States—serve as the body's
five permanent members. These permanent members can veto any
substantive Security Council resolution, including those on the
admission of new member states or candidates for Secretary-
General. The Security Council also has 10 non-permanent
members, elected on a regional basis to serve two-year terms
58 the question is incorrect
The Maximum strength of Lok sabha can be 552 (530 from states

57
20 from UT and 2 Anglo Indian). But at Present total seat of Lok
sabha are 545.
59 C The Ancient Greek philosopher Aristotle is known as the father of
political science. His famous book was The Politics and
Nicomachean Ethics .
60 A A federation is a political entity characterized by a union of
partially self-governing States or regions under a central
government.
61 A There are two type of expenditure- Plan and non plan.plan
expenditures are estimated after discussions between each of the
ministries concerned and the Planning Commission.
Non-plan revenue expenditure is accounted for by interest
payments, subsidies , wage and salary payments to government
employees, grants to States and Union Territories governments,
pensions, police, economic services in various sectors, other
general services such as tax collection, social services, and grants to
foreign governments.
Non-plan capital expenditure mainly includes defence, loans to
public enterprises, loans to States, Union Territories and foreign
governments.
62 A The area under irrigation has during the five year plans are
Increased
63 A In India the yield of grams is not improving even after green
revolution because It is difficult to produce high yield variety
64 A Biogas is gaseous fuel, especially methane, produced by the
fermentation of organic matter.
65 C Opium is the dried latex obtained from the opium poppy. Opium
latex contains approximately 12 percent of the
analgesic alkaloid morphine, which is processed chemically to
produce heroin and other synthetic opioids for medicinal use and
for the illegal drug trade
66 C The Equator is an imaginary line which dividing the Earth into
the Northern Hemisphere and Southern Hemisphere. The latitude of
the Earth's equator is by definition 0° of arc. In the cycle of
Earth's seasons, the plane of the equator passes through the Sun
twice per year at the March and September equinox. The Equator
traverses the land of 11 countries-
Gabon,congo,Uganda,Somalia,Kenya,Indonesia,Ecuador,Colombia
,brazil
67 A Ruhr Industrial Belt is situated in Germany.It is richest mineral
resources in Germany.The chotanagpur Plateau also known as The
Rhur of India
68 A The Bering Sea is a marginal sea of the Pacific Ocean. The Bering
Sea is separated from the Gulf of Alaska by the Alaska Peninsula.
The Bering Sea is named for Vitus Bering, a Danish navigator ,
who was the first European to explore it.

58
69 C The Rhine is a European river that begins in the Swiss canton of
Graubünden in the southeastern Swiss Alps.
70 B Tropical moist forest a convenient term that describes tropical rain
forest plus tropical seasonal forest
71 D Bandipur National Park is in Karnataka
72 C Psyllium seed husks, also known as isabgol, are portions of the
seeds of the plant Plantago ovate.
73 B The Narmada also called the Rewa, is a river in central India and
the fifth longest river in the Indian subcontinent.the origin of
Narmada is Amarkantaka hill(MP)
74 A The Uri Hydro Electric Project is located in Jammu & Kashmir.
75 A Kanyakumari is a coastal town in the state of Tamil Nadu on India's
southern tip , the town was known as Cape Comorin .
76 B A promise made without the intention of performing it amounts to
fraud
77 C
78 C
79 B
80 A
81 C K.C.Wheare describes Indian federalism as quasi federalism,its
means the system of governance which is federal in form but
unitary in spirit
82 A Fundamental right contained in the part three and article 12 to
35 Constitution of India.there are 6 fundamental right in Indian
constitution.
83 D The Constitution originally provided for the right to property under
Articles 19 and 31, The Forty-Fourth Amendment of 1978 deleted
the right to property,now right to property is a legal right
84 B The Indian Constitution is Lengthiest Constitution in the world . It
is the longest written constitution of any sovereign country in the
world. B. R. Ambedkar is regarded as the chief architect of the
Indian Constitution. The Constitution was adopted by
the Constituent Assembly on 26 November 1949, and came into
effect on 26 January 1950, At the time of commencement, the
constitution had 395 articles in 22 parts and 8 schedules. The
Constitution, in its current form , consists 25 parts containing
465 articles, 12 schedules, and 100 amendments.
85 B According to article 21 Right to life and Personal liberty is A
Fundamental Right.
86 A
87 A
88 A In the 1970s, an organized resistance to the destruction of forests
spread throughout India and came to be known as the Chipko
movement.Sundarlal Bahuguna was the leader of this movement
89 B According to Biomedical waste (Management and Handling)

59
Rules, 1998 of India Any waste which is generated during the
diagnosis, treatment or immunization of human beings or animals
or in research activities pertaining thereto or in the production or
testing of biologicals.
90 B According to prohibition of child marriage act the minimum age
for marriage is that the boy must be 21 years and the girl 18 years.
91 B According to article 76 The Attorney general of India is the leagal
advisor of Govt of India . He is appointed by the President , and
holds office during the pleasure of the President. The 14th and
current Attorney General is Mukul Rohatgi.
92 C On population basis the States have their representation in Rajaya
sabha . At present the total seats in rajya sabha are 245(233
members are elected by the MLAs & 12 are nominated by the
president)
93 A Accoding to Article 63 of Indian Constitution that there shall be a
Vice-President of India. The Vice President shall act as President in
the absence of the President due to death, resignation,
impeachment, or other situations. The Vice President of India is
also ex officio Chairperson of the Rajya Sabha(article 64).
94 D Habeas corpus is a writ issued by Supreme court (article32) &High
court (art.226) for requiring a person under arrest to be brought
before a judge or into court, especially to secure the person's release
unless lawful grounds are shown for their detention
95 C
96 B
97 C
98 C Mandamus a judicial writ issued by Supreme court and High court
as a command to an inferior court or ordering a person to perform a
public or statutory duty.
99 B The Indian Constitution is Lengthiest Constitution in the world . It
is the longest written constitution of any sovereign country in the
world. B. R. Ambedkar is regarded as the chief architect of the
Indian Constitution. The Constitution was adopted by
the Constituent Assembly on 26 November 1949, and came into
effect on 26 January 1950, At the time of commencement, the
constitution had 395 articles in 22 parts and 8 schedules. The
Constitution, in its current form , consists 25 parts containing
465 articles, 12 schedules, and 100 amendments
100 A

2003
Question Answer Explanation
number
01 C
02 B

60
03 D
04 A
05 A
06 A
07 C
08 D First Plan (1951-56)
Second Plan (1956-61)
Third Plan (1961-66)
Fourth Plan (1969-74)
Fifth Plan (1974-79)
Currently, In operation
Twelfth Plan (2012-17)
09 D
10 C
11 C
12 A
13 A
14 B
15 A
16 B
17 A
18 B The permanent members of the United Nations Security Council, also
known as the Permanent Five, Big Five, or P5, include the following
five governments: China, France, Russia, the United Kingdom, and the
United States. The members represent the five great powers considered
the victors of World War II.
19 B
20 C Question was Modified.
The PM is the De-Facto Chairman of the Planning Commission;
However the Planning commission has been reconstituted as NITI
Aayog (Jan 1, 2015).
21 A
22 C Our research team finds the answer to be factually incorrect; the correct
answer now shall be seventh largest in terms of size.
23 C
24 A
25 B
26 B
27 C
28 A
29 D
30 B
31 A
32 C
33 C The answer might be correct for that year; however the answer now

61
shall be Saturn Planet.
34 D
35 C Mars is the nearest planet to Earth.
36 C
37 B
38 B
39 C
40 B
41 B
42 A
43 D The answer might be correct for that year; however the answer now
shall be 31 Judges.
30 Supreme Court Judges &
01 Chief Justice of India.
44 C
45 C
46 D
47 B
48 B
49 C
50 B
51 D The answer might be correct for that year; however the answer now
shall be 192. Since 2015, the flags of the two non-member observer
states are raised alongside those of the 193 member states. There are
193 United Nations (UN) member states, and each of them is a member
of the United Nations General Assembly.
52 D
53 A
54 B
55 C
56 D
57 C
58 A
59 B
60 B
61 B
62 C
63 B
64 A
65 A
66 B
67 C Question was Modified.
68 A
69 D
70 D

62
71 A
72 B He was awarded Bharat Ratna in 1987.
73 A
74 C Question was Modified.
75 C

2004
Question Answer Explanation
number
42 B Saliva is a watery substance located in the mouths of humans and
animals, secreted by the salivary glands
43 D vitamin is one of a group of organic substances, contains carbon.
Vitamins A, D, E and K are fat-soluble. Vitamins C and all the B
vitamins are water-soluble.

44 C The Retina converts the light rays into impulses that travel through
the optic nerve to our brain, where they are interpreted as the images
we see
45 C In jaundice, the skin and whites of the eyes look yellow. Jaundice
occurs when there is too much bilirubin in the blood.
46 A Purva mimasa written by Jaimini is one of the most important ancient
Hindu philosophical texts. Six Phillosophies are-Samkhya by
Kapilmuni, Yoga by Patanjali, Nyaya by Gautam, Vaisheshika by
kanada,,Purva Mimamsa by Jaimini & Vaadarayana & chaarvaka by
chaarvaka.

47 B Samudragupta was the fourth ruler of the Gupta Empire and the son
of Chandragupta I.The source of Samudragupta's history is
the Allahabad Pillar( by HARISHEN) .
48 A Mitakshara was written by Vijnanesvara who was a scholar in
the Western Chalukya court.
49 A Pitt's India Act 1784 provided for the appointment of a Board of
Control.
50 A All India Home rule League was established by Balgangadhar tilak
in April 1916 and established by Annie Besant in September 1916,
which was a political organization which aimed at self-government,
termed as Home Rule
51 C An Emergency was decleared in 1962 (Indo-China war), 1971
(Indo-Pakistan war), and 1975 (on the basis of internal disterbence).
52 B PM presides over the meeting of Council of Ministers of India.
53 D According to article 75 The Council of Ministers is jointly
responsible to the Lok Sabha
54 C Right to Freedom (Article 19)
55 C The present strength of Rajya sabha is 245 members of whom 233
are representatives of the States and Union territories and 12 are

63
nominated by the President.[
56 C According to article 56 the Tenure of the President is 5 years.
57 A Panchayati raj was formalized in 1992 by the 73rd amendment to the
Indian Constitution. The system has three levels: gram panchayat
(village level), panchayat samiti (block level) and zila parishad
(district level). The Panchayat Raj system was first adopted by the
state of Rajsthan in Nagaur district on 2nd Oct 1959. The second state
was Andhra Pradesh in 11 Oct.1959.
58 A The Supreme Court of India came into being on 28 January 1950.
59 A The Consumer Protection Act came into effect from 1986 to protect
the interests of consumers in India.
60 C Dr. B.N. Rau,he was also constitutional advisor of constituent
assembly.
61 B NDP=GDP-Depreciation
62 A Deficit is the excess of total expenditure over revenue receipts
63 C Stock exchange
64 A Indirect tax are sales tax, per unit tax, value added tax (VAT),
or goods and services tax (GST) & excise tax etc.
65 D Mix economy is an economic system combining private and state
enterprise. A mixed economic system is an economic system that
features characteristics of both capitalism and socialism

66 C Tarapur Atomic Power Station is located in Tarapur, Maharashtra


67 B Konark Sun Temple(13th-century) is located in Odisha, the temple
was built by king Narasimhadeva I
68 C India's first ship-building yard was established in Visakhapatnam in
1941,also known as Hindustan shipyard ltd
69 B Kangchenjunga is the third highest mountain in the world, and lies
partly in Nepal and partly in Sikkim.
70 D Security Printing Press Is located in Nasik ,Maharashtra
71 B The ozone layer shield is a region of Earth's stratosphere that absorbs
most of the Sun's ultraviolet (UV) radiation
72 D
90 D IRDA an agency of Government of India for insurance sector
supervision and development. its headquarters
at Hyderabad, Telangana where it shifted from Delhi in 2001
91 A W.R. Grace and Company is an American chemical conglomerate
based in Columbia, Maryland.
92 C NJA is located in Bhopal(MP)
93 Vasundhara Raje of the Bharatiya Janata Party is the current Chief
Ministerof Rajasthan
94 C
95 A State Bank of India is an Indian multinational, public sector bank. It
is a government-owned corporation with its headquarters in Mumbai,
Maharashtra

64
96 C Nokia is a communications and information technology company,
founded in 1865. It’s headquarter is in Helsinki .
97 At present Dmitry Anatolyevich Medvedev is the Prime Minister of
Russia.
98 A
99 A Sachin tendulkar the holder of the record for the number of runs in
both ODI andTest cricket, and the only player to complete more than
30,000 runs in international cricket.
100 A Oscar award is an annual American awards ceremony. The awards
ceremony was first broadcast to radio in 1930 and televised in 1953
101 A Ernest Rutherford was a New Zealand physicist who came to be
known as the father of nuclear physics
102 C Adrenalin Hormone
103 C About 30 dB
104 D NH 15
105 A Compact Disc (CD) is a digital optical disc data storage format.
106 B Roger Federer is a Swiss professional tennis player who is currently
ranked world No. 3 by the A.T. P
107 D India
108 A World AIDS Day held for the first time in 1988.
109 D The nobel peace prize 2007was shared, in two equal parts, between
the Intergovernmental Panel on Climate Change (IPCC) and Al Gore
110 B The 2010 FIFA World Cup was the 19th FIFA World Cup, the world
championship for men's national association football teams. It took
place in South Africa from 11 June to 11 July 2010. The final was
held on 11 July 2010 at Johannesburg. Spain defeated
the Netherlands 1–0.

2005
Question Answer Explanation
number
26 A Amniocentesis is a method used for sex determination .
27 A
28 B Hookwarm is a parasitic nematode worm which inhabits the
intestines of humans and other animals
29 A The RH Factor is a type of protein on the surface of red blood cells.
Most people who have the RH Factor are RH-positive. Those who do
not have the RH Factor are RH-negative.
30 D Vijay Tendulkar was a Fmous writer, literary essayist, political
journalist, and social commentator primarily in Marāthi.
31 B Sikander Lodi(Nizam Khan) was the Sultan of Delhi between 1489 to
1517. He became the next ruler of the Lodi dynasty after the death of
his father Bahlul Lodi . Sikandar lodi built the City of Agra in 1504
32 B The Gita Govinda was written by Jayadev in the 12th-century.

65
33 c The Doctrine of Lapse was an annexation policy introduced by Lord
Dalhousie. Doctrine of Policy was a policy of the British East India
Company under which if the ruler of a princely state or territory under
the paramountcy of the Company died without a natural heir, the
state/territory would automatically be annexed to the British empire.
34 B Indian council act 1909 was also known as Morley Minto reform
35 C The Constitutional head of the Union is The President
36 D According to article 64 The Vice-President of India is the Ex-officio
Chairman of the Rajya.
37 A According to article 74 The tenure of the Council of Ministers is
during the pleasure of the president
38 A Parliament can amend Fundamental Rights (article 368)
39 C The Constitution originally provided for the right to property under
Articles 19 and 31. Article 19 guaranteed to all citizens the right to
acquire, hold and dispose of property. By The Forty-Fourth
Amendment of 1978 deleted the right to property from the list of
fundamental rights. A new provision, Article 300-A, was added to the
constitution.now it is a legal right.
40 A According to article 60 The Oath of Office to the President is
conducted by The Chief Justice of India
41 A Panchayati raj was formalized in 1992 by the 73rd amendment to the
Indian Constitution. The system has three levels: gram panchayat
(village level), panchayat samiti (block level) and zila parishad
(district level). The Panchayat Raj system was first adopted by the
state of Rajsthan in Nagaur district on 2nd Oct 1959
42 C According to article 124; the judge of the Supreme Court holds
office till he reaches the age of 65 years
43 B The present Income Tax Act was enacted during 1961
44 B Judicial Review, a feature which was borrowed from the Constitution
of U.S.A. in the Indian Constitution
45 A Gross domestic product (GDP) is the monetary value of all the
finished goods and services produced within a country's borders in a
specific time period.

46 A AD VALOREM is a TAX whose amount is based on the value of a


transaction or of property. It is typically imposed at the time of a
transaction, as in the case of a sales or value-added TAX (VAT).
47 C A bull market is a financial market of a group of securities in which
prices are rising or are expected to rise. The term "bull market" is
most often used to refer to the stock market. A market condition in
which the prices of securities are falling, and widespread pessimism
causes the negative sentiment to be self-sustaining are bear .

66
48 C The Foreign Exchange Management Act, 1999 is an Act of the
Parliament of India "to consolidate and amend the law relating to
foreign exchange
49 B A general increase in prices and fall in the purchasing value of money
are called Inflation
50 C The earth satellite station located in Trivandrum
51 B Narmada is one of the rivers in India that flows in a rift valley,
flowing west between the Satpura and Vindhya ranges. The other
rivers which flow through rift valley include Damodar River in Chota
Nagpur Plateau and Tapti.
52 D Hirakud Dam is built across the Mahanadi River, about 15 km from
Sambalpur in the state of Odisha . it is longest dam in the world.
53 D West flowing rivers are Narmada,mahi,luni,tapti etc
54 C Mudumalai Sanctuary is situated in the Tamil Nadu
55 C Discharge of effluents into water
56 C "Sustainable Development" means Balance between Protection of
Environment and the need for development.
74 C WIPO means The World Intellectual Property Organization . WIPO
was created in 1967.
75 A Moshe Katsav is an Israeli former politician who was the eighth
President of Israel from 2000 to 2007.
76 B Wen Jiabao served as the sixth Premier of the State Council of the
People's Republic of China
77 AT present – Democratic party(Barack Hussein Obama II is an
American politician serving as the 44th President of the United
States)
78 B IIM are a group of 19 public, autonomous institutes of management
education and research in India.
79 B Mithali Raj is the captain of the Indian Women's cricket team. At the
2013 Women's World Cup, Mithali Raj starred as the No.1 Cricketer
in the ODI chart among women. Raj is the second person to achieve
5000 runs in ODI women cricket.
80 B Basically the commission had only a Chief Election Commissioner.
Two additional Commissioners were appointed to the commission for
the first time on 16 October 1989, but they had a very short tenure—
until 1 January 1990. The Election Commissioner Amendment Act,
1993 made the Election Commission a multi-member body. The
concept of a 3-member Commission has been in operation since then.
81 C Late John Paul II was pope from 1978 to 2005.
82 The 10th World Bamboo Congress was held in koria in 2015
83 D
84 B The Lord of the Rings is a film series consisting of three high fantasy
adventure films directed by Peter Jackson.
85 C A viral infection that's serious for small children
86 B Alzheimer is a progressive disease that destroys memory and other

67
important mental functions.
87 B Particle Physics is the branch of physics that deals with the
properties, relationships, and interactions of subatomic particles.
88 C Udaipur is a city of Rajasthan, formerly the capital of the Rajput
kingdom of Mewar, Founded by Udai Singh II in 1559,also known as
city of lakes.
89 A The oldest mountain range in India is Aravalli,also known as residual
mountain,located in - the Indian states of Gujarat, Rajasthan, and
Haryana, ending in Delhi( Raisina hill ). The highest peak of Aravalli
is Guru Shikhar in Mount Abu in Rajasthan.
90 A Roger Federer is a Swiss professional tennis player who is currently
ranked world No. 3 by the A.T. P.(2016).
91 Dato' Sri Haji Mohammad Najib bin Tun Haji Abdul Razak is the
sixth and current Prime Minister of Malaysia (2016)
92 B International Broadcasting Day has been celebrated every year, on
December 10.
93 But at present David John Richardson is a South African former
cricketer and current CEO of the International Cricket Council.(2016)
94 A Rajyavardhan Singh Rathore is a Indian shooter and politician,
winning the Silver Medal in Men's Double Trap at the 2004 Summer
Olympics in Athens. currently he is the Minister of State for Ministry
of Information and Broadcasting under the Narendra
Modi government.

2006
Question Answer Explanation
number
23 A Human immunodeficiency virus infection and acquired immune
deficiency syndrome is a spectrum of conditions caused by
infection with the human immunodeficiency virus (HIV).
24 C A vitamin is one of a group of organic substances. Vitamins A,
D, E and K are fat-soluble and .Vitamins C and all the B
vitamins are water-soluble.

25 B Leprosy is a chronic, curable infectious disease mainly causing


skin lesions and nerve damage.
26 B Calcium was discovered by Sir Humphry Davy.
Approximately 99 percent of the calcium in the human body is
in the bones and teeth
27 D The current national calendar of India is based on the Saka
Calendar, was adopted on March 22, 1957 .
28 C The Din-i Ilahi was propounded by the Mughal emperor Akbar
in 1582 AD.
29 A Both A and R rate true and R is the correct explanation of A.

68
30 A The Dual Government of Bengal was a double system of
administration, which was introduced by Robert Clive in 1765
after battle of Buxar . The East India company obtained the
actual power; where as the responsibility and charge of
administration was entrusted to the Nawab of Bengal. It was
abolished by Warren Hastings.

31 A On June 23rd 1757 the East India Company under Robert


Clive defeated the army of Siraj-ud-daulah( the Nawab of
Bengal) in the battle of Plassey.
32 B Right to constitutional remidies under article 32 to 35
empowers the citizens to move a court of law in case of any
denial of the fundamental Right.According to Dr Ambedkar
constitutional remidies is soul and heart of our constitution.
34 C According to article 75 PM is appointed by the president,he is
the chief of the Govt. chief adviser to the President , head of the
Council of Ministers and the leader of the majority party in the
parliament.
35 D Parliamentary system was borrowed from the constitution of
England in the Indian constitution.
36 D The President is indirectly elected by the people through elected
members of both the houses of Parliament and the Legislative
Assemblies of all the states.(article 54)
37 A The Government of India has three different independent
branches namely the Executive, the Legislative and the
Judiciary. The different types of courts form the different levels
of judiciary in the country. The apex court of India is the
Supreme Court, followed by the high courts in different states.
The high courts are followed by the district courts and
subordinate courts which are also known as the lower
courts.The feature of Indian judiciary are –
1-Judicial activism
2-Judicial review
3-Independence Judiciary
4-Guardian of fundamental right
5-Separation of judiciary from executive
6-Supreme court as the Arbiter of legal dispute between the
union and states.
7-Judiciary as the interpreter of constitution.
8-Single and intregated judiciary system etc.
38 C The Rjya sabha is a upper house of the Parliament .The
maximum Membership of Rajya Sabha is 250, but current
membership is 245 . Most of the members of the House are
indirectly elected by state legislatures using single transferable
votes. President can nominate 12 members for their
contributions to art, literature, science, and social services.

69
Members sit for six-year terms, with one third of the members
retiring every two years.
39 C According to article 103 President is authorized to decide over
a dispute regarding disqualification of member of the
parliament on advice of chief Election commissioner.

40 D Attorney general of India is the Indian government's chief legal


advisor. He is appointed by the President of India under Article
76 and holds office during the pleasure of the President. The
Attorney General has the right of audience in all Courts in India
as well as the right to participate in the proceedings of
the Parliament, though not to vote.
41 B Defamation means the action of damaging the good reputation
of someone; slander or libel.
42 D The Association of south east asian nation is a geo-poltical and
economic organization of ten countries located insouth east
asia.The countries are-
Singapore,Myanmar,Malaysia,Philippines,Indonesia,Laos,Cam
bodia,Burnei,Thailand & Vietnam.ASEAN was founded in
1967.
43 C Wall street is a street in lower Manhattan that is the original
home of the New York Stock Exchange.
44 D National Development Council is an extra-constitutional and
non-statutory body. NDC is an advisory body to Planning
Commission but it's advice is not binding. It is presided by
the PM. It was set up on 6 August 1952.It is also called super
cabinet.
45 C The origin of Indian textile can be traced to the Indus valley
civilization.In Medieval period Banaras,Bengal ,Multan &
bharuch etc.are the main center of the textile Industry.In 19th
&20th centuary the cotton textile in India was initiated with the
establishment of the first cotton textile factory at fort glaster
near Culcutta in 1818 but it is closed down very soon.Actual
development of the industry had been taking place since
1854(by parsi merchant k.Nanabhai) with the establishment of
the cotton mill at Bombay.
46 B The Balance of trade is the difference between the monetary
value of a nation's exports and imports over a certain period.
47 A Chilika Lake(salt lake) is a brackish water lagoon lake in
Odisha, Wular Lake(tectonic lake) is one of the largest fresh
water lakes in Asia located in Jammu and Kashmir, The
Sambhar Salt Lake is India's largest inland salt lake located in
Rajasthan and Periyar
Lake is located in Kerala.
48 B Thien Dam(Ranjeet singh dam) is part of a hydroelectric
project constructed by the Government of Punjab on the Ravi

70
River in the state of Punjab.
49 C Afforestation is the establishment of a forest an area where
there was no forest. At present only 23% of India is covered
by the forest.
50 D Dachigam National Park is located in Jammu and Kashmir.
51 C
71 A The Devil's Dictionary is a satirical dictionary written by
American journalist and author Ambrose Bierce, published in
1906
72 A The first General Elections were held in India between 25
October 1951 and 27 March 1952.
73 B Bankim Chandra Charterjee was the composer
of India's national song Vande Mataram in1882 ( Anandamath).
It was written in Bengali and Sanskrit language. It was first
sung at the 1896 session(culcutta) of the Indian National
Congress
74 C Jana Gana Mana is our national anthem, adopted by
the Constituent Assembly as the National Anthem on 24
January 1950. It was first sung on 27 December 1911 at
the Calcutta congress session
75 B Attorney General of India is a chief legal advisor of Govt of
India. He is appointed by the President ( Article 76) and holds
office during the pleasure of the President. M. C. Setalvad was
the first Attorney General for India and current Attorney
General is Mukul Rohtagi(14 th)

76 B The Family Court Act’ was passed in the year of 1984.This Act
to provide for the establishment of Family Courts with a view
to promote conciliation in, and secure speedy settlement of,
disputes relating to marriage and family affairs and for matters
connected therewith
77 B May 17 is observed as World Information Day

78 D UNESCO means united Nations Educational,Scientific and


Cultural Organization is a agency of UN.At present it has 195
member and its headqrater is in Paris.
80 - At present chairman of Law commission is BS Chauhan(21st ).
The first Law Commission was established during the British
Period in 1834 under the chairmanship of Lord Macauley. The
first Law Commission of independent India was established in
1955. The Chairman of this Commission was Mr. M. C.
Setalvad, who was also the First Attorney General of India. The
term of this Commission was established as three years.
81 A Prafullachandra Natwarlal Bhagwati was the 17th Chief Justice
of India(1985-1986). he introduced the concepts of Public
Interest Litigation in India. The first PIL known as Husanara vs

71
bihar state in 1979.
82 B M. C. Setalvad, was the First Attorney General of India Who
was also the first chairman of LAW commission.
85 C The “Universal Declaration of Human Rights’ was adopted by
the UN in 10TH December 1948
86 - The first chief election commissioner was Mr Sukumar Sen &
present chief election commissioner is Mr .Nasim Zaidi(20TH ).
87 D Justice B.N. Kirpal is the Chairman of India’s First National
Forest Commission
88 C But at present Max Emilian Verstappean (Dutch ) became the
youngest driver to compete in Formula racing.
89 C London was struck by a series of bomb attacks first time on
July 7, 2005.
90 B Virender Shewag is the only Indian Cricketer to hit a triple
century in test cricket((319 against South Africa at Chennai).
*The first Test triple century in cricket history was achieved by
Andy Sandham of England against the West Indies in 1930 in
the first Test series hosted in the West Indies.

*Model Answers to NALSAR 2006 Question 23 to 90.(52 TO


70,79,83 &84 are LEGAL Gk) (provided hereinwith)

52 C a duly instituted organ of the government that administers justice,


whether on the basis of legislation, previous court decisions, or
other authoritative services
53 D A court decision that is cited as an example or analogy to resolve
similar questions of law in later cases
54 A Sovereignty is understood in jurisprudence as the full right and
power of a governing body to govern itself without any
interference from outside sources or bodies. In political
theory, sovereignty is a substantive term designating supreme
authority over some polity.
55 B Monopolies and Trade Restrictive Practices Act.
56 A There shall be two classes of advocates, namely, senior advocates
and other advocates.
57 A The Prohibition of Child Marriage Act (formerly known as Child
Marriage Restraint Act) provides for the age of marriage for a girl
who is 18 or above and boy who is 21 or above.
58 A Lok Sabha, Rajya Sabha
59 A Council of Ministers shall not exceed fifteen per cent. of the total
number of members of the House of the People.
60 B 21 years
61 B
62 B Flag Code of India, 2002.
63 A Whoever by force compels, or by any deceitful means induces,
any person to go from any place, is said to abduct that person.

72
64 C Liberty is defined by circumference of law.
65 B Civil Contract.
66 B 7 years.
67 B S. 375
68 C It provides for conditions in the Act which if followed will not
make the abortion illegal.
69 Option C Abdication. Renunciation of the privileges and prerogatives of an
wrong. office.
Abduction. Whoever by force compels, or by any deceitful means
induces, any person to go from any place, is said to abduct that
person.
Abatement. Reduction.
70 D
79 A Bank holidays are generally public holidays. These are the days
when banks don't operate. In India bank holidays are declared
under the Negotiable Instrument Act, 1881.
83 B J.L. Nehru
84 B Charles Dickens

2007
Question Answer Explanation
number
23 B Bat are flying mammals.The bat produces an ultrasonic sound.The
ultrasonic sound reflect from the objects is received to the bat , then
he change his flight pattern.
24 B The first atomic reactor of India is known as Apsara.Apsara is the
oldest of India's research reactor, designed by the BARC and built
with the help of United Kingdom in 1956. Apsara is a light water
swimming pool-type reactor.Other reactor are- CIRUS (1960 With
the help of Canada), ZERLINA (1961, Zero Energy Reactor) ,
Purnima I (1972), Purnima II (1984), Dhruva (1985), Purnima III
(1990), KAMINI and Kalpakkam( India's first Pressurised water
reactor).

25 A With the help of Periscope instrument can a man in a submarine see


objects on the surface Of sea.A Periscope works by using two mirrors
to bounce light from one place to another.

26 C It has to be real. Otherwise we cannot get print.


27 C Ibn Batuta was a Moroccan Muslim traveler , who wrote a famous
book Rihla. Ibn Battuta was a guest at Muhammad bin Tughlauq’s
court.
28 B Lord Rippon was appointed as the viceroy of India(1880-84) by
Gladstone(PM of England).Rippon believed that self Govt. is the
highest and noblest principles of politices.so he introduced Local self
Govt. in India in 1882.He was also introduced first factory

73
act(1881),Hunter commission(1882) and Ilbert bill (1883).
29 B Hoysala sculputure reffer to as an amalgamation between the
Dravidian and Indo-Aryan sculpture . Hoysala sculptures are carved
in soft chloristic schism.In Hoysala sculpture the predominant motifs
found the temple wall include the deites of gods & goddesses and
animals.Thick foliage sculpture are also present on these temples.The
ceilings of the mandapas are designed with prolific sculpture.The
huge gate ways are decorated with wellcarved sculptures.The temples
at bellur and Halebeedu is the best example of Hoysala
sculpture.THE BEST EXAMPLE OF Hoysala sculpture are
Salabhanjika statue.

30 A Doctrine of lapse was an annexation policy introduced by Lord


Dalhousie(1848-56).This policy was based on the idea that in case a
ruler died childless,the right of ruling over the state reverted to the
british Govt.The first state which was annexed by Dalhousie was
Satara(1848).
31 D Ashvaghosa was an Indian philosopher-poet.He wrote
Buddhacarita(Sanskrit). Nagarjuna was the founder of
the Madhyamaka school of Mahayana Buddhism. Buddhaghoṣa was a
5th-century Indian Buddhist commentator and scholar. His famous
book is Visuddhimagga (Path of Purification).

32 C
33 D
34 C A money Bill must be returned by the Rajya Sabha with its -
recommendations to the LokSabha within 14 days.
35 D The Deputy Prime Minister is not a constitutional office. The first
Deputy Prime Minister of India was Sardar Vallabhbhai Patel.

38 D The Control of expenditure of the Government rests with The


Parliament

39 A Forest is contained in the Concurrent List. The legislative section is


divided into three lists: Union List, State List and Concurrent List(7th
schedule , 52 items ).It is borrowed from Australian constitution.
40 D Rajya sabha is the upper house . Maximum membership of Rajya
sabha is 250 , but current laws have provision for 245 members. Most
of the members of the House are indirectly elected . President can
appoint 12 members for their contributions to art, literature, science,
and social services. Members sit for staggered six-year terms, with
one third of the members retiring every two years.

41 B
42 A The Free Port used to designate areas in which companies are taxed
very lightly or not at all to encourage economic activity.

74
43 A Amnesty International is a non-governmental organisation focused on
human rights.Its headquater is in London and founded in 1961.
44 C Mazagon Dock Limited' was founded in 1934(Mumbai). The
shipyard was nationalised in 1960.
45 B Black pagoda is a Sun Temple at Konark in Odisha. It was built by
Narasimhadeva I in 1255 AD. Now the temple is a UNESCO World
Heritage Site.
46 D
47 A In Anti cyclone Isobar is oval with high pressure at the centre and it
circulation is Anti-clockwise in southern hemisphere and clockwise
in northern hemisphere.

48 A The Mississippi River is the chief river of the largest drainage system
on the North American continent.
49 A Kohoutek is formally designated C/1973 E1, 1973 XII, and 1973f,
was first sighted on 7 March 1973 by Czech astronomer Lubos
Kohoutek. It attained perihelion on 28 December that same year.
50 D Selvas are equatorial rain forest. Large part of the Amazon basin are
covered by selvas.
51 D Zoji La pass is in Kashmir between Srinagar and Leh in the western
section of the Himalayan mountain range.
64 D INTERPOL is the world's largest international police organizatioN
(190 member countries).It headquater is in Lyon(France).
65 D The International Bank for Reconstruction and Development(World
Bank) was created in 1945.its headquarter is in Washington, D.C (189
member)
66 C Magna Carta, is a charter agreed to by King John of England, on 15
June 1215.
68 B Amartaya Sen was awarded the Nobel Memorial Prize in Economic
Sciences in 1998 and Bharat Ratna in 1999 for his work in welfare
economics.
69 A Gilt edges market means Market in government securities
71 A EURO is the official currency of the eurozone, which consists of 19
of the
28 member states of the European Union. The euro was introduced to
world financial markets on 1 January 1999.
72 B Kalidasa was a Classical Sanskrit writer of the Chandragupta-2 court.
His famous book was Meghdoot, Abhigyan Shakuntala, Kumara
Sambhavam,Malvikagni Mitra and Vikram Urvashi
73 B
74 C Village Panchayats are elected by Gram Sabhas consisting of the
Entire adult population of the village.Gram panchayat a local self-
government organisation in India of the panchayati raj system at the
village and has a sarpanch as its elected head.
75 B Ms. Jenny Shipley was the 36th Prime Minister of New Zealand

75
from 1997 to December 1999.
77 C The Secretary General of the United Nations is appointed by
the General Assembly upon the recommendation of the Security
Council. The current Secretary-General is Ban Ki-moon ( South
Korea)
78 A Glan McGarth (Australian cricketer ) was the man of the series in the
finals of the World Cup Cricket 2007
79 C Italy won the tournament, claiming their fourth World Cup title.
They defeated France.
80 A Maximum term of imprisonment for Contempt of Courts is 3
months.
81 B Buddha died at the age of 80 years in 483 BCE. at Kushinagar.
82 - Technically India was a founder member of UN IN OCTOBER
1945,Despite it being a British colony.
83 B Maria Sharapova is a Russian professional tennis player.
84 B But in this time Pedro Pablo kuczynski is the president of Peru
85 - But in this time current chief jutice is Justice TS Thakur (43rd)
86 B Kiran Desai is an Indian author. Her novel The Inheritance of Loss
won the 2006 Man Booker Prize
87 - The 14th and current Attorney General is Mukul Rohatgi .
88 - Lalitha Kumaramangal is the current chairperson of the National
Commission for Women.National commission for women was was
established in January 1992 . The first head of the commission
was Jayanti Patnaik
89 B P. Bandopadhyay is the first women Vice-Air Marshal of India
90 C Australia won the tournament ,Australian bowler Glenn McGrath was
named Player of the Series.

NUJS
2000
Question Answer Explanation
number
61 A The correct answer is Pervez Musharaff
62 A The correct answer is for that year is A.S Anand.
63 A The correct answer is for that year is Soli Sorabjee.
64 B The correct answer is for that year is Harish Salve.
65 A The correct answer is for that year is Ram Jethmalani.
66 B The correct answer is for that year is G.M.C Balyogi.
67 A The correct answer is Meghana Narayan (Swimming).
68 A The correct answer is 108.
69 A The correct answer is for that year is J.S Verma.
70 A The correct answer is Audiography.
71 B The correct answer is for that year is India.
72 C The correct answer is Development Economics

76
73 The Question was deleted, as it was factually incorrect.
74 B The correct answer is Clement Atlee (1947)
75 C The correct answer is for that year is Uma Bharti.
76 B Australia Defeated Pakistan in the finals at Lords, London.
77 A The correct answer is Maharashtra
78 B The correct answer is Buddha.
79 B The correct answer is Shobha De.
80 A The correct answer is S. Jaipal Reddy.
81 A The correct answer is for that year is A.P.J Abdul Kalam.
82 B The correct answer is for that year is Yahwant Sinha.
83 A The correct answer is for that year is Maneka Gandhi.
84 B The correct answer is for that year is N. Vittal
85 C The correct answer is for that year is Jacques Chirac.
86 C The correct answer is for that year is Kumaratunga.
87 B The correct answer is for that year is UNICEF.
The United Nations Children's Emergency Fund is a United Nations
program headquartered in New York City founded on December 11,
1946 that provides long-term humanitarian and developmental
assistance to children and mothers in developing countries.
88 C The correct answer is Jaya Deva.
89 C The correct answer is for that year is P.A Sangma.
90 A The correct answer is Sahara Desert.
The Sahara is the largest hot desert in the world, and the world's third
largest desert after Antarctica and the Arctic.

2001
Question Answer Explanation
number
56 B Surat was the first factory setup by EIC.
57 D Freedom of the individual.
58 C Jamnagar, Junagarh & Jammu and Kashmir were added later by
prospective amendments.
59 D President of India.
60 D We, The People of India.
61 B It has minimal economic contacts with the rest of the world.

62 A Octroi-tax is tax levied on entry of goods into a municipal area
.
63 D In macroeconomics, the balanced-growth path of a dynamic model is
a trajectory such that all variables grow at a constant rate. In the
standard exogenous growth model, balanced growth is a basic
assumption, while other variables like the capital stock, real GDP,
and output per worker are growing.
64 A The correct answer is Silicon
65 A The correct answer is Teflon and Dacron is a polymer.
66 D The correct answer is Centripetal Force.

77
67 D The correct answer is Ultrasound.
68 B The correct answer is foreign planters.
69 A The correct answer is Nilgiri.
70 A It is in between Karnataka & Tamil Nadu.
71 A An attitude of judges acting in an independent manner
.
72 A The correct answer is Prithvi.
Prithvi (Sanskrit: pṛthvī "Earth") is a tactical surface-to-surface short-
range ballistic missile (SRBM) developed by DRDO of India under
the Integrated Guided Missile Development Program (IGMDP).
India’s Strategic Forces Command deploys it.
73 B The correct answer is Seven.
The member states are Afghanistan, Bangladesh, Bhutan, India,
Maldives, Nepal, Pakistan, and Sri Lanka. SAARC was founded by
seven states in 1985.
74 B The correct answer is Param.
PARAM is a series of supercomputers designed and assembled by the
Centre for Development of Advanced Computing (C-DAC) in Pune,
India. The latest machine in the series is the PARAM
Kanchenjunga. Param means supreme in Sanskrit
75 B Sindhus was an ancient river on which the name India was derived.
76 C The correct answer is Medha Patkar.
77 A The correct answer is Ionosphere.
78 B Drifting organisms in oceans, lakes or rivers.
79 D India & Pakistan borderline is called as Radcliffe.
80 C Amnesty International deals with issues related to human rights.
81 B Application of the knowledge of the living world characteristics to the
world of machines.
82 B S.C. Bose named Gandhi as ‘Father of the Nation’
83 C The correct answer is Ethyl Alcohol.
84 D The correct answer is Northern Ireland, Scotland, & Wales.
85 C Razia Sultan became the first women ruler in India. Raziya al-Din,
throne name Jalâlat-ud-Dîn Raziyâ, usually referred to in history as
Razia Sultan, was the Sultan of Delhi in India from 10 November
1236 — 14 October 1240. She was famously the only female ever to
rule the Delhi Sultanate.
86 C The correct answer is 1858.
87 B The correct answer is Gopal Krishna Gokhale
88 D The correct answer is Bhakti.
89 B The correct answer is December 1911.
90 B The correct answer is Dead Sea.
91 D The correct answer is Temperature, rainfall & Vegetation.
92 C It is an economic system in which transactions between private
parties are free from government interference such
as regulations, privileges, tariffs, and subsidies. The phrase laissez-
faire is part of a larger French phrase and literally translates to "let

78
(it/them) do", but in this context usually means to "let go".
93 A The correct answer is C.R Das
94 A She was the PM of Srilanka.
95 B He is associated with Commonwealth of nations.
96 C The correct answer is British Author.
97 A First President of China Republic.
98 B Plato was disciple of Socrates.
99 B State Government owns it.
100 B It rises higher.
101 C Carbon Dioxide is often used in fire extinguishers.
102 B Convex lenses are used for short sight corrections, whereas concave
lenses for long sight corrections.
103 D An agrarian economy is rural, rather than urban, based. It is centered
upon the production, consumption, trade, and sale of agricultural
commodities, including plants and livestock.
104 B The correct answer is Textiles.
105 B Communist party of India in Kerala.
106 D Secularism in India stands for the state having no religion of its own
rather all religion shall be treated as equal.
107 D B.C Chatterjee composed Vande Mataram —literally, "I pray/bow
down to thee, Mother"—is a poem from Bankim Chandra
Chattopadhyay's 1882 novel Anandamath. It was written in Bengali
and Sanskrit. It is a hymn to the Mother Land.
108 A The correct answer is preventing felling of trees.
109 B The correct answer is Seismograph.
110 C Narmada River.

2002
Question Answer Explanation
number
56 B The correct answer is Dalai Lama.
57 B The correct answer is Prevention of Terrorism Ordinance.
58 D The correct answer is Doha.
59 B The correct answer is Nalanda.
60 C The correct answer is Zahir Shah.
61 C The correct answer is Medha Patkar.
62 C The correct answer is Mamallapuram.
63 C The correct answer is prairies.
64 C The correct answer is Karnal.
65 C The correct answer is Nuclear reactor.
66 B The correct answer is Washington D.C.
67 A NH-7 is the longest road about 770 Kms. National Highway 7 is a
highway connecting Fazilka to Mana in India.
68 C The correct answer is Mumbai Riots.

79
69 B The correct answer is Rubber.
70 A The correct answer is East Africa.
71 D The correct answer is IBM.
72 A The correct answer is Chipko Movement.
73 D The correct answer is Subramanyam Chandrashekhar.
74 B She was the PM of Srilanka.
75 B The answer might be correct for that year i.e. Mizoram. However the
answer for now 2016 shall be Kerala.
According to the 2011 census, literacy level is 93.91 per cent
in Kerala and 91.58 per cent in Mizoram, among the most literate
states in the country. The national literacy rate, according to the 2011
census, is 74.04 per cent.
76 B The correct answer is M.N. Roy.
77 D The correct answer is Vhava Sheve.
78 A The correct answer is Nathula Pass
Nathu La is a mountain pass in the Himalayas. It connects the Indian
state of Sikkim with China's Tibet Autonomous Region. The pass, at
4,310 m above mean sea level, forms a part of an offshoot of the
ancient Silk Road.
79 B The correct answer is Secret society of revolutionaries
.
80 D The answer Switzerland might be correct for that year however it
shall be incorrect now (2016).
81 A The correct answer is Aurobindo Ghose.
82 A The correct answer is AIDS.
83 C The correct answer is Arunachal Pradesh.
84 A The correct answer is Durban.
85 A The correct answer is Nepal.
86 A The correct answer is Glacier.
87 B The correct answer is Lithosphere.
88 D The correct answer is Bhrama Gupta.
89 A The correct answer is Biometry.
90 B The correct answer is Justice N.M. Venkatachalliah.
91 C The correct answer is Justice Venkataswami Committee.
92 B The correct answer is Subramanyam Committee.
93 A The correct answer is Nishant.
94 C Army Vocational Training and Rehabilitation
95 C The correct answer is Mumbai.
96 C The correct answer is A.P.J Abul Kalam.
97 B World Human Rights Day

98 A The correct answer is Al Quaeda.
99 D The correct answer is K.R. Narayan.
100 B The correct answer is Jamshedpur.
101 B The correct answer is International Standard Book Numbering.
102 C The correct answer is Vibratory Motion.
103 D Due to gravitational force which remains to be the same 9.8 m/s.

80
104 B The law of inertia is the basis of the new physics of the seventeenth
century. This law is also true according to modern
physics. Galileo discovered the law during the first decade of the
seventeenth century, but in fact he did not understand the law in the
general way we have formulated it here. Descartes - a French
philosopher, mathematician and physicist, devised by Galileo’s pupils
and the general formulation of the law of inertia. This law is also the
first of Newton's three laws.
105 C Due to change in gravitational force.
106 B It is called as Sphygmomanometer.
107 C The correct answer is J.L. Braid.
108 C The correct answer is (95-110) Degrees Fahrenheit.
109 C Abraham Lincoln abolished Slavery.
110 A The correct answer is Khan Abdul Gaffar Khan.

2003
Question Answer Explanation
number
01 D The non-cooperation movement was a significant phase of the Indian
independence movement from British rule. Mahatma Gandhi led it after
the Jallianwala Bagh Massacre. It aimed to resist British rule in India
through nonviolent means.
The movement was withdrawn because of the Chauri Chaura incident.
Although he had stopped the national revolt single-handedly, on March
10, 1922, Gandhi was arrested.
02 D Portuguese were the first Europeans (1505) to enter India and last
(1961) to leave India.
03 A The correct answer is Delhi.
04 B He was also the political Guru of Gandhi.
05 D The correct answer is 1911. It was shifted from Calcutta to Delhi.
06 C B.C Chatterjee composed Vande Mataram —literally, "I pray/bow down
to thee, Mother"—is a poem from Bankim Chandra Chattopadhyay's
1882 novel Anandamath. It was written in Bengali and Sanskrit. It is a
hymn to the Mother Land.
07 B The correct answer is Sir Syed Ahmed Khan.
08 B Equality of all religion.
09 C Determine the constitutionality of law or executive action
.
10 B Collegium System consisting of panel of Senior-most Judges of
Supreme Court.
11 D They were elected from provincial assemblies.
12 D It was removed by the 44rth amendment 1978.
13 C The correct answer is Election Commission.
14 C The correct answer is Habeas Corpus.
15 B The correct answer is Jammu & Kashmir.

81
16 C The correct answer is Vinobha Bhave.
17 B The correct answer is 250 BC.
18 A The correct answer is 10 December 1948, henceforth celebrated as
Human Rights Day.
19 B The correct answer is Development Theory (Economics).
20 D The correct answer is Equality before law.
21 C The correct answer is Huntington.
22 C The correct answer is 26 November 1949, Henceforth celebrated as Law
day.
23 D The correct answer is Isthmus.
24 A The Earth is closest to the Sun, or at the Perihelion, about two
weeks after the December Solstice, when it is winter in the Northern
Hemisphere. Conversely, the Earth is farthest away from the Sun, at the
Aphelion point, two weeks after the June Solstice, when the Northern
Hemisphere is enjoying warm summer months.
25 A It’s between Afghanistan & Pakistan.
26 B The correct answer is Kerala.
27 A Narmada, Sons & Tapti is all west flowing river hance falls in the
Arabian Sea.
28 B Parts of the ocean near the equator which is calm with light winds.
29 B It rotates from west to east.
30 C The correct answer is Beas.
31 C The correct answer is Earthquake.
32 C Standard of Quality.
33 C The correct answer is Vitamin C.
34 D The correct answer is Badminton.
35 D The correct answer is Spanish.
Pablo Ruiz y Picasso, also known as Pablo Picasso, was a Spanish
painter, sculptor, printmaker, ceramicist, stage designer, poet and
playwright who spent most of his adult life in France.
36 B The correct answer is Hazrat Ali.
37 C The Sun is a main-sequence star, and thus generates its energy
by nuclear fusion of hydrogen nuclei into helium. In its core, the Sun
fuses 620 million metric tons of hydrogen each second.
38 B The correct answer is Brain.
An inherited condition in which nerve cells in the brain break down
over time.
39 C The correct answer is Sher Shah Suri.
40 D The correct answer is Bal Gangadhar Tilak.
41 C The correct answer is Salt Tax.
42 B The correct answer is Garha.
43 B The correct answer is Lord Curzon.
44 A The correct answer is Andhra Pradesh.
45 D The correct answer is Negroid.
46 C The correct answer is Losses. Net national product (NNP) refers

82
to gross national product (GNP), i.e. the total market value of all
final goods and services produced by the factors of production of a
country or other polity during a given time period,
minus depreciation.[1] Similarly, net domestic product (NDP)
corresponds to gross domestic product (GDP) minus depreciation.
47 A The correct answer is Gold and foreign securities
48 D Expensive commodities become more expensive.
49 B Destroys harmful bacteria.
50 B The correct answer is Zinc
51 B It remains constant.
52 D Manohar Joshi vs Nitin Bhaurao Patil & Another on 11 December,
1995.
53 B The correct answer is Carrot.
54 B The correct answer is Portugal.
55 A The correct answer is Handicrafts.
56 D The correct answer is Mould.
57 A The correct answer is Population.
58 B The correct answer is Sri Aurobindo Ghose.
59 D The correct answer is Bibhuti Bhusan Banerjee.
60 C The correct answer is Gobar Gas.
61 D The correct answer is Geneva.
62 A The correct answer is Acetylene.
63 C The correct answer is Chlorine.
64 D The correct answer is 72 mts.
65 A The correct answer is Virus.
66 B The correct answer is 24 hours.
67 C The correct answer is Mulberry.
68 D The correct answer is xy.
69 A The correct answer is PAN.
70 C The correct answer is Robert Koch.
71 D The correct answer is Manduka Upanishad.
72 B The correct answer is ADR (Alternate Dispute Resolution).
73 D India is a global agricultural powerhouse. It is the world's largest
producer of milk, pulses, and spices, and has the world's largest cattle
herd (buffaloes), as well as the largest area under wheat, rice and cotton.
74 C The correct answer is 0.5%.
75 B The correct answer is thermal.
76 C The correct answer is Kautilya.
77 B The correct answer is Dayanand Saraswati.
78 C It is mentioned as Fundamental duties under article 51-A (h) of the
Constitution of India.
79 A The correct answer is President.
80 A The correct answer is 1952.

83
2004
Question Answer Explanation
number
01 D Sheep were the first to be domesticated.
02 C The correct answer is Rajendra Chola I.
03 C He led second biggest army after EIC.
04 B The correct answer is Shahjahan.
05 B The correct answer is Samudra Gupta
06 A The correct answer is Bhrama Gupta
07 A Mars - the red planet. Mars is often called the 'Red Planet' because it
appears in the sky as an orange-red star. The colour caused the ancient
Greeks and Romans to name it after their god of war. Today, thanks to
visiting spacecraft, we know that the planet's appearance is due to rust
in the Martian rocks.
08 C Its equal at Equator.
09 C The difference between India and that of England is 5 hours and 30
mins. England is behind by 5.5 hours to India.
10 B Movements of plates of earth. Which float on the hot volcanic mantle
below the earth's surface.
11 C The correct answer is Rio-De-Janeiro.
12 C The correct answer is Bhutan & Nepal.
13 B The question might be correct for that year. However Afghanistan is
now the Member of the SAARC.
14 C The correct answer is Indian Ocean.
15 D The correct answer is Geneva.
16 C The correct answer is Tropical.
17 C The correct answer is Jowar.
18 C The correct answer is Chemical Industry.
19 C The number of people living per square kms.
20 D The correct answer is New Zealand.
21 C The correct answer is India.
22 C It flows from Karnataka.
23 B The correct answer is Meghalaya.
24 D The correct answer is Bangladesh.
25 D The correct answer is Tulsidas.
26 A The correct answer is Portuguese.
27 D The correct answer is Young India.
28 C The correct answer is Vangaurd.
29 A Sarojani Naidu was elected as the governor of U.P.
30 A The correct answer is M. Visvesvaraya.
31 C The correct answer is Trsuteeship.
32 B Freely permitting the conversion of rupee to other major currencies and
vice versa
.
33 C The correct answer is 1969.
34 C The correct answer is Raj Krishna.

84
35 D Recession a period of temporary economic decline during which trade
and industrial activity is reduced generally identified by a fall in GDP in
two successive quarters.
36 A Multi National is a company operating in several countries.
37 C The correct answer is Adam Smith.
38 B The correct answer is Rivandra Varma.
39 C The correct answer is Washington D.C.
40 D The correct answer is 1999.
41 D The policy steps which the borrower country must take
.
42 C The correct answer is 2003.
43 B The correct answer is 2003.
44 B The correct answer is Hyderabad.
45 C The correct answer is China.
46 A The correct answer is 14.
47 B The correct answer is Mexico.
48 C The correct answer is Stephen Hawking.
49 C The correct answer is Agriculture.
50 B The correct answer is Dev Anand.
51 B The correct answer is Athletics.

2005
Question Answer Explanation
number
01 B The front crawl or forward crawl is a swimming stroke usually regarded
as the fastest of the four front primary strokes. As such, the front
crawl stroke is nearly universally used during a
freestyle swimming competition; hence freestyle is used metonymically
for the front crawl.
02 C Dennis Anthony Tito is an American engineer and multimillionaire,
most widely known as the first space tourist to fund his own trip into
space.
03 B The Correct Answer is Bhutan.
04 C The Answer might be correct for that year however the answer now in
2016 would be 12.
Three each from India and Australia.
Two from West Indies & Srilanka.
One each from South Africa & England.
05 D The correct answer is Agatha Chistie.
06 D The Correct Answer is None of the above.
El Hombre Dorado:-
Described as (the golden man), El Indio Dorado (the golden Indian),
or El Rey Dorado (the golden king), is the term used by Europeans to
describe a mythical tribal chief of the Muisca native people of
Colombia.

85
07 A The Correct Answer is Doctors.
08 B Abu'l-Fath Jalal-ud-din Muhammad Akbar, popularly known as Akbar I
and later Akbar the Great, was Mughal Emperor from 1556 until his
death. He was the third and one of the greatest rulers of the Mughal
Dynasty in India.
09 A The Correct Answer is Russia followed by Iran and then Qatar.
10 A On January 30, 1948, Mahatma Gandhi fell to his assassin Nathuram
Vinayak Godse's bullets during an evening prayer ceremony at Birla
House in Delhi.
11 B The Correct Answer is Argentina defeated Paraguay 1-0.
12 C The correct answer is December 17, 1903.
13 B Marsh gas, swamp gas and bog gas are common names for biogas which
forms in wetlands, whose principal component
is methane with hydrogen sulfide and carbon dioxide as minor
compositions, produced naturally within some
geographical marshes, swamps, and bogs.
14 C The correct answer is Thomas Edison.
15 C The correct answer is Aurobindo Ghosh.
16 A The correct answer is Mahmoud Abbas.
17 A The correct answer is Harbour wave.
18 B The correct answer is Birsa Munda.
19 A The correct answer is Indira Gandhi.
20 C Harsha had his capital at Kanauj.
21 A The correct answer is Shankaracharya.
22 B The correct answer is September 8.
23 C Lake Baikal is the deepest lake
Lake Baikal is an ancient, massive lake in the mountainous Russian
region of Siberia, north of the Mongolian border. Considered the
deepest lake in the world, it’s circled by a network of hiking paths
called the Great Baikal Trail. The village of Listvyanka, on its western
shoreline, is a popular starting point for summertime wildlife-spotting
tours, plus wintertime ice-skating and dog sledding.
24 B Wilhelm Roentgen discovered it.
25 D The correct answer is Davos.
26 B Raslila is based on life of Lord Krishna.
27 B Ravi Shankar founded the Kinnara School of Music in Mumbai.

28 D It’s a state of limited competition, in which a market is shared by a


small number of producers or sellers.
29 C John Maynard Keynes, 1st Baron Keynes, CB, FBA, was an English
economist whose ideas fundamentally changed the theory and practice
of macroeconomics and the economic policies of governments.
30 B David Ricardo was an English political economist. He was one of the
most influential of the classical economists, along with Thomas
Malthus, Adam Smith, and James Mill.

86
31 D The correct answer is Mynmmar.
32 B It was established on January 25, 1950.
33 D The correct answer is Douglas Engelbart.
34 C The correct answer is Intel 8080.
35 B The correct answer is Egyptian.
36 C The correct answer is Mulk Rai Anand.
37 A Devdas is a Bengali Romance novel by Sarat Chandra Chattopadhyay
written when he was only seventeen years of age on 30 June 1917.
38 B Samuel Phillips Huntington (April 18, 1927 – December 24, 2008) was
an American political scientist, adviser and academic. He spent more
than half a century at Harvard University, where he was director of
Harvard's Center for International Affairs and the Albert J. Weatherhead
III University Professor. During theCarter administration, Huntington
was the White House Coordinator of Security Planning for the National
Security Council. He is most well known by his 1993 theory, "The
Clash of Civilizations", of a post-Cold Warnew world order. He argued
that future wars would be fought not between countries, but between
cultures, and that Islamic extremism would become the biggest threat to
Western world domination. Huntington is credited with helping to shape
U.S. views on civilian-military relations, political development, and
comparative government.
39 C The correct answer is Ranajit Guha
40 C The correct answer is Ronald Coase.
41 B The correct answer is Anti caste
42 C The correct answer is B.R. Ambedkar.
43 C The correct answer is Andhra Pradesh
44 A The correct answer is Annie Basant.
45 B The correct answer is Edward Said.
46 C The correct answer is John Lennon.
47 C The correct answer is 28.
48 B The correct answer is Films.

2006
Question Answer Explanation
number
01 A Hon’ble Mr. Justice R.C. Lahoti.
02 D James Braid (19 June 1795 – 25 March 1860) was a Scottish surgeon
and "gentleman scientist". He was a significant innovator in the
treatment of clubfoot and an important and influential pioneer
of hypnotism and hypnotherapy. Many as the first genuine
“hypnotherapist” and the “Father of Modern Hypnotism” regard him.
03 C Douglas Engelbart invented the Mouse of the computer.
04 C Mainly used during the process of CT Scan
05 A Adool Gopalakroshnan was awarded the Dada Phalke award in 2005.

87
06 A The correct answer is Vikram Seth.
07 A Justice Leila Seth was the first woman judge on the Delhi High Court
and the first woman to become Chief Justice of a State High Court.
08 C The correct answer is Gutteres
09 C The correct answer is John Roberts
10 C The correct answer is A Tactless Mistake.
11 B The correct answer is Karl Marx.
12 B The correct answer is Mohd. Elbaradaei.
13 B The correct answer is Immanuel Kant
14 A The correct answer is 3 December.
15 A The correct answer is Felling of trees.
16 C The correct answer is Nani Palkhiwala
17 C Our Research team appreciates the question.
The correct answer is Mahatma Gandhi.
18 A The correct answer is Value Added Tax.
19 C The answer for that year might be correct but the current membership
(2016) comprises of 28 Countries.
The EU countries are: Austria, Belgium, Bulgaria, Croatia, Republic of
Cyprus, Czech Republic, Denmark, Estonia, Finland, France, Germany,
Greece, Hungary, Ireland, Italy, Latvia, Lithuania, Luxembourg, Malta,
Netherlands, Poland, Portugal, Romania, Slovakia, Slovenia, Spain,
Sweden and the UK.
20 A The correct answer is Peacebuilding Commission.
21 B The correct answer is Jagdish Bhagwati
22 C The correct answer is Honk Kong.
23 C The correct answer is Nadia Petrova
24 A The correct answer is Paris
25 B Sir Roger Penrose OM FRS (born 8 August 1931) is an
English mathematical physicist, mathematician andphilosopher of
science. He is the Emeritus Rouse Ball Professor of Mathematics at
the Mathematical Instituteof the University of Oxford, as well as an
Emeritus Fellow of Wadham College.
26 C It is named after Planet Saturn.
27 A Suryakant Thripathi is also named as ‘Nirala’.
28 D It emerged during Kushan Dynasty.
29 D The correct answer is Sutlej.
30 B Michael Madhusudan Dutt, or Michael Madhusudan Dutta was a
popular 19th-century Bengali poet and dramatist. He was born in
Sagordari, a village in Keshabpur Upazila, Jessore District of Undivided
Bengal.
31 A It’s a period of temporary economic decline during which trade and
industrial activity are reduced, generally identified by a fall in GDP in
two successive quarters.
32 D An economic theory of total spending in the economy and its effects on
output and inflation. Keynesian economics was developed by the British

88
economist John Maynard Keynes during the 1930s in an attempt to
understand the Great Depression.
33 A The correct answer is Sabha.
34 C The correct answer is Bangladesh.
35 C The correct answer is 5600 Kms.

2007
Question Answer Explanation
number
01 C Medha Patkar (born 1 December 1954) is an Indian social activist and
social reformer turned politician. She is the founder member
of Narmada Bachao Andolan and was National Convener of National
Alliance of People's Movements (NAPM), an alliance of progressive
people's organisations. She was a representative to theWorld
Commission on Dams, to research the environmental, social and
economic impacts of the development of large dams globally. National
Alliance of People's Movements, the organisation she leads, filed a
public interest litigation in the Bombay High Court against Lavasa.
02 A World Bank HQ at Washington DC
03 Our Research team finds the question to be factually incorrect.
Since 2015, the flags of the two non-member observer states are raised
alongside those of the 193 member states. There are 193 United
Nations (UN) member states, and each of them is a member of the
United Nations General Assembly.
South Sudan being the 193rd member.
04 Our Research team finds the question to be factually incorrect.
1st December is Worlds AIDS Day.
The red ribbon is the global symbol for solidarity with HIV-positive
people and those living with AIDS. World AIDS Day, observed on 1
December every year, is dedicated to raising awareness of the AIDS
pandemic caused by the spread of HIV infection.
05 D Portuguese were not only the first to visit but also last to exit.
06 B State does not have a religion of its own; it respects and treats all
religions as equal.
07 A The power of the courts to examine the constitutionality of legislation
and executive acts
.
08 B Collegium System comprising of Judges of SC.
09 D Right to property was removed by 44rth Amendment 1978.
10 B Habeas Corpus is a writ that is used to bring a party who has been
criminally convicted in state court into federal court. Usually, writs
of habeas corpus are used to review the legality of the party's arrest,
imprisonment, or detention.
11 D Article 370 talks about special status of J& K.
12 C Jayaprakash Narayan gave the call for total revolution.
13 D It is written by Thomas Friedman

89
14 D Making Globalization Work is a book written by Nobel Prize laureate
Joseph E. Stiglitz, who also wrote Globalization and Its Discontents and
several other books.
15 D Lok Adalat is a non-adversarial system, whereby mock courts
(called Lok Adalats) are held by the State Authority, District Authority,
Supreme Court Legal Services Committee, High Court Local Services
Committee, or Taluk Legal Services Committee.
16 C During the pleasure of the President.
17 B Justice M. Fathima Beevi was the first female judge to be appointed to
the Supreme Court of India (1989).
18 A Tactless Mistake
19 C Igneous Rocks.
20 A It is related to Theory of Economic Determination.
21 A Written by Arthur Llewellyn Basham (24 May 1914 – 27 January 1986)
were a noted historian and indologist and author of a number of books.
As a Professor at the School of Oriental and African Studies, London in
the 1950s and the 1960s, he taught a number of famous Indian
historians, including Professors R.S. Sharma, Romila Thaparand V.S.
Pathak.
22 C He was 37th CJI of India.
23 D Tata Nano Singur Controversy refers to the controversy generated
by land acquisition of the proposed Nano factory of Tata Motors at
Singur in Hooghly district, West Bengal, India.
24 A Formed on July 2005
Sam Pitroda was appointed as its chairperson.
25 B The answer might be correct for that year
Currently it’s Anurag Thakur (2016).
26 B Sania Mirza (Tennis) Youngest Player to Achieve Padma Shree at 19.
27 D It is Banker of government as well as Banker of Banks.
28 C National Common Minimum Programme.
29 B Adam Smith is called the father of Economics.
30 A Stock Exchange
31 C Finance Minister represents the Budget every year.
32 C A white paper is an authoritative report or guide that informs readers
concisely about a complex issue and presents the issuing body's
philosophy on the matter. It is meant to help readers understand an
issue, solve a problem, or make a decision.
33 A Domestic industry to be treated at par with foreign industries.

34 C Psychographics is the study of personality, values, opinions, attitudes,


interests, and lifestyles. Because this area of research focuses on
interests, attitudes, and opinions, psychographic factors are also called
IAO variables.
35 A It is located at Kolkata.
36 B Milon Kumar Banerji (c. 1928 – July 20, 2010) was an Indian jurist who

90
was Attorney General of India from 1992 to 1996 and again from 2004
to 2009. He was also Solicitor General from 1986 to 1989. He died on
July 20, 2010 after a long illness, having suffered a stroke in December
2009. He was 82.
37 B He addressed the speech at Chicago on September 15, 1893.
38 A Hinduism is considered as the oldest religion.
39 B It is located in Karnataka.
40 C Rig Veda is the oldest Vedas among all four Vedas.
41 C Vijneshwara wrote Mitakshara.
42 D Dr. Rajendra Prasad was elected as the President of the constituent
assembly on September 11, 1946.
43 B Warren Hasting- Regulating Act (1773).
44 C It started from Meerut.
45 D The correct answer is Mugal Emperor Jahangir.
46 C Preservation of Turkish empire with the Khilaftat as the temporal head
of Indian Muslims.
47 B Khan Abdul Gaffar Khan.
48 A Satyajit Ray received it in 1992.
49 A The Question was Modified.
50 C
51 B The Question was Modified.
52 D The Question was Modified.
53 A
54 B The Question was Modified.
55 C
56 D Amitabh Bachchan is an Actor not a Musician.
57 B It is located at Orissa, now called as ODIA.
58 B Santhals have the maximum tribal population.
59 D No effects to domestic prices.
60 C Kumar Gandharva or Shivaputra Siddharamayya Komkalimath was a
Hindustani classical singer, well known for his unique vocal style and
his refusal to be bound by the tradition of any gharana.

CLAT
2008
Question Answer Explanation
number
41. D
42. C A hedge fund is an alternative investment vehicle available only to
sophisticated investors, such as institutions and individuals with
significant assets. Hedge funds are pools of underlying securities.
43. A

91
44. B
45. B Morarji Desai has presented 10 budgets in toto. Subsequent to him is P.
Chidambaram who has presented 8 budgets up until now.
46. A
47. C
48. A Vijay Mallaya was the purchaser of this sword.
49. C Built on the Chenab River.
50. C
51. B Kishore Biyani is an Indian businessman. He is the founder and chief
executive officer (CEO) of Future Group.
52. C TCS was the fifth-largest United States visa recipient in 2008
(after Infosys, CTS,Wipro and Mahindra Satyam). In 2012,
the Tata group companies, including TCS, were the second largest
recipient of H-1B visas. As of June 2014, TCS has over 300,000
employees. It is world's third largest IT employer behind IBM and HP
53. C
54. C
55. C Gross National Happiness is a term coined by His Majesty the Fourth
King of Bhutan, Jigme Singye Wangchuck in the 1970s. The concept
implies that sustainable development should take a holistic approach
towards notions of progress and give equal importance to non-
economic aspects of wellbeing.
56. C
57. C Its aim is to achieve major reform of the international trading system
through the introduction of lower trade barriers and revised trade rules.
The Doha Ministerial Declaration provided the mandate for the
negotiations, including on agriculture, services and an intellectual
property topic, which began earlier.
58. B There are 2 equinoxes every year – in September and March – when the
sun shines directly on the equator and the length of day and night is
nearly equal.
59. B
60. C Capital account convertibility (CAC) means the freedom to convert
local financial assets into foreign financial assets and vice versa at
market determined rates of exchange. This implies that Capital Account
Convertibility allows anyone to freely move from local currency into
foreign currency and back.
61 D The Kyoto Protocol is an international agreement linked to the United
Nations Framework Convention on Climate Change, which commits its
Parties by setting internationally binding emission reduction targets.
62 B A carbon credit is a financial instrument that allows the holder, usually
an energy company, to emit one ton of carbon dioxide. Credits are
awarded to countries or groups that have reduced their greenhouse
gases below their emission quota.
63 B Dumping, in reference to international trade, is the export by a country
or company of a product at a price that is lower in the foreign market
than the price charged in the domestic market.
64 B Habeas corpus is a writ that is used to bring a party who has been
criminally convicted in state court into federal court. Usually, writs

92
of habeas corpus are used to review the legality of the party's arrest,
imprisonment, or detention
65 C
66 A
67 C
68 C
69 B 1931-1932
70 B t is the largest and longest, spanning at 3,237,500 square kilometers, the
peninsula consists of the countries of
Yemen, Oman,Qatar, Bahrain, Kuwait, Saudi Arabia and the United
Arab Emirates as well as parts of southern Iraq and Jordan.
71 B
72 A The National Commission on Agriculture, Government of India, first
used the term 'social forestry' in 1976. It was then that India embarked
upon a social forestry project with the aim of taking the pressure off
the forests and making use of all unused and fallow land.
73 A
74 A
75 B the study of the physical features of the surface of the earth and their
relation to its geological structures.
76 B The midnight sun is a natural phenomenon. It reaches its peak during
the summer solstice, which occurs around June 21 each year. On the
day of the solstice, areas in Norway and in other regions around the
world north of the Arctic Circle may receive almost 24 hours of
sunlight.
77 C Guru Padmasambhava was known in Tibet as one of the founding
fathers of Tibetan Buddhism, who appeared in Tibet in 749 A.D, and
spent 54 years.
78 C The TRIPS Agreement has been in force since 1995 and is to date the
most comprehensive multilateral agreement on intellectual property.
79 B Carbon dating is a variety of radioactive dating which is applicable only
to matter which was once living and presumed to be in equilibrium with
the atmosphere, taking in carbon dioxide from the air for
photosynthesis.
80 C
81 B
82 A The Bali Road Map was adopted at the 13th Conference of the Parties
and the 3rd Meeting of the Parties in December 2007 in Bali. The Road
Map is a set of a forward-looking decisions that represent the work that
needs to be done under various negotiating “tracks” that is essential to
reaching a secure climate future.
83 A
84 B
85 B In the Gondi language of the tribals of Dantewada and Bastar, Salwa
Judum means peace march. But in effect, it involved authorities arming
tribal villagers to fight the Maoists.
86 B
87 A
88 A A free-trade area is the region encompassing a tradebloc whose member

93
countries have signed a free-trade agreement (FTA). Such agreements
involve cooperation between at least two countries to
reducetrade barriers – import quotas and tariffs – and to
increase trade of goods and services with each other.
89 B Affirmative Action: The System Of Reservations And Quotas In India.
Discrimination of a person or a community whether it is due to his/her
race, colour or nationality not only hampers the mobility of the
particular community but also leads to the degeneration of a nation
90 C A special economic zone (SEZ) refers to designated areas in countries
with special economicregulations that differ from other areas in the
same country. These regulations tend to contain measures that are
conducive to foreign direct investment.
91 C
92 C
93 B
94 C Biofuels have forced global food prices up by 75% - far more than
previously estimated - according to a confidential World Bank report
95 C “Evergreening,” is referred to the practice whereby pharmaceutical
firms extend thepatent life of a drug by obtaining additional 20-
year patents for minor reformulations or other iterations of the drug,
without necessarily increasing the therapeutic efficacy.
96 A The Poona Pact was the agreement between Mahatma Gandhi and Dr.
Br Ambedkar reached on 25 September 1932. The major points in this
pact were as follows: 148 seats were to be allotted to the depressed
classes in the provincial legislatures. This was more than double from
the 71 seats as promised in the Communal Award.
97 C
98 A
99 C In 2011: the top carbon dioxide (CO2) emitters were China, the United
States, the European Union, India, the Russian Federation, Japan, and
Canada. These data include CO2 emissions from fossil fuel combustion,
as well as cement manufacturing and gas flaring.
100 B
101 B General Agreement on Tariffs and Trade (GATT) was a multilateral
agreement regulating international trade.
102 C
103 C
104 B The ozone layer is important because it absorbs ultraviolet (UV)
radiation from the sun, preventing most of it from reaching the earth's
surface. Radiation in the UV spectrum has wavelengths just shorter
than those of visible light.
105 B
106 C A tsunami is a huge wave, usually caused by volcanic or earthquake
activity under the ocean, which can eventually crash onto the shoreline.
107 A Chipko Movement, started in 1970's, was a non
violentmovement aimed at protection and conservation of trees and
forests from being destroyed. The name of the Chipko moment
originated from the word 'embrace' as the villagers used to hug the trees
and protect them from wood cutters from cutting them.

94
108 A Mahapadma Nanda was the first great historical emperor of
Northern India.
109 B
110 A The East India Company gained a footing in India in 1612 after then
Mughal ruler Jahangir gave the Company rights to build a factory and
trading port in Surat
111 C Sir Muhammad Iqbal, also known as Allama Iqbal, was a philosopher,
poet and politician in British India who is widely regarded to have
inspired the Pakistan
112 C The Khilafat movement (1919-1924) was an agitation by Indian
Muslims allied with Indian nationalism in the years following World
War I. Its purpose was to pressure the British government to preserve
the authority of the Ottoman Sultan as Caliph of Islam following the
breakup of the Ottoman Empire at the end of the war
113 C He was the chairman of committee of All Parties Conference which
prepared the famous "Nehru Report". The Indian Constitution had its
genesis in this report and is heavily influenced by it. Most of its
recommendations like Bill of Rights, Constitutional division and
separation of Powers, Ideas of Secularism and Unified electorate,
federal form of Government etc form the cardinal features of Indian
Constitution and a number of them have been declared as basic features
of constitution by the Supreme Court of India.
114 B
115 D Malayalam poems, Odakhuzzal
116 A
117 D
118 B
119 B Maneka Gandhi v UOI
120 B Adverse on flora
121 C Directive Principles of State Policy (DPSP). Article 39A : Equal justice
and free legal aid.
122 B members were chosen by indirect election by the members of the
Provincial Legislative Assemblies, according to the scheme
recommended by the Cabinet Mission. The arrangement was: (i) 292
members were elected through the Provincial Legislative Assemblies;
(ii) 93 members represented the Indian Princely States; and (iii) 4
members represented the Chief Commissioners' Provinces.
123 B On 29 August, 1947, the Constituent Assembly set up a Drafting
Committee under the Chairmanship of Dr. B.R. Ambedkar to prepare a
Draft Constitution for India. While deliberating upon the draft
Constitution, the Assembly moved, discussed and disposed of as many
as 2,473 amendments out of a total of 7,635 tabled.
124 D
125 D
126 B
127 B The People
128 B Indra Sawhney v UOI
129 C It was removed as a Fundamental Freedom under 19(1)(f).
130 A

95
131 D Right to education is guaranteed within Article 21.
132 B It is a principle organ and is optional. It is only open to Member states.
133 C The Commission was set up 10 days after the demolition of Babri
Masjid and it took 17 long years and 48 extensions before it submitted
the report
134 A This act is not in force in India currently as it was repealed and was
replaced by Competition Act 2002 with effect from September 1, 2009.
The MRTP commission was replaced by Competition Commission of
India.
135 B The first-ever impeachment initiated against a judge-
V Ramaswami who retired as an SC judge in 1994
136 B The bulk of the Malimath Committee recommendations revolve around
the idea that whittling down the rights of the accused and increasing the
rate of convictions will help tackle crime.
137 B
138 D Lok Adalat is one of the alternative dispute redressal mechanisms, it is
a forum where disputes/cases pending in the court of law or at pre-
litigation stage are settled/ compromised amicably. Lok Adalats have
been given statutory status under the Legal Services Authorities Act,
1987. Please see: http://www.legalserviceindia.com/articles/lok_a.htm
139 B
140 B

2009
Question Answer Explanation
Number
41 C Capital markets include primary markets, where new stock and bond
issues are sold to investors, and secondary markets, which trade
existing securities.
42 None
43 B The Audacity of Hope is Barack Obama's call for a new kind of
politics—a politics that builds upon those shared understandings that
pull us together as Americans.
44 C
45 A A progressive tax is a tax that takes a larger percentage from high-
income earners than it does from low-income individuals.
46 A
47 C
48 None
49 A ISO 9000 is a series of standards, developed and published by the
International Organization for Standardization (ISO), that define,
establish, and maintain an effective quality assurance system for
manufacturing and service industries.
50 D Agmark is an acronym for agricultural marketing. Agmark is a quality
certification mark provided by the Government of India.
51 C The Secretariat of DOS and ISRO Headquarters are located
at Antariksh Bhavan in Bangalore.
52 A Saras is designed and manufactured jointly by National Aerospace

96
Laboratories and Hindustan Aeronautics Limited for the Indian Air
Force
53 A She was elected PM of Srilanka who retired at the age of 84.
54 C
55 B
56 B It is the folk dance of Gujarat.
57 B Promotes human rights, environmental sanitation, health and hygiene,
non-conventional sources of energy, waste management and social
reforms
58 B
59 A Liver is the largest gland in human body. It is also the largest (internal)
organ in our body and can weigh up to 1.5-1.6 kg for a human adult.
60 A
61 C Ashok Kumar, fondly known as Dadamoni, was one of the most
versatile and iconic actors of all times.
62 C
63 A
64 D
65 D
66 B
67 A
68 D The vision of a world-class South Asian University to be collectively
established ... functioning from Akbar Bhawan Campus in
Chanakyapuri, New Delhi before it
69 B
70 D Vitamin K is a fat-soluble vitamin that is most well known for the
important role it plays in blood clotting.
71 A Gilt-edged securities are a high-grade investment with very low risk.
Typically, these are issued by blue chip companies that dependably
meet dividend or interest payments because they are well-established
and financially stable .
72 C Now, Vijay Sharma
73 C Sania Mirza was conferred with Padma Shri award, becoming one of
the youngest sportspersons to receive the honour
74 D Elizabeth: The Golden Age is a 2007 sequel to the 1998 film Elizabeth,
directed by Shekhar Kapurand produced by Universal Pictures and
Working Title Films.
75 A
76 B Actually the train runs between New Delhi and Attari in India and
Wagah and Lahore in
Pakistan.
77 B The term "blue revolution" refers to the remarkable emergence of
aquaculture as an important and highly productive agricultural activity.
Aquaculture refers to all forms of active culturing of aquatic animals
and plants, occurring in marine, brackish, or fresh waters.
78 D
79 B Kanya Vidya Dhan Yojana is a scholarship scheme for meritorious girl
students of Uttar Pradesh government to promote higher studies among
girls.

97
80 C
81 B Among the given options largest amount of Savannah is in Africa,
therefore, Africa is the most likely correct response.
82 A
83 C Apsara is the oldest of India's research reactors. The reactor was
designed by the Bhabha Atomic Research Center (BARC) and built
with assistance from the United Kingdom. Apsara is a light water
swimming pool-type reactor with a maximum power output of one
megawatt thermal (MWt).
84 A
85 A Egypt was the first country to dig a man-made canal across its lands to
connect the Mediterranean Sea to the Red Sea via the branches of the
River Nile.
86 D
87 C
88 B IPCC actually expands to Intergovernmental Panel on Climate Change
.
89 A
90 C

2010
Question Answer Explanation
number
41. C Lumbinī is a Buddhist pilgrimage site in the Rupandehi District of
Nepal. It is the place where, according to Buddhist tradition, Queen
Mayadevi gave birth to Siddhartha Gautama in 623 BCE. Gautama,
who achieved Enlightenment some time around 588 BCE became the
Gautama Buddha and founded Buddhism. Lumbini is one of many
magnets for pilgrimage that sprang up in places pivotal to the life of
Gautama Buddha; other notable pilgrimage sites include Kushinagar,
Bodh Gaya and Sarnath.
42. B
43. A Vishakhadatta was an Indian Sanskrit poet and playwright. Although
Vishakhadatta furnishes the names of his father and grandfather as
Maharaja Bhaskaradatta and Maharaja Vateshvaradatta in his political
drama Mudrārākṣasa, we know little else about him. only two of his
plays, the Mudrārākṣasa and the Devichandraguptam are known to us.
44. C The Asian Games, also known as Asiad, is a Pancontinental multi-sport
event held every four years among athletes from all over Asia. The
Games were regulated by the Asian Games Federation (AGF) from the
first Games in New Delhi, India, until the 1978 Games. Since the 1982
Games they have been organized by the Olympic Council of Asia
(OCA), after the breakup of the Asian Games Federation. The Games
are recognized by the International Olympic Committee (IOC) and are

98
described as the second largest multi-sport event after the Olympic
Games.
45. B
46. A
47. D Astra is an active radar homing beyond-visual-range air-to-air missile
(BVRAAM) developed by the Defence Research and Development
Organisation (DRDO), India. Astra is designed to be capable of
engaging targets at varying range and altitudes allowing for
engagement of both short-range targets (up to 20 km) and long-range
targets (up to 80 km) using alternative propulsion modes except for a
failure in one test, the missile has successfully completed all its tests.
The missile was last tested on 18 March 2015 from a Su-30MKI fighter
against a simulated live target. Astra uses a smokeless propulsion
system.
48. D The Nehru Institute of Mountaineering (NIM) is a mountaineering
school in Uttarkashi, India, established on 14 November 1965
49. C
50. D The Liberation Tigers of Tamil Eelam commonly known as the LTTE
or the Tamil Tigers) is a now-defunct militant organisation that was
based in northern Sri Lanka. Founded in May 1976 by Velupillai
Prabhakaran, it waged a secessionist nationalist insurgency to create a
monoethnic, independent state in the north and east of Sri Lanka for
Tamil people. This campaign led to the Sri Lankan Civil War, which
ran from 1983 until 2009, when the LTTE was defeated by the Sri
Lankan Military under the leadership of President Mahinda Rajapaksa.
51. B Dilip Vengsarkar who earned the sobriquet ‘colonel’ has been chosen
for the BCCI’s Col. C.K. Nayudu Lifetime Achievement Award for
2013-14.
52. A The European Union is a politico-economic union of 28 member states
that are located primarily in Europe. The EU operates through a system
of supranational institutions and intergovernmental-negotiated
decisions by the member states.
53. B Premchand better known as Munshi Premchand, Munshi being an
honorary prefix, was an Indian writer famous for his modern
Hindustani literature. He is one of the most celebrated writers of the
Indian subcontinent, and is regarded as one of the foremost Hindustani
writers of the early twentieth century. Born Dhanpat Rai Srivastav, he
began writing under the pen name "Nawab Rai", but subsequently
switched to "Premchand". A novel writer, story writer and dramatist, he
has been referred to as the "Upanyas Samrat" ("Emperor among
Novelists") by some Hindi writers. His works include more than a

99
dozen novels, around 250 short stories, several essays and translations
of a number of foreign literary works into Hindi.
54. C
55. B The Storming of the Bastille occurred in Paris, France, on the morning
of 14 July 1789. The medieval fortress and prison in Paris known as the
Bastille represented royal authority in the centre of Paris. The prison
contained just seven inmates at the time of its storming but was a
symbol of the abuses of the monarchy: its fall was the flashpoint of the
French Revolution.
56. B Peru officially the Republic of Peru is a country in western South
America. It is bordered in the north by Ecuador and Colombia, in the
east by Brazil, in the southeast by Bolivia, in the south by Chile, and in
the west by the Pacific Ocean. Peru is an extremely biodiverse country
with habitats ranging from the arid plains of the Pacific coastal region
in the west to the peaks of the Andes mountains vertically extending
from the north to the southeast of the country to the tropical Amazon
Basin rainforest in the east with the Amazon river.
57. C The Ganges is a trans-boundary river of Asia which flows through the
nations of India and Bangladesh. The 2,525 km (1,569 mi) river rises in
the western Himalayas in the Indian state of Uttarakhand, and flows
south and east through the Gangetic Plain of North India into
Bangladesh, where it empties into the Bay of Bengal. It is the third
largest river by discharge.
58. A
59. D The Maldives is a sovereign island country and archipelago in the
Indian Ocean. It is located southwest of India and Sri Lanka in the
Laccadive Sea. The chain of twenty six atolls stretches from
Ihavandhippolhu Atoll to the Addu Atoll. The capital and largest city is
Malé, traditionally called the "King's Island".
60. D The United Liberation Front of Assam is the major independentist outfit
operating in Assam, North East India. It seeks to establish a sovereign
Assam via an armed struggle in the Assam conflict. The government of
India banned the organisation in 1990 citing it as a terrorist
organization, while the United States Department of State lists it under
"other groups of concern.
61. C The Galápagos Islands are an archipelago of volcanic islands
distributed on either side of the Equator in the Pacific Ocean, 906 km
(563 mi) west of continental Ecuador, of which they are a part.
62. C
63. C

100
64. B
65. C Zimbabwe is a landlocked country in southern Africa known for its
dramatic landscape and diverse wildlife, much of it within parks,
reserves and safari areas. On the Zambezi River, Victoria Falls make a
thundering 108m drop into narrow Batoka Gorge, where there’s white-
water rafting and bungee-jumping. Downstream are Matusadona and
Mana Pools national parks, home to hippos, rhinos and birdlife.
66. B
67. D
68. C
69. B
70. A
71. B Bal Gangadhar Tilak (23 July 1856 – 1 August 1920), born as Keshav
Gangadhar Tilak, was an Indian nationalist, journalist, teacher, social
reformer, lawyer and an independence activist. He was the first leader
of the Indian Independence Movement. The British colonial authorities
called him "Father of the Indian unrest." He was also conferred with the
honorary title of "Lokmanya", which literally means "accepted by the
people (as their leader)".
72. C Nur-ud-din Mohammad Salim, known by his imperial name Jahangir
(30 August 1569 – 7 November 1627), was the fourth Mughal Emperor
who ruled from 1605 until his death in 1627. He is considered to be one
of the greatest Indian Emperors and the fourth of the Grand Mughals in
Indian historiography. Much romance has gathered around his name,
and the tale of his illicit relationship with the Mughal courtesan,
Anarkali, has been widely adapted into the literature, art and cinema of
India.
73. C
74. B Abu'l-Fath Jalal ud-din Muhammad Akbar, popularly known as Akbar I
was Mughal Emperor from 1556 until his death. He was the third and
one of the greatest rulers of the Mughal Dynasty in India.
75. B
76. D Azadirachta indica, also known as Neem, is a tree in the mahogany
family Meliaceae. It is one of two species in the genus Azadirachta, and
is native to India and the Indian subcontinent including Nepal, Pakistan,
Bangladesh and Sri Lanka. It is typically grown in tropical and semi-
tropical regions. Neem trees now also grow in islands located in the
southern part of Iran. Its fruits and seeds are the source of neem oil.

101
77. D
78. D Björn Borg is a former world No. 1 tennis player from Sweden widely
considered to be one of the greatest in tennis history. Between 1974 and
1981 he became the first male professional to win 11 Grand Slam
singles titles: six at the French Open and five consecutive at
Wimbledon. He also won three year-end championships and 15 Grand
Prix Super Series titles.
79. A Vande Mataram literally, "I praise thee, Mother"—is a poem from
Bankim Chandra Chatterjee's 1882 novel Anandamath. It was written in
Bengali and Sanskrit.
80. C Turkey is a parliamentary republic in Eurasia, largely located in
Western Asia, with the smaller portion of Eastern Thrace in Southeast
Europe.
81. A
82. B
83. D Ernest Miller Hemingway (July 21, 1899 – July 2, 1961) was an
American novelist, short story writer, and journalist. His economical
and understated style had a strong influence on 20th-century fiction,
while his life of adventure and his public image influenced later
generations. Hemingway produced most of his work between the mid-
1920s and the mid-1950s, and won the Nobel Prize in Literature in
1954.
84. A
85. A World War I (WWI or WW1), also known as the First World War or
the Great War, was a global war centered in Europe that began on 28
July 1914 and lasted until 11 November 1918. More than 9 million
combatants and 7 million civilians died as a result of the war, a casualty
rate exacerbated by the belligerents' technological and industrial
sophistication, and tactical stalemate. It was one of the deadliest
conflicts in history, paving the way for major political changes,
including revolutions in many of the nations involved
86. A
87. C Periyar National Park and Wildlife Sanctuary is a protected area in the
districts of Idukki and Pathanamthitta in Kerala, India. It is notable as
an elephant reserve and a tiger reserve.
88. D
89. B Yamīn-ud-Dawla Abul-Qāṣim Maḥmūd ibn Sebüktegīn, more
commonly known as Mahmud of Ghazni, also known as Mahmūd-i
Zābulī, was the most prominent ruler of the Ghaznavid Empire

102
90. C The Battle of Trafalgar was a naval engagement fought by the Royal
Navy against the combined fleets of the French and Spanish Navies,
during the War of the Third Coalition of the Napoleonic Wars

2011
Question Answer Explanation
number
41. (b)
42. (a) The Man Booker Prize for Fiction (formerly known as the Booker-
McConnell Prize and commonly known simply as the Booker Prize)
is a literary prize awarded each year for the best original novel,
written in the English language, and published in the UK. The
winner of the Man Booker Prize is generally assured of international
renown and success; therefore, the prize is of great significance for
the book trade.
43. (a)
44. (c) The Nagoya Protocol on Access to Genetic Resources and the Fair
and Equitable Sharing of Benefits Arising from their Utilization to
the Convention on Biological Diversity is an international agreement
which aims at sharing the benefits arising from the utilization of
genetic resources in a fair and equitable way.
45. (a)
46. (b)
47. (b) The Tunisian Revolution, also known as the Jasmine Revolution,
was an intensive campaign of civil resistance, including a series of
street demonstrations taking place in Tunisia. The events began on
18 December 2010 and led to the ousting of longtime President Zine
El Abidine Ben Ali in January 2011. It eventually led to a thorough
democratization of the country and to free and democratic elections.
They saw the victory of a coalition of the Islamist Ennahda
Movement with the centre-left Congress for the Republic and the
left-leaning Ettakatol as junior partners.
48. (a) Ferit Orhan Pamuk (generally known simply as Orhan Pamuk; born
7 June 1952) is a Turkish novelist, screenwriter, academic and
recipient of the 2006 Nobel Prize in Literature. One of Turkey's most
prominent novelists, his work has sold over eleven million books in
sixty languages making him the country's best-selling writer
49. (a)
50. (a)

103
51. (b)
52. (c) Shivaji Bhonsle, also known as Chhatrapati Shivaji Maharaj, was an
Indian warrior king and a member of the Bhonsle Maratha clan.
53. (a)
54. (a)
55. (d) National Innovation Council is the think-tank council of India to
discuss, analyse and help implement strategies for innovation in
India and suggest a Roadmap for Innovation 2010-2020.
56. (d)
57. (d) Armed Forces (Special Powers) Acts (AFSPA), are Acts of the
Parliament of India that grant special powers to the Indian Armed
Forces in what each act terms "disturbed areas"
58. (c)
59. (d)
60. (d)
61. (b)
62. (d)
63. (a)
64. (d)
65. (b) The Justice P.C. Phukan Commission of Inquiry probing the clashes
between Bodos and Muslims in northern Assam's Udalguri district.
66. (a)
67. (b)
68. (b)
69. (d)
70. (d)
71. (a)
72. (a)
73. (d)
74. (c)
75. (c)

104
76. (d)
77. (a)
78. (b)
79. (b)
80. (b)
81. (c)
82. (b)
83. (b)
84. (d)
85. (c)
86. (c)
87. (c) The United Nations Framework Convention on Climate Change
(UNFCCC) is an international environmental treaty (currently the
only international climate policy venue with broad legitimacy, due in
part to its virtually universal membership) negotiated at the United
Nations Conference on Environment and Development (UNCED).
88. (a)
89. (c)
90. (c)

2012
Question Answer Explanation
number
41. (b) Aristotle was a Greek philosopher and scientist born in the
Macedonian city of Stagira, Chalkidice, on the northern periphery of
Classical Greece.
42. (c)
43. (a) Jean-Jacques Rousseau was a philosopher, writer, and composer of
the 18th century. His political philosophy influenced the
Enlightenment in France and across Europe, as well as aspects of the
French Revolution and the overall development of modern political
and educational thought.
44. (d)
45. (b)

105
46. (c) Shabbir Ali, is an Indian association football manager and former
player. He was awarded the Dhyan Chand Award, the highest award
in Indian sports for lifetime achievement, given by Government of
India in 2011.
47. (b)
48. (a)
49. (c) Tomas Gösta Tranströmer was a Swedish poet, psychologist and
translator. His poems captured the long Swedish winters, the rhythm
of the seasons and the palpable, atmospheric beauty of nature.
50. (b) The UNESCO King Sejong Literacy Prize was created by the
Government of the Republic of Korea in 1989. The Prize honours the
outstanding contribution made to literacy by Sejong the Great (1397
- 1418/1450) who created the Korean alphabet Hangul.
51. (d)
52. (b)
53. (a)
54. (b)
55. (c)
56. (a) Vijay Amritraj is a former tennis player from India, sports
commentator and actor. He was awarded the Padma Shri, India's
fourth highest civilian honor in 1983.
57. (c)
58. (b)
59. (d)
60. (c)
61. (a)
62. (c)
63. (c)
64. (a)
65. (b)
66. (d)
67. (b) The Indira Gandhi Canal is one of the largest canal projects in India.
It starts from the Harike Barrage at Firozpur, a few kilometers below
the confluence of the Sutlej and Beas rivers in the Indian state of

106
Punjab and terminates in irrigation facilities in the Thar Desert in the
north west of Rajasthan state.
68. (c)
69. (b)
70. (c)
71. (a)
72. (c)
73. (c)
74. (c)
75. (c)
76. (a)
77. (c)
78. (a)
79. (b)
80. (a)
81. (c)
82. (a)
83. (b)
84. (a)
85. (a)
86. (b)
87. (d)
88. (b)
89. (b)
90. (c)

2013
Question Answer Explanation
number
61. (d) The European Union (EU) is a politico-economic union of 28
member states that are located primarily in Europe. The EU operates

107
through a system of supranational institutions and
intergovernmental-negotiated decisions by the member states. The
institutions are: the European Parliament, the European Council, the
Council of the European Union, the European Commission, the
Court of Justice of the European Union, the European Central Bank,
and the Court of Auditors. The European Parliament is elected every
five years by EU citizens
62. (b) Chandrayaan-1 was India's first lunar probe. It was launched by the
Indian Space Research Organisation in October 2008, and operated
until August 2009. The mission included a lunar orbiter and an
impactor. India launched the spacecraft using a PSLV-XL rocket,
serial number C11 on 22 October 2008 from Satish Dhawan Space
Centre, Sriharikota, Nellore District, Andhra Pradesh, about 80 km
north of Chennai, at 06:22 IST (00:52 UTC). Prime minister Atal
Bihari Vajpayee announced the project on course in his
Independence Day speech on 15 August 2003. The mission was a
major boost to India's space program, as India researched and
developed its own technology in order to explore the Moon. The
vehicle was successfully inserted into lunar orbit on 8 November
2008
63. (a) Viswanathan "Vishy" Anand (born 11 December 1969) is an Indian
chess Grandmaster and former World Chess Champion. He held the
FIDE World Chess Championship from 2000 to 2002, at a time
when the world title was split. He became the undisputed World
Champion in 2007 and defended his title against Vladimir Kramnik
in 2008. He then defended his title in the World Chess
Championship 2010 against Veselin Topalov and in the World Chess
Championship 2012 against Boris Gelfand. In the World Chess
Championship 2013 he lost to challenger Magnus Carlsen and he lost
again to Carlsen in the World Chess Championship 2014.
64. (b) Kapilavastu formerly Taulihawa, is a municipality and
administrative center of Kapilvastu District in the Lumbini Zone of
southern Nepal. It is located roughly 25 kilometres (16 mi) to the
northwest of Lumbini, a UNESCO World Heritage Site that is
widely believed to be the birthplace of Gautama Buddha.
65. (a) The Green Revolution refers to a series of research and development
and technology transfer initiatives, occurring between the 1930s and
the late 1960s (with prequels in the work of the agrarian genetist
Nazareno Strampelli in the 1920s and 1930s), that increased
agricultural production worldwide, particularly in the developing
world, beginning most markedly in the late 1960s. The initiatives,
led by Norman Borlaug, the "Father of the Green Revolution," who
won the Nobel Peace Prize in 1970, credited with saving over a
billion people from starvation, involved the development of high-

108
yielding varieties of cereal grains, expansion of irrigation
infrastructure, modernization of management techniques, distribution
of hybridized seeds, synthetic fertilizers, and pesticides to farmers.
66. (d) Bluetooth is a wireless technology standard for exchanging data over
short distances (using short-wavelength UHF radio waves in the ISM
band from 2.4 to 2.485 GHz from fixed and mobile devices, and
building personal area networks (PANs). Invented by telecom vendor
Ericsson in 1994, it was originally conceived as a wireless alternative
to RS-232 data cables. It can connect several devices, overcoming
problems of synchronization.
67. (a)
68. (b) Baglihar Dam, also known as Baglihar Hydroelectric Power Project,
is a run-of-the-river power project on the Chenab River in the
southern Doda district of the Indian controlled state of Jammu and
Kashmir
69. (a) Navajivan Trust is a publishing house based out of Ahmedabad,
India. It was founded by Mahatma Gandhi in 1929 and has published
more than 800 titles in English, Gujarati, Hindi and other languages
to date. Earlier, Navajivan referred to a weekly newspaper published
by Gandhi, in Gujarati, from 1919 (September 7) to 1931, from
Ahmedabad.
70. (b) Brazil, a vast South American country, stretches from the Amazon
Basin in the north to vineyards and massive Iguaçu Falls in the south.
Rio de Janeiro, symbolized by its 38m Christ the Redeemer statue
atop Mt. Corcovado, is famed for its busy Copacabana and Ipanema
beaches as well as its enormous, raucous Carnival festival, featuring
parade floats, flamboyant costumes and samba.
71. (d) Fiscal deficit is the difference between the government's
expenditures and its revenues (excluding the money it's borrowed). A
country's fiscal deficit is usually communicated as a percentage of its
gross domestic product (GDP).
72. (c) Odisha (formerly Orissa), an eastern Indian state on the Bay of
Bengal, is known for its tribal cultures and its many ancient Hindu
temples. The capital, Bhubaneswar, is home to hundreds of temples,
notably the Nagara-style Mukteswar and the Lingaraj, both built in
the 11th century and set around sacred Bindusagar Lake. The Odisha
State Museum is dedicated to the area’s indigenous history.
73. (b) Bharat Ratna is the highest civilian award of the Republic of India.
Instituted on 2 January 1954, the award is conferred "in recognition
of exceptional service/performance of the highest order", without
distinction of race, occupation, position, or sex. The award was
originally limited to achievements in the arts, literature, science, and

109
public services but the government expanded the criteria to include
"any field of human endeavour" in December 2011.
Recommendations for the Bharat Ratna are made by the Prime
Minister to the President, with a maximum of three nominees being
awarded per year. Recipients receive a Sanad (certificate) signed by
the President and a peepal-leaf–shaped medallion; there is no
monetary grant associated with the award. Bharat Ratna recipients
rank seventh in the Indian order of precedence, but are
constitutionally prohibited from using the award name as a title.
Padma Awards were instituted in the year 1954. Except for brief
interruption(s) during the years 1977 and 1980 and 1993 to 1997,
these awards have been announced every year on Republic Day. The
award is given in three categories, namely, Padma Vibhushan,
Padma Bhushan and Padma Shri.
74. (b)
75. (c)
76. (b)
77. (c)
78. (d)
79. (c) Argentina is a massive South American nation with terrain
encompassing Andes mountains, glacial lakes and Pampas grassland,
the traditional grazing ground of its famed beef cattle. The country is
known for tango, steak and football. Its big, cosmopolitan capital,
Buenos Aires, is centered on the 16th-century Plaza de Mayo, lined
with stately buildings including Casa Rosada, the iconic, balconied
presidential palace.
80. (b)
81. (b)
82. (a) Kumbha is a mass Hindu pilgrimage of faith in which Hindus gather
to bathe in a sacred river. It is considered to be the largest peaceful
gathering in the world where around 100 million people were
expected to visit during the Maha Kumbh Mela in 2013 in
Allahabad. It is held every third year at one of the four places by
rotation: Haridwar, Allahabad (Prayaga), Nashik and Ujjain. Thus
the Kumbh Mela is held at each of these four places every twelfth
year. Ardha ("Half") Kumbh Mela is held at only two places,
Haridwar and Allahabad, every sixth year. The rivers at these four
places are: the Ganges (Ganga) at Haridwar, the confluence
(Sangam) of the Ganges and the Yamuna and the mythical Saraswati
at Allahabad, the Godawari at Nashik, and the Shipra at Ujjain. The
name Kumbh Mela comes from Hindi, and in the original Sanskrit

110
and other Indian languages it is more often known as Kumbha Mela.
Kumbha means a pitcher and Mela means fair in Sanskrit.
83. (b)
84. (d) Since 1947, the Indian economy has been premised on the concept of
planning. This has been carried through the Five-Year Plans,
developed, executed, and monitored by the Planning Commission.
With the Prime Minister as the ex-officio Chairman, the commission
has a nominated Deputy Chairman, who holds the rank of a Cabinet
Minister. Montek Singh Ahluwalia is the last Deputy Chairman of
the Commission (resigned on 26 May 2014). The Eleventh Plan
completed its term in March 2012 and the Twelfth Plan is currently
underway. Prior to the Fourth Plan, the allocation of state resources
was based on schematic patterns rather than a transparent and
objective mechanism, which led to the adoption of the Gadgil
formula in 1969. Revised versions of the formula have been used
since then to determine the allocation of central assistance for state
plans.
85. (b) The gender ratio is the ratio of males to females in a population. In
the majority of species, this is 1:1, the reasons for which are
described in Fisher's principle. Some eusocial wasps, such as the
Polistes fuscatus and the Polistes exclamans, seem to defy this ratio
at times.
86. (d) Venezuela is a country on the northern coast of South America with
diverse natural attractions. Along its Caribbean coast are tropical
resort islands including Isla de Margarita and the Los Roques
archipelago. To the northwest are the Andes Mountains and the
colonial town of Mérida, a base for hiking Sierra Nevada National
Park. Caracas, the capital, is to the north.
87. (c)
88. (c)
89. (d)
90. (d) Ali Gauhar (25 June 1728 – 19 November 1806), historically known
as Shah Alam II, was the eighteenth Mughal Emperor and the son of
Alamgir II. Shah Alam II became the emperor of a crumbling
Mughal empire; his power was so depleted during his reign that it led
to a saying in Persian, Sultanat-e-Shah Alam, Az Dilli ta Palam,
meaning, 'The kingdom of Shah Alam is from Delhi to Palam',
Palam being a suburb of Delhi.
91. (b) Andhra Pradesh is a state bordering India’s southeastern coast.
Tropical forests, rivers, hills and caves make it a popular ecotourism
destination. Beaches line the Bay of Bengal, offering spots for

111
swimming and surfing. Major cultural landmarks include Tirumala
Venkateswara Temple, an ornate hilltop shrine to Hindu’s Vishnu, in
the southern part of the state. It’s visited by tens of millions of
pilgrims annually.
92. (b)
93. (d)
94. (a) John Milton (9 December 1608 – 8 November 1674) was an English
poet, polemicist, man of letters, and a civil servant for the
Commonwealth of England under Oliver Cromwell. He wrote at a
time of religious flux and political upheaval, and is best known for
his epic poem Paradise Lost (1667), written in blank verse.
Milton's poetry and prose reflect deep personal convictions, a
passion for freedom and self-determination, and the urgent issues
and political turbulence of his day. Writing in English, Latin, Greek,
and Italian, he achieved international renown within his lifetime, and
his celebrated Areopagitica (1644)—written in condemnation of pre-
publication censorship—is among history's most influential and
impassioned defences of free speech and freedom of the press.
95. (b) Padmanabhaswamy temple is located in Thiruvananthapuram,
Kerala, India . The temple is built in Dravidian style of architecture
associated with the temples located in the neighboring state of Tamil
Nadu, featuring high walls and a 16th-century Gopuram. While the
Moolasthanam of the temple is the Ananthapuram Temple in
Kasargod, architecturally to some extent, the temple is a replica of
the Adikesava Perumal temple located in Kanyakumari District. It is
the richest Hindu temple in the world. In fact, in terms of gold and
precious stones, it is by far the wealthiest institution and place of
worship of any kind in the recorded history of the world
96. (a) The International Red Cross and Red Crescent Movement is an
international humanitarian movement with approximately 97 million
volunteers, members and staff worldwide[2] which was founded to
protect human life and health, to ensure respect for all human beings,
and to prevent and alleviate human suffering.
97. (c)
98. (b)
99. (c)
100. (a)
101. (b)
102. (d)

112
103. (c) The Stone Age is a broad prehistoric period during which stone was
widely used to make implements with a sharp edge, a point, or a
percussion surface. The period lasted roughly 3.4 million years, and
ended between 6000 BCE and 2000 BCE with the advent of
metalworking. Stone Age artifacts include tools used by modern
humans and by their predecessor species in the genus Homo, as well
as the earlier partly contemporaneous genera Australopithecus and
Paranthropus. Bone tools were used during this period as well but are
rarely preserved in the archaeological record. The Stone Age is
further subdivided by the types of stone tools in use
104. (b)
105. (d) Jivatram Bhagwandas Kripalani (11 November 1888 – 19 March
1982), popularly known as Acharya Kripalani, was an Indian
politician, noted particularly for holding the presidency of the Indian
National Congress during the transfer of power in 1947. During the
election for the post of the future Prime Minister of India held by the
Congress party, he had the second highest number of votes after
Sardar Patel. However, on Gandhi's insistence, both Patel and
Kripalani backed out to allow Jawaharlal Nehru to become the first
Prime Minister of India.
106. (b) Environment Protection Act, 1986 is an Act of the Parliament of
India. In the wake of the Bhopal Tragedy, the Government of India
enacted the Environment Protection Act of 1986 under Article 253 of
the Constitution. Passed in March 1986, it came into force on 19th
November 1986. The purpose of the Act is to implement the
decisions of the United Nations Conference on the Human
Environments they relate to the protection and improvement of the
human environment and the prevention of hazards to human beings,
other living creatures, plants and property. The Act is an “umbrella”
legislation designed to provide a framework for central government
coordination of the activities of various central and state authorities
established under previous laws, such as the Water Act and the Air
Act.
107. (a)
108. (c) Maues had his capital in Sirkap and minted most of his coins in
Taxila. Maues did not manage however to conquer the Punjab
territories of the Indo-Greeks east of the Jhelum, which remained
under Greek control. After his death the Indo-Greeks regained most
of their territory.
109. (a)
110. (a) The Non-Aligned Movement (NAM) is a group of states which are
not formally aligned with or against any major power bloc. As of

113
2012, the movement has 120 members and 17 observer countries.
The organization was founded in Belgrade in 1961, and was largely
conceived by India's first prime minister, Jawaharlal Nehru;
Indonesia's first president, Sukarno; Egypt's second president, Gamal
Abdel Nasser; Ghana's first president Kwame Nkrumah; and
Yugoslavia's president, Josip Broz Tito. All five leaders were
prominent advocates of a middle course for states in the Developing
World between the Western and Eastern Blocs in the Cold War. The
phrase itself was first used to represent the doctrine by Indian
diplomat V. K. Krishna Menon in 1953, at the United Nations.

2014
Question Answer Explanation
number
61. (c)
62. (d)
63. (b) The Defence Research and Development Organisation (DRDO) is
an agency of the Republic of India, charged with the military's
research and development, headquartered in New Delhi, India. It
was formed in 1958 by the merger of the Technical Development
Establishment and the Directorate of Technical Development and
Production with the Defence Science Organisation. It is under the
administrative control of the Ministry of Defence, Government of
India.
64. (b)
65. (d)
66. (d) The Fundamental Rights, Directive Principles of State Policy and
Fundamental Duties are sections of the Constitution of India that
prescribe the fundamental obligations of the State to its citizens and
the duties of the citizens to the State. These sections comprise a
constitutional bill of rights for government policy-making and the
behaviour and conduct of citizens. These sections are considered
vital elements of the constitution, which was developed between
1947 and 1949 by the Constituent Assembly of India.
67. (c) The Kharoṣṭhī script is an ancient script used in ancient Gandhara
(primarily modern-day Afghanistan and Pakistan) to write the
Gandhari Prakrit and Sanskrit. An abugida, it was in use from the
middle of the 3rd century BCE until it died out in its homeland
around the 3rd century CE. It was also in use in Bactria, the Kushan
Empire, Sogdia and along the Silk Road, where there is some
evidence it may have survived until the 7th century in the remote

114
way stations of Khotan and Niya.
68. (b) Nur-ud-din Mohammad Salim, known by his imperial name
Jahangir "conqueror of the world" (30 August 1569 – 7 November
1627), was the fourth Mughal Emperor who ruled from 1605 until
his death in 1627. He is considered to be one of the greatest Indian
Emperors and the fourth of the Grand Mughals in Indian
historiography. Much romance has gathered around his name, and
the tale of his illicit relationship with the Mughal courtesan,
Anarkali, has been widely adapted into the literature, art and
cinema of India.
69. (a)
70. (d) Devaki Nandan Khatri (June 18, 1861 – 1913) was an Indian
writer, who belonged to the first generation of popular novelists in
the modern Hindi language. Also known as Babu Devakinandan
Khatri, he was the first author of mystery novels in Hindi.
Chandrakanta is the most popular of his works.
Premchand better known as Munshi Premchand, Munshi being an
honorary prefix, was an Indian writer famous for his modern
Hindustani literature. He is one of the most celebrated writers of the
Indian subcontinent, and is regarded as one of the foremost
Hindustani writers of the early twentieth century.
Rishi Bankim Chandra Chattopadhyay (26 June 1838[1] – 8 April
1894) was a Bengali writer, poet and journalist. He was the
composer of India's national song Vande Mataram, originally a
Bengali and Sanskrit stotra personifying India as a mother goddess
and inspiring the activists during the Indian Independence
Movement. Chattopadhyay wrote thirteen novels and several
'serious, serio-comic, satirical, scientific and critical treaties' in
Bengali. His works were widely translated into other regional
languages of India as well as in English.
71. (d)
72. (c)
73. (b)
74. (a) Civil disobedience is the active, professed refusal to obey certain
laws, demands, and commands of a government, or of an
occupying international power. Civil disobedience is a symbolic or
ritualistic violation of the law, rather than a rejection of the system
as a whole. Civil disobedience is sometimes, though not always,
defined as being nonviolent resistance.
75. (c)

115
76. (c) Jupiter is the fifth planet from the Sun and the largest in the Solar
System. It is a giant planet with a mass one-thousandth that of the
Sun, but two and a half times that of all the other planets in the
Solar System combined.
77. (d)
78. (d)
79. (c)
80. (b) The Dadasaheb Phalke Award is India's highest award in cinema. It
is presented annually at the National Film Awards ceremony by the
Directorate of Film Festivals, an organisation set up by the Ministry
of Information and Broadcasting.
81. (c)
82. (d)
83. (a)
84. (a)
85. (c) The Finance Commission of India came into existence in 1951. It
was established under Article 280 of the Indian Constitution by the
President of India. It was formed to define the financial relations
between the centre and the state.
86. (d)
87. (c)
88. (d) The Harrod–Domar model is an early post-Keynesian model of
economic growth. It is used in development economics to explain
an economy's growth rate in terms of the level of saving and
productivity of capital.
89. (d)
90. (c)
91. (d) Hickey's Bengal Gazette was an English newspaper published from
Kolkata (then Calcutta), India. It was the first major newspaper in
India, started in 1780. It was published for two years. Founded by
James Augustus Hicky, a highly eccentric Irishman who had
previously spent two years in Jail for debt.
92. (a)
93. (c)
94. (c) Raghuram Govind Rajan (born 3 February 1963) is the current and
the 23rd Governor of the Reserve Bank of India, having taken

116
charge of India's central banking institution on 4 September 2013,
and succeeding Duvvuri Subbarao. Rajan was chief economic
adviser to India's Ministry of Finance during the previous year and
chief economist at the International Monetary Fund from 2003 to
2007. He is on leave of absence as a professor of finance at the
graduate business school at the University of Chicago.
95. (b)
96. (c)
97. (c)
98. (d) The Arjuna Awards are given by the Ministry of Youth Affairs and
Sports, government of India to recognize outstanding achievement
in National sports. Instituted in 1961, the award carries a cash prize
of ₹ 500,000, a bronze statue of Arjuna and a scroll.
99. (a)
100. (b)
101. (b)
102. (d) Hydrogen is a chemical element with chemical symbol H and
atomic number 1. With an atomic weight of 1.00794 u, hydrogen is
the lightest element on the periodic table
103. (c) Gregor Johann Mendel was scientist and Augustinian friar who
gained posthumous fame as the founder of the modern science of
genetics. Though farmers had known for centuries that
crossbreeding of animals and plants could favor certain desirable
traits, Mendel's pea plant experiments conducted between 1856 and
1863 established many of the rules of heredity, now referred to as
the laws of Mendelian inheritance.
104. (b)
105. (a)
106. (c)
107. (c)
108. (d)
109. (a)

2015
Question Answer Explanations
number

117
1. B
2. B Anil Kumble is a former international cricketer and former captain of
the Indian cricket team. A right-arm leg spin (leg break googly)
bowler, he took 619 wickets in Test cricket and remains the third-
highest wicket taker—only behind Muttiah Muralitharan and Shane
Warne—as of 2015. Unlike his contemporaries, Kumble was not a
big turner of the ball, rather relied much on pace and accuracy. His
ability to make the ball bounce with subtle variations in pace made
him a tough bowler to face for the batsmen; thus earning him the
sobriquet "Jumbo". Kumble was selected as the Indian Cricket
Cricketer of the Year in 1993 and one of the Wisden Cricketers of
the Year three years later.
3. A The Indian Councils Act 1909 commonly known as the Morley-
Minto Reforms [or as the Minto-Morley Reforms], was an Act of the
Parliament of the United Kingdom that brought about a limited
increase in the involvement of Indians in the governance of British
India.
4. A
5. C
6. A
7. A Astra is an active radar homing beyond-visual-range air-to-air
missile (BVRAAM) developed by the Defence Research and
Development Organisation (DRDO), India. Astra is designed to be
capable of engaging targets at varying range and altitudes allowing
for engagement of both short-range targets (up to 20 km) and long-
range targets (up to 80 km) using alternative propulsion modes.[4][5]
Except for a failure in one test, the missile has successfully
completed all its tests. The missile was last tested on 18 March 2015
from a Su-30MKI fighter against a simulated live target.[6] Astra
uses a smokeless propulsion system.
8. C The Indian Space Research Organization has developed a Flood
Hazard Atlas by mapping flood prone and vulnerable areas in
Assam. As per report all flood prone areas have been divided into 3
categories and accordingly district administrations will prepare
action plan on relief and evacuation in advance. Lakhs of people are
rendered homeless in Assam due to flood almost every year.
9. C
10. A
11. A

118
12. B
13. B
14. B
15. B The South Eastern Railway is one of the seventeen railway zones in
India. It is headquartered at Garden Reach, Kolkata, West Bengal,
India.
16. A
17. A Khan was the Chairman of the RBI Internal Group on Rural Credit
and Microfinance, which is popularly known as the Khan
Committee. Based on the recommendations of the Khan Committee,
RBI prepared guidelines for expansion of the banking outreach
through the Business Facilitator and Business Correspondent models
with ICT support, which revolutionized the spread of financial
inclusion in India.
Subrahmanyam was appointed Chairman of the Kargil Review
Committee in 1999, an inquiry commission set up by the Indian
government to analyse perceived Indian intelligence failures with the
Kargil War. The committee's final report (also referred to as the
'Subrahmanyam Report') led to a large-scale restructuring of Indian
Intelligence.
18. A Paramparagat Krishi Vikas Yojana (Traditional Farming
Improvement Programme) has been launched by Government of
India to support and promote organic farming and thereby improving
soil health. This will encourage farmers to adopt eco-friendly
concept of cultivation and reduce their dependence on fertilizers and
agricultural chemicals to improve yields.
19. A
20. D Mohiniyattam, also spelled Mohiniattam is a classical dance form
from Kerala, India. Believed to have originated in 16th century CE, it
is one of the eight Indian classical dance forms recognized by the
Sangeet Natak Akademi. It is considered a very graceful form of
dance meant to be performed as solo recitals by women.
21. D
22. B
23. C Space Exploration Technologies Corporation (SpaceX) is an
American aerospace manufacturer and space transport services
company with its headquarters in Hawthorne, California, USA. It
was founded in 2002 by former PayPal entrepreneur and Tesla
Motors CEO Elon Musk with the goal of creating the technologies

119
that will enable humanity to reduce space transportation costs and
enable the colonization of Mars (fully and rapidly reusable rockets).
24. C
25. A Bhalchandra Vanaji Nemade (born 1938) is a Marathi writer from
Maharashtra, India.Famous for his books "Hindu" & "Kosala". Also
he is known for his novel "Hindu jagnyachi samrudhha adgal". He is
a recipient of the civilian honour of Padma Shri (2011) and the
Jnanpith Award (2014).
26. B A Snickometer, commonly known as Snicko, is used in televising
cricket to graphically analyse sound and video, and show whether a
fine noise, or snick, occurs as ball passes bat. It was invented by
English computer scientist Allan Plaskett in the mid-1990s. The
snickometer was introduced by Channel 4 in the UK, who also
introduced the Hawk-Eye and the Red Zone, in 1999.
27. C
28. C
29. B Countries across the globe committed to create a new international
climate agreement by the conclusion of the U.N. Framework
Convention on Climate Change (UNFCCC) Conference of the
Parties (COP21) in Paris in December 2015. In preparation, countries
have agreed to publicly outline what post-2020 climate actions they
intend to take under a new international agreement, known as their
Intended Nationally Determined Contributions (INDCs). The INDCs
will largely determine whether the world achieves an ambitious 2015
agreement and is put on a path toward a low-carbon, climate-resilient
future.
30. B A complex of vesicles and folded membranes within the cytoplasm
of most eukaryotic cells, involved in secretion and intracellular
transport.
31. B
32. B
33. B
34. A Sankalp is an initiative from D/o Pension & Pensioners' Welfare,
Government of India, to provide a platform for the pensioners to
access opportunities available for useful interventions in the society.
It also facilitates the Organizations working in these areas to select
appropriate skill and expertise from the available pool of volunteers.
35. C A tax, such as income tax, which is levied on the income or profits of
the person who pays it, rather than on goods or services.

120
36. D
37. D
38. C The Aegean Sea is an elongated embayment of the Mediterranean
Sea located between the Greek and Anatolian peninsulas, i.e.,
between the mainlands of Greece and Turkey.
39. D Jamsedhji Nusserwanji Tata (3 March 1839 – 19 May 1904) was an
Indian pioneer industrialist, who founded the Tata Group, India's
biggest conglomerate company. He was born to a Parsi Zoroastrian
family in Navsari then part of the princely state of Baroda. He
founded what would later become the Tata Group of companies.
Jamsetji Tata is regarded as the legendary "Father of Indian
Industry".
40. B
41. B
42. C INS Varsha is a new naval base being developed under Project
Varsha for the Indian Navy. This base will be the home of the navy's
new fleet of nuclear submarines and ships. It was planned to be
located within a radius of approximately 200 kilometres (124.27
statute miles) from Visakhapatnam, the headquarters of the navy's
Eastern Naval Command. Previous news reports suggested that
Gangavaram had been the initial site for the new base. The base is
now being developed at Rambilli, which is 50 km from
Visakhapatnam.
43. C
44. A
45. B
46. A
47. A
48. D Madhav Dhananjaya Gadgil (b. 1942) is an Indian ecologist,
academic, writer, columnist and the founder of the Centre for
Ecological Sciences, a research forum under the aegis of the Indian
Institute of Science. He is a former member of the Scientific
Advisory Council to the Prime Minister of India and the Head of the
Western Ghats Ecology Expert Panel (WGEEP) of 2010, popularly
known as the Gadgil Commission. He is a recipient of the Volvo
Environment Prize and the Tyler Prize for Environmental
Achievement. The Government of India awarded him the fourth
highest civilian award of the Padma Shri in 1981 and followed it up
with the third highest award of the Padma Bhushan in 2006.

121
49. B
50. A Ovarian cancer is a cancer that begins in an ovary. It results in
abnormal cells that have the ability to invade or spread to other parts
of the body. When this process begins, symptoms may be vague or
not apparent, but they become more noticeable as the cancer
progresses. These symptoms may include bloating, pelvic pain, and
abdominal swelling, among others. Common areas to which the
cancer may spread include the lining of the abdomen, lining of the
bowel and bladder, lymph nodes, lungs, and liver.

2016
1 C Other famous books by Dr. A.P.J Abdul Kalam are wings of fire, 2020-
A Vision for the New Millennium, Ignited Minds, Transcendence My
Spiritual Experiences with Pramukh Swamiji and Reignited: Scientific
Pathways to a Brighter Future etc.
42 D Under directives of Supreme Court, a floor test in the Uttarakhand
Legislative Assembly was held on May 10. While ruling that the
President's rule was temporarily lifted for two hours in Uttarakhand
when the vote of confidence will take place, the apex court scheduled
the floor test for May 10 from 11 am to 1 pm. Harish Rawat wins floor
test and became Chief Minister again, Center lifted the President's rule.
43 C Ashoka Chakra (4-Jan-1952) is awarded excellence at peace time. It’s
the highest in its kind.
44 A Mr. Arun Jaitly is the Union Finance Minister of India. Educated at Sri
Ram College of Commerce, University of Delhi and Faculty of Law,
University of Delhi. By
Profession he is Senior Advocate, Supreme Court of India.
45 A Mars’ reddish color is quite visible to the naked eye. This color is due
to the presence of iron oxide dust, a.k.a. rust. While this doesn’t mean
that Mars is made up of rust, it would be safe to say that the planet is
fully covered with it.
46 B CJI, In case of further calamity Senior-Most Judge of Supreme Court
shall do the needful.
47 B The programme aims at making all national highways Railway Level
Crossing free by 2019. 208 new “road over bridges / road under
bridges” are envisaged for construction, while 1500 bridges will be
widened, rehabilitated or replaced.
48 B The Paris Agreement shall enter into force on the 30th day after the
date on which at least 55 Parties to the Convention accounting in total
for at least an estimated 55% of the total global greenhouse gas
emissions have deposited their instruments of ratification, acceptance,
approval or accession with the Depositary. The first of these thresholds
was achieved on 22 September 2016.
49 B The ritual was established 80 years ago for the Berlin Games, based on
a ceremony in Ancient Olympia where games were held for more than
1,000 years. Actor Katerina Lehou, who lit the torch, offered a mock

122
prayer to Apollo, the old Greek god of light and music.
50 D Along with tactile maps of station describing the location and distance
of entrance, platforms, counters, washrooms etc, schedule of trains,
showing their arrival and departure time, the authorities have also put
up 400 metal signages in Braille along the railings of the staircases
leading various platforms. Anuprayaas, an NGO assisted the railways
in make Mysuru railway station blind-friendly.
51 D The Assembly elections in West Bengal, Tamil Nadu, Kerala and
Assam and Union Territory of Puducherry were held between April 4,
2016 and May 16, 2016, spread over 43 days. It was single-day polling
in Tamil Nadu, Kerala and Puducherry on May 16, while it will be in
six phases in West Bengal and two in Assam.
52 D World Consumer Day – 15 March.
National Consumer Day is on 24 December.
53 D In January 2016, Sikkim state was officially announced as the country’s
first fully organic state i.e. the farmers in the state are using natural
manure form cow in place of chemical fertilisers, pesticides or
insecticides. The Sikkim model can be emulated by the other states in
country, especially those states which are focussing on organic
agricultural practices.
54 D Greece has become the first developed country in history to default to
the International Monetary Fund.
The cash-strapped nation failed to make a €1.5bn payment to the IMF
by an 11pm deadline on Tuesday, triggering an arrears process which
was last suffered by Zimbabwe in 2001.
55 C The country’s first “smokeless village” was Vychakurahalli of
Gauribidanur taluk of Chickballapur district. Around 60 villages in
Karnataka have successfully moved away from traditional stoves that
use firewood and cow dung cakes to liquefied petroleum gas (LPG)
stoves, thereby becoming “smokeless villages”.
56 D A diaspora is a large group of people with a similar heritage or
homeland who have since moved out to places all over the world.
The term diaspora comes from an ancient Greek word meaning "to
scatter about." And that's exactly what the people of a diaspora do —
they scatter from their homeland to places across the globe, spreading
their culture as they go.
57 A The National Human Rights Commission is an expression of India's
concern for the protection and promotion of human rights. It came into
being in October,1993.
58 B India, the world's third largest emitter of greenhouse gases, ratified the
landmark Paris climate deal, giving a significant push for the deal to
enter into force by the end of this year 2016. Permanent Representative
to the UN Ambassador Syed Akbaruddin handed over the Instrument of
Ratification signed by President Pranab Mukherjee, to Santiago
Villalpando, the Head of the Treaties Division at the UN, on Mahatma
Gandhi's birth anniversary 2nd October 2016.
59 C Telangana has become the first State in the country to introduce
compulsory gender education at the graduate level. In this regard, state
government has introduced bilingual textbook titled ‘Towards a World
of Equals’ on a pilot basis in engineering colleges.

123
60 B Crime and Punishment is a novel by the Russian author Fyodor
Dostoevsky. It was first published in the literary journal The Russian
Messenger in twelve monthly installments during 1866. It was later
published in a single volume. It is the second of Dostoyevsky's full-
length novels following his return from 10 years of exile in Siberia.
61 C Brendon MacCullum (New Zealand) blasted the fastest century in Test
history, a 54-ball effort that broke the record jointly held by Viv
Richards and Misbah-ul-Haq (56 Balls). In his 101st and final
Test against Australia at Christchurch, New Zealand
62 B Boy Scout Motto is Be Prepared. The Scout motto means that you are
always ready to do what is necessary to help others. It also means you
are ready, willing, and able to do what is necessary in any situation that
comes along.
63 D PriyadarshiniChatterji (Delhi Model) Born on 12 Aug 1996 in
Guwahati, Assam.
64 A Bhubaneswar top the list of twenty smart cities in India.
65 C The Head of State of the Republic of Sri Lanka is the President. The
President is also the Head of the Executive, the Head of the
Government, and the Commander in Chief of the Armed Forces. The
President is elected by the people and holds office for a period of six
years. The President has the right to attend, address and send messages
to Parliament at any time.
66 D Sir Benegal Narsing Rau, (born February 26, 1887, Karkala or
Mangalore, Mysore [now Karnataka], India—died November 30, 1953,
Zürich, Switzerland) one of the foremost Indian jurists of his time. He
helped draft the constitutions of Burma (Myanmar) in 1947 and India in
1950. As India’s representative on the United Nations Security Council
(1950–52), he was serving as president of the council when it
recommended armed assistance to South Korea (June 1950). Later he
was a member of the Korean War cease-fire commission
67 A The Duke of Cambridge (Prince William) is the second in line to the
throne and the elder son of The Prince of Wales and Diana, Princess of
Wales. Kate Middleton is Duchess of Cambridge and wife of Prince
William.
68 B 16 Universities.
69 D National Green Tribunal is a statutory body established by a
Government Notification using the powers of Section 3 of the NGT Act
2010. It has replaced National Environment Appellate Authority.
Qualification for Chairperson: The person should have been either a
Judge of India’s Supreme Court or Chief Justice of a High Court in
India.
70 C SAMRUPA: The world's first cloned buffalo calf, was to be India's
answer to Dolly the sheep. But unlike Dolly, the first mammal cloned
13 years ago, who lived for seven years, Samrupa succumbed to a lung
infection, five days after it was born.
71 A One of the recommendations of the Special Investigation Team (SIT)
on Black Money was that quoting of PAN should be made mandatory
for all sales and purchases of goods and services where the payment
exceeds Rs.1 lakh. Considering the representations, it has been decided
that quoting of PAN will be required for transactions of an amount

124
exceeding Rs.2 lakh regardless of the mode of payment.
72 A President of India will submit resignation to Vice-President. Article 56
of the Indian Constitution makes it clear that the term of the President
is five years from the date on which he enters the office. His term may
terminate earlier (i) by resignation addressed to the Vice-President, or
(ii) by removal by impeachment for violation of the Constitution.
73 B DipaKarmakar (born 9 August 1993, Agartala) is
an Indian gymnast from Agartala, Tripura who represents India at the
international circuit. She came to recognition when she won a bronze
medal at the2014 Commonwealth Games, held at Glasgow becoming
the first Indian female gymnast to do so in the history of the games.
74 D According to Article 79 of the Constitution of India, the Parliament
consists of President of India and the two Houses of Parliament known
as Council of States (Rajya Sabha) and House of the People (Lok
Sabha).
75 D The Rajya Sabha in April, 2016 passed two bills to repeal 1,053
obsolete laws. The Appropriation Acts (Repeal) Bill 2015 that seeks to
repeal 758 old appropriation acts and The Repealing and Amending
(Third) Bill 2015 to repeal 295 enactments and amend certain others,
were passed by the Rajya Sabha. The bills had been passed by the Lok
Sabha in May 2015.
76 C The term the Constitution (100th Amendment) Act, 2015 was in news
in the fourth week of May 2015 as the President of India Pranab
Mukherjee gave his assent to the Constitution (119th Amendment) Bill,
2013 that related to the Land Boundary Agreement (LBA) between
India and Bangladesh.
77 C Mercury is the closest planet to the Sun, and so it’s the fastest to orbit
the Sun. In fact, Mercury only takes 88 days to orbit the Sun. In other
words, Mercury’s orbit only takes 24% as long as Earth’s orbit.
78 D KuldipNayar (born 14 August 1923) is a
veteran Indian journalist, syndicated columnist, human right activist
and author, noted for his long career as a left-wing political
commentator. He was also nominated as a Member of the upper house
of the Indian Parliament in 1997.
79 D The 'Start up India Stand up India' initiative was announced by Prime
Minister Narendra Modi in his address to the nation from the ramparts
of Red Fort on 15th August, 2015.
80 C Baht (Thailand)
81 C On November 5, 2015 Prime Minister of Nepal KP Sharma Oli
expanded his cabinet by inducting highest numbers of deputy prime
ministers in the history of the country. With the expansion, the Oli-led
government has six deputy PMs.
82 D After English & French, Punjabi became the third most common
language in Parliament of Canada.
83 D Sania Mirza (Tennis)
84 C Pakistan
85 C According to CII-Deloitte report on the annual status of higher
education of states and Union Territories in India-2015, Karnataka is
reported to have around 13,241 foreign students, which constitutes

125
around 38.08% of total foreign students studying in India. It ranks
highest in attracting students from overseas.
86 A Japan has again demonstrated its prowess in high-speed rail travel with
its state-of-the-art maglev train setting a world record of just over
600km/h (373mph), just days after it broke its previous 12-year-old
record.
87 D Under provisions Article 85, The President shall from time to time
summon each House of Parliament to meet at such time and place as he
thinks fit, but six months shall not intervene between its last sitting in
one session and the date appointed for its sitting in the next session.
88 B The Prime Minister is the De-Facto Chair of the NITI Aayog (Formerly
known as Planning Commission).
89 A Under Article 343 (1), The official language of the Union shall be
Hindi in Devanagari script.
90 D In this edition of Asian Indoor Athletics Championships Indian won 1
Gold medal, 3 silver and 3 bronze medals. The 2017 Asian Athletics
(Outdoor) Championship will be held Ranchi (Jharkhand), India.

NLU-D
AILET
2009
Question Answer Explanation
Number
28. B Resonance is a phenomenon in which a vibrating system or external force
drives another system to oscillate with greater amplitude at a specific
preferential frequency.
29. D

30. C The most abundant metal on Earth is aluminum. Aluminum is the third
most abundant element in the Earth's crust, and makes up 8 percent of it by
parts per million.
31. B

32. D Self-explanatory.

33. A Balban introduced the practice of Sijda in his court. The ceremony
required for each noble to bend down on their knees and touch the ground
with their forehead in salutation to the Sultan.
34. A Woods Despatch (1854) was the first Milestone of Indian education
because it had suggested the various ideas and ways for the development
of Indian education. The Wood’s Despatch (1854) recommended the
establishment of teacher training schools in each of the provinces.
35. A PWD came into existence in July 1854 when Lord Dalhousie established a
central agency for execution of public works and set up Ajmer Provincial
Division.
36. C In 1867, Dadabhai Naoroji put forward the 'drain of wealth' theory in

126
which he stated that the Britain was completely draining India. He
mentioned this theory in his book Poverty and Un-British Rule in India.
37. A The special session of the Congress began in Calcutta on September 4th,
1920. It was convened to consider the recommendations of Hunter
Committee, the Treaty of Sevres, and non-cooperation which finally led to
the Non Cooperation Movement.
38. A In meteorology, a cyclone is a large scale air mass that rotates around a
strong center of low atmospheric pressure.
39. C Contour plowing or contour farming or Contour bunding is
the farming practice of plowing and/or planting across a slope following
its elevation contour lines. These contour lines create a water break which
reduces the formation of rills and gullies during times of heavy water run-
off; which is a major cause of soil erosion.
40. D The Red Sea is an extension (or inlet) of the Indian Ocean, located
between Africa and Asia. For map reference, please see:
http://www.worldatlas.com/aatlas/infopage/redsea.htm
41. B

42. B Orbital eccentricity measures the flatness of the orbit. Earth comes closest
to the sun every year around January 3. It is farthest from the sun every
year around July 4.
43. A The gilt-edged market is the market in government securities or the
securities guaranteed (as to both principal and interest) by the government.
44. B The Laffer Curve is a theory developed by supply-side economist
Arthur Laffer to show the relationship between tax rates and the amount of
tax revenue collected by governments.
45. A In 1997, the United Front government had raised around Rs 10,000 crore
in taxes through VDIS, a brainchild of the then Finance Minister P
Chidambaram.
46. A An Industrial Company being a company registered for not less than five
years which has at the end of any financial year accumulated losses equal
to or exceeding its entire net worth should be considered as a Sick
Company
47. A The author culminated in his study in three-volume Asian Drama: An
Inquiry into the Poverty of Nations, published in 1968.
48. D The wealth tax is not leviable on urban land which is “classified as
agricultural land in the records of the government; and used for
agricultural purposes.”
49. D
50. B
51. None There were 395 Articles and 8 schedules.
52. D This demand was officially asserted for the first time by the Indian
National Congress (INC) in 1935. However, only in 1940, the Coalition
Government in Britain, recognised the principle that the people of India
had the right to frame a Constitution of their own through Constituent
Assembly of India elected for this purpose.
53. D There is no Party by this name in India.
54. A The Expenditure Charged on CFI (112(2)(a)) which generally deals with
expenditures mentioned in the constitution itself. For example the salary

127
and allowances. The reason why Parliament isn't allowed to vote on this
'charged' expenditure is to allow financial independence and security to the
executive and judiciary during the term of their office.
55. D Between 25 June 1975 to 21 March 1977 under controversial
circumstances of political instability under Indira Gandhi's prime
ministership — "the security of India" having been declared "threatened
by internal disturbances".
56. B Citizenship of India by naturalization can be acquired by a foreigner (not
illegal migrant) who is ordinarily resident in India for TWELVE YEARS
(throughout the period of twelve months immediately preceding the date
of application and for ELEVEN YEARS in the aggregate in the
FOURTEEN YEARS preceding the twelve months).
57. C The Constitution originally provided for the right to property under
Articles 19 and 31. Article 19 guaranteed to all citizens the right to
acquire, hold and dispose of property. Article 31 provided that "no person
shall be deprived of his property save by authority of law." It also provided
that compensation would be paid to a person whose property has been
taken for public purposes.
58. D Most of the DPSPs reflect the ideology of socialism and welfare state.
Some of them are directly inculcating the Gandhian principles for
example: Article 40: Organization of village Panchayats Article 43:
Promotion of cottage industries Article 46: Promotion and protection of
interests of educational and economic interests of SCs, STs, and other
weaker sections of the society and to protect them from social injustice
and exploitation Article 47: Prohibition of consumption of intoxicating
drinks and drugs which are injurious to health Article 48: Prohibition of
slaughter of cows, calves and other milch and draught cattle and to
improve their breeds
59. B Art 360

http://lawprojectsforfree.blogspot.in/2010/08/constitution-of-india-
fundamental.html
60. A The case is automatically referred to the concerned High Court for
confirmation by the virtue of section 366(1) of CrPC.
61. B In India, a Motion of No Confidence can be introduced only in the Lok
Sabha (the lower house of the Parliament of India). The motion is admitted
for discussion when a minimum of fifty members of the house support the
motion.
62. B The decisions of other High Courts only have persuasive value. This
means that when one High Court is deciding upon a matter, the lawyers
can refer to how other High Courts have decided such matters but the the
deciding High Court will have autonomy to accept or reject other High
Court’s interpretation on the matter.
63. A It is mandatory for a person to be acquitted that he is unaware of nature of
act and/or, its legality. Sometimes a person knows the illegality of his act
but then also in fit of anger, emotions as delusions he might commit some
crime. In such situations, conditions like irresistible impulse, obsessive-
compulsive disorder, delusion, emotions, fits of anger can offer a ground
for medical insanity but will not constitute a legal ground for acquittal.
64. A In People's Union for Democratic Rights and others v. Union of India and

128
others [1982 II LLJ 454 SC (1982) 3 SCC 235], otherwise referred to as
the Asiad Workers Case, the Supreme Court said, "We are, therefore, of
the view that when a person provides labour of service to another for
remuneration which is less than the minimum wage, the labour or service
provided by him clearly falls within the scope and ambit of the words
"forced labour" under Article 23 (of the Constitution of India).
65. C
66. C All the elements of Undue Influence are present in the present case and
hence the contract is voidable.
67. A
68. C The purpose of a contract is to establish the agreement that the parties
have made and to fix their rights and duties in accordance with that
agreement. The courts must enforce a valid contract as it is made, unless
there are grounds that bar its enforcement.
69. C
70. D
71. D
72. A
73. D
74. C Indira Nehru Gandhi v Raj Narayan
75. C The Janata Party government headed by Morarji Deasi constituted an
enquiry commission to inquire into all the excesses committed in the
Indian Emergency. It was headed by Justice J.C. Shah, a former chief
Justice of India.
76. A
77. None The Companies Act 2013 has increased the maximum limit to 100
members from 10 and 20 for banking and other business respectively as
provided on the 1956 Act.
78. B
79. D An accomplice by accepting a pardon under S.306 of CrPC becomes a
competent witness and may be examined as any other witness on oath.
80. A Now updated by MOTOR VEHICLES (AMENDMENT) BILL, 2016
81. C
82. B
83. D

2010
Question Answer Explanation
Number
36. D The 2009 Nobel Peace Prize was awarded to United States President
Barack Obama for his "extraordinary efforts to strengthen international
diplomacy and cooperation between peoples".
37. A The current Prime Minister, Sheikh Hasina Wajed, were appointed on 6
January 2009 by the President of Bangladesh.
38. B The first President to be elected under the new constitution was Nelson
Mandela, and the incumbent president is Jacob Zuma.
39. D Roh committed suicide on 23 May 2009 by jumping from a mountain cliff
behind his home, after saying that "there are too many people suffering

129
because of me" on a suicide note on his PC.
40. D
41. None Nasim Zaidi
42. A The April 2010 Dantewada Maoist attack, was an 6 April 2010 ambush by
Naxalite-Maoist insurgents from the Communist Party of India (Maoist)
near Chintalnar village in Dantewada district, Chhattisgarh, India, leading
to the killing of 76 CRPF policemen and 8 Maoists — the deadliest attack
by the Maoists on Indian security forces.
43. C The world’s second cloned buffalo calf through the Advanced Hand-
guided Cloning Technique was born at NDRI, Karnal on June 6, 2009. It
survived for more than two years but she died of heart failure on August
18, 2011.
44. C
45. B The International Criminal Court announced on March 4, 2009 that it was
issuing an arrest warrant against Sudanese President Omar al-Bashir for
masterminding a campaign of crimes against humanity and war crimes by
government troops and Arab militias in the Darfur region.
46. D Dr. MS Gill WAS CEC in between 12 December 1996 to 13 June 2001.

47. None 1926. When British monarch King George V and his consort Queen Mary
held one of the grandest darbars on December 12, 1911, where he assumed
the crown of India and announced the shifting of the imperial capital from
Calcutta (now Kolkata) to Delhi, the new capital had remained
unchristened. T he King and the Queen, the first monarchs to attend their
coronation darbar in India, laid two plain foundation stones of the new
city, bearing just the date “December 15, 1911” engraved on it. According
to archival documents of the period at the National Archives of India
(NAI) here, it was on “December 31, 1926” that King George V officially
named the new capital as ‘New Delhi’. “It is hereby notified for general
information that His Majesty the King-Emperor has signified his approval
to the new capital being named “New Delhi”. This name will be brought
into use forthwith,” says the 1926 public notification in this regard by the
Home Department.
48. None 1916. After reading information about Japan Women's University in
Tokyo, Japan, Dhondo Keshav Karve felt inspired to establish in 1916 in
Pune the first university for women in India, with five students.
49. B The eighth schedule to Indian Constitution includes the recognition of 22
languages. Of these languages, 14 were initially included in the
Constitution. Sindhi language was added in 1967. Thereafter three more
languages viz., Konkani, Manipuri and Nepali were included in 1992.
Subsequently Bodo, Dogri, Maithili and Santhali were added in 2004.
50. None Justice Punchi commission was set up by UPA Government in 2007 and it
gave its recommendations in 2010.
51. C In the Government of India Act 1935 the British Raj set up a system of
separate electorates and separate seats for women.
52. B Srikrishna Committee on Telangana or the Committee for Consultations
on the Situation in Andhra Pradesh (CCSAP) was a committee headed by
former chief justice B. N. Srikrishna to look into the demand for separate
statehood for Telangana or keep the State united in the present form,
Andhra Pradesh. The committee was constituted by the Government of
India on 3 February 2010 and submitted its report on 30 December 2010 to

130
the Ministry of Home Affairs.
53. C Mr. Malimath headed the Committee on Criminal Justice System which
made series of recommendations on the need for reforming the system.
54. B Dr. Sachchidananda Sinha was the first elected chairman (temporary) of
Constituent Assembly. Later, Dr. Rajendra Prasad was elected as the
president.
55. A The Republic of India is governed in terms of the Constitution of India
which was adopted by the Constituent Assembly on 26th November, 1949
and came into force on 26th January, 1950.
56. D Most nominations received by a single film: Two films received 14
nominations:
 All About Eve (1950)
 Titanic (1997)
57. C Vijender Singh briefly reached World No.1 in the middle weight (75 kg)
category class in 2009, when the International Boxing Association's
(AIBA) list was updated after 2009 AIBA World Boxing Championships
held in Milan, Where he won India's first medal in an AIBA-WBC.
58. B Athletes from China won the most gold medals, with 51 gold medals.
Athletes from the United States won the most total medals, with 110.
59. B Article 331 of the Constitution says that President may appoint two
members from Anglo-Indian Community to Lok Sabha, if in his opinion
that this community is not represented adequately, notwithstanding
anything mentioned in Article 81 which deals with Composition of Lok
Sabha.
60. B "The Right to Vote" is neither Fundamental Right nor Constitutional Right
but a mere Statutory Right or Legal Right. The right originates from the
Constitution and in accordance with the constitutional mandate contained
in Article 326, but the right has been shaped by the statute, namely,
Representation of Peoples Act, 1951.
61. C section 20 (1) of the RTI Act states that where the CIC or the SIC at the
time of deciding any complaint or appeal is of the opinion that the central
public information officer or the state public information officer has
without any reasonable cause refused to receive an application for
information or has not furnished information within the time specified or
denied the request with mala fide intention or knowingly given incorrect,
incomplete or misleading information or destroyed information which was
the subject of the request or obstructed in any manner in furnishing the
information, it shall impose a penalty of Rs 250 each day till application is
received or information is furnished. However, the total amount of such
penalty shall not exceed Rs. 25,000.
62. D In winning eight gold medals at the 2008 Beijing Games, Michael Phelps
broke fellow American swimmer Mark Spitz's record of seven first-place
finishes at any single Olympic Games.
63. B Henry Dunant was shocked by the terrible aftermath of the battle at
Solferino, the suffering of the wounded soldiers, and the near-total lack of
medical attendance and basic care. After this, he promoted the idea of
establishment of ICRC.
64. C Boutros Boutros-Ghali (born 14 November 1922) is an Egyptian diplomat
who was the sixth Secretary-General of the United Nations from 1 January
1992 to 1 January 1997. Before being elected in 1991, he was Egypt's

131
Minister of State for Foreign Affairs, and played a part in the peace
agreements between Egypt and Israel. Annan replaced Boutros Boutros-
Ghali, becoming the first person from a black African nation to serve as
Secretary-General.
65. A Ban Ki-moon (born 13 June 1944) is a South Korean diplomat and the
current Secretary-General of the United Nations.
66. B Tilak was one of the first and strongest proponent for Swaraj (complete
independence) in the Indian consciousness, and is considered the father of
Hindu nationalism. Tilak’s famous quote “Swaraj is my birthright, and I
shall have it” is well-remembered in India even today. Known as the father
of the Indian unrest, Tilak was the first popular leader of the Indian
Independence Movement.
67. D In 1937, Burma began to be administered separately by the Burma Office
under the Secretary of State for India and Burma.
68. B Sirima Ratwatte Dias Bandaranaike commonly known as Sirimavo
Bandaranaike was a Sri Lankan stateswoman and politician and the
modern world's first female head of government. She served as Prime
Minister of Ceylon and Sri Lanka three times, 1960–65, 1970–77 and
1994–2000, and was a long-time leader of the Sri Lanka Freedom Party.
69. A At 17,075,200 square kilometres (6,592,800 sq mi), Russia is the largest
country in the world by surface area, covering more than one-eighth of the
Earth's inhabited land area, and the ninth most populous, with over 146.6
million people at the end of March 2016.
70. A In cloud computing, the word cloud (also phrased as "the cloud") is used
as a metaphor for "the Internet," so the phrase cloud computing means "a
type of Internet-based computing," where different services — such as
servers, storage and applications — are delivered to an organization's
computers and devices through the Internet.

2011
Question Answer Explanation
Number
36 B Kashiwazaki-Kariwa is the world's largest rated nuclear power
station. With seven reactors generating 8,212MW, the station,
owned and operated by the Tokyo Electric Power Company
(TEPCO), can provide electricity to 16 million households.
"Kashiwazaki-Kariwa is the world's largest rated nuclear power
station."
37 C
38 C On the INES scale, Level 7 describes an accident serious enough to
require a country to implement countermeasures to protect the public
from the health
39 C
40 B Now Vijay Sharma
41 D France
42 A Dr. Nazim Zaidi
43 B Mohammad Hidayatullah was the sixth Vice-President of India who
was in this position from 31st August 1976 to 30th August 1984 who

132
also served as the Chief Justice
44 C
45 D Fathimath Dhiyana Saeed is a Maldivian diplomat, and was the
Secretary-General of the South Asian Association for Regional
Cooperation.
46 A Inventors who came before Willis Haviland Carrier tinkered with
cooling machines. But it was Carrier's creation that launched this
modern idea
47 B Another early internal combustion, petroleum fueled motorcycle was
the Petroleum Reitwagen. It was designed and built by the
German inventors Gottlieb Daimler and Wilhelm Maybach in Bad
Cannstatt, Germany in 1885.
48 C
49 D Afghanistan
50 C Asthma
51 C
AGREEMENT FOR COOPERATION BETWEEN THE
GOVERNMENT OF THE UNITED STATES OF AMERICA
AND THE GOVERNMENT OF INDIA CONCERNING
PEACEFUL USES OF NUCLEAR ENERGY (123
AGREEMENT) is between The Government of India and the
Government of the United States of America,

52 D Rajasthan
53 A J. Ramaswamy
54 B As per the 4th schedule, UP due to its population has the most
number of seats allocated.
55 D 6 years
56 C P.J. Thomas
57 B The Lok Sabha at present consists of 545 members including the
Speaker and two nominated members.
58 B A seven-judge Constitution bench of the High Court had by a
majority judgement of 5:2 held the Andhra Pradesh Reservation for
Socially and Educationally Backward Classes of Muslims Act, 2007
“unsustainable” and violative of Article 14 (equality before law) and
other provisions pertaining to prohibition of discrimination by State
on grounds of religion, race, caste, sex or place of birth.
59 D
60 A
61 C
62 D
63 B Sir B. N. Rau, the Constitutional Adviser to the Constituent
Assembly who prepared a rough draft of the Constitution for the
Assembly.
64 B
65 B

133
66 B The wrote A Memory of Solferino , which eventually led to the
creation of the International Committee for Relief to the Wounded,
the future International Committee of the Red Cross (ICRC). Dunant
was a member and acted as secretary.
67 D The Maastricht Treaty was signed on February 7, 1992, by the
leaders of 12 member nations, and it reflected the serious intentions
of all countries to create a common economic and monetary union
68 D
69 A The Kyoto Protocol is an international agreement linked to the
United Nations Framework Convention on Climate Change
70 A he Archean rocks inIndia are called Purana Rocks means the oldest
rocks. The Archean or Purana rock system in Indiais found in
Aravallis mountains, 2/3rd of the Deccan peninsula and some parts
of north east.

2012
Question Answer Explanation
Number
36 C
37 B Any currency that can have its exchange rate affected by the
intervention of a central bank. This is opposed to a currency that is
determined solely by the forces of supply and demand in the world
market.
38 C The Central Statistics Organisation is responsible for coordination of
statistical activities in the country, and evolving and maintaining
statistical standards. Its activities include National Income
Accounting
39 A
The year 1921 is taken as the demographic divide for the reason that
before this year, the population was not stable, sometimes it
increased and at other times it decreased.

The growth rate of population was generally low before 1921. But
after this year, there has been considerable and continuous increase
in the population.

40 A Auroras are but one symptom of a larger space weather system in


which solar material and radiation can affect Earth's own magnetic
environment and block radio communications, disturb onboard
satellite computers, or -- at their worst -- cause electrical surges in
power grids.
41 B The value 9.8 m/s^2 is the average acceleration of a falling object
due to the force of gravity on Earth.
42 C Due to increase in air pressure, mercury rises.
43 B Tube opens so as to make air pass from the middle of the ear to the
back of the nose which is open thereby causing the pop.
44 B Much like a thermostat regulates the temperature inside your home,
the hypothalamus regulates your body temperature, responding to

134
internal and external stimuli and making adjustments to keep the
body within one or two degrees of 98.6 degrees.
45 D There are nine PIN zones in India, including eight regional zones
and one functional zone (for the Indian Army). The first digit of the
PIN code indicates the region. The second digit indicates the sub-
region, and the third digit indicates the sorting district within the
region.
46 D
47 B Chand Bibi, Queen Regent of Nizam Shahi Sultanate, defended the
Kingdom against the Mughals during the siege of Ahmednagar.
48 D His external relations were aimed at seeking the support of foreign
power against English. His embassies to Paris and Constantinople
etc were focussed at single point of confrontation with the English.
49 D Annexation policy was Doctrine of Lapse.
50 A A routine procedure of the time at the Pietermaritzburg train station
– Gandhi’s arrest for defending his right to travel in the whites only
wagon – would later change the world. This event is what initiated
Gandhi’s contemplation of racial discrimination, and represents the
beginning of his philosophy of nonviolent protest and many arrests
in the defence of the Indian people.
51 A On February 18, 1946, a section of non-commissioned officers and
sailors known as Ratings, serving in the Royal Indian Navy,
mutinied against the British Officers. The mutiny started as a strike
by the ratings to protest against the hardships regarding pay, food
and racial discrimination.
52 D The Preamble to Indian constitution is based on “Objective
Resolution” of Nehru.
53 B Hyperinflation is a situation where the price increases are so out of
control that the concept of inflation is meaningless
54 D the moon possesses a solid, iron-rich inner core with a radius of
nearly 150 miles and a fluid, primarily liquid-iron outer core with a
radius of roughly 205 miles.
55 D Convection currents in the magma drive plate tectonics. Heat
generated from the radioactive decay of elements deep in the interior
of the Earth creates magma (molten rock) in the aesthenosphere. The
aesthenosphere (70 ~ 250 km) is part of the mantle, the middle
sphere of the Earth that extends to 2900 km.
56 C
57 A The aorta is the largest artery in the body. The aorta begins at the
top of the left ventricle, the heart's muscular pumping chamber.
58 D A sunken stomata is a stomata in a small pit, which protects the
escaping water vapor from air currents, decreasing water loss from
the leaf. Sunken stomataare commonly found in plants in arid
environments as one of their adaptations to preserve water.
59 B
60 B
61 A
62 A
63 C Keeping the State united by simultaneously providing certain

135
definite Constitutional/Statutory measures for socio-economic
development and political empowerment of Telangana region -
creation of a statutorily empowered Telangana Regional Council
64 B
65 A
66 B Aung San Suu Kyi , Burma's pro-democracy leader and Nobel Peace
laureate, spent more than 15 years in detention, most of it
under house arrest. She was released from her current third period of
detention on Saturday 13th November 2010.
67 A
68 B
69 B
70 B

2013
Question Answer Explanation
Number
36 B either of the two times in the year, the summer solstice and
the winter solstice, when the sun reaches its highest or lowest point
in the sky at noon, marked by the longest and shortest days
37 D
38 A It is coldest just about the time of sunrise because this is the hour
at which the atmosphere has been without the heat of the sun for
the longest time.
39 A The main difference between a geyser and a hot spring is that
a geyser is plugged with an obstruction near the opening of the spout
and a hot spring is allowed to flow freely.
40 B Revenue system during the reign of Sher Shah Suri was quite an
efficient one. The main sources of income of the state were the land-
revenue
41 B Indeed, there is a monument identical to the Taj that stands in
Aurangabad in Maharashtra. It is a mausoleum called Bibi Ka
Maqbara ("Tomb of the Lady") built by Prince Azam Shah, the son
of the sixth Mughal Emperor Aurangzeb, between 1651 and 1661
A.D, in the memory of his mother, Dilras Banu Begum.
42 A Ishwar Chandra Vidyasagar worked towards providing education to
women. He opened and ran many schools for girls at his own
expense. He was also known as “Daya-r Sagar” or “Karunar Sagar” (
literally, “ocean of kindness”) because of his charitable nature and
generosity.
43 C He was one of the most remarkable rulers of his time. He was highly
educated and was well versed in Arabic and Persian language. He
was well read in the subjects of religion, philosophy, astronomy,
mathematics, medicine and logic.
44 B The Permanent Settlement had come into operation in 1793. The
East India Company had fixed the revenue that each zamindar had to
pay.
45 B

136
46 B 39(d)- Equal pay for equal work
50- Uniform civil Code
47 A
48 B
49 B Motion of No-Confidence need not set out any grounds
50 D Electoral roll is prepared by the Election Commission of India
51 C A fiscal deficit occurs when a government's total expenditures
exceed the revenue that it generates, excluding money from
borrowings. Deficit differs from debt, which is an accumulation of
yearly deficits.
52 C
53 A A bear raid is a type of stock market strategy, where a trader (or
group of traders) attempts to force down the price of a stock to cover
a short position. The name is derived from the common use
of bear or bearish in the language of market sentiment to reflect the
idea that investors expect downward price movement.
54 D Foreign Exchange Reserve is the total of a country’s gold holdings
and convertible foreign currencies held in its banks, plus Special
Drawing Rights (SDR) and exchange reserve balances
55 B as interest rates are lowered, more people are able to borrow more
money. The result is that consumers have more money to spend,
causing the economy to grow andinflation to increase.
56 C
57 A Potassium bromide (KBr) is a salt, widely used as an anticonvulsant
and a sedative in the late 19th and early 20th centuries, with over-
the-counter use extending to 1975 in the US. Its action is due to
the bromide ion (sodium bromide is equally effective).
58 D There is an increase in frequency.
59 C If there is a wound in the wall of the blood vessel the blood will
coagulate, (just as in a living person), in the minutes immediately
after death.
If there is no wound in the walls of the blood vessel, the blood will
remain liquid for several days, after which it starts to decompose.
The blood in the arteries tends to drain into the capillaries and veins.
60 B
61 B NATO
62 B European space Agency
63 C Manpreet Kaur
64 A IOC
65 B 94th
66 C Navneet Kaur Dhillon
67 A Paan Singh Tomar
68 D Pope Francis
69 A Russia
70 D All

2014
Question Answer Explanation

137
Number
36 C INS Vikramaditya in place of INS Vikrant.
37 C Srilanka
38 D India and Bangladesh signed a liberalised visa agreement and a
landmark extradition treaty
39 B Now A. Nasim Zaidi
40 A New Delhi
41 A South Korea
42 C P V Sindhu
43 D Serena Williams
44 D 24
45 A Uhuru Kenyatta
46 C Lionel Messi
47 D Bitcoin is a cryptocurrency and a payment system invented by an
unidentified programmer, or group of programmers, under the name
of Satoshi Nakamoto.
48 A Sunil Gavaskar
49 D None. An impeachment proceeding that was initiated did not result
in impeachment.
50 B The amount of penalty shall be Rs. 250.00 per day, till the
information is furnished or the application is received, subject to a
maximum of Rs. 25,000.
51 B 30 years
52 C On 11 February 2011, the UN General Assembly, in its resolution
65/154, decided to proclaim 2013 International Year of Water
Cooperation. The objective of the Year is to raise awareness of both
the potential for increased cooperation and the challenges facing
water management in the context of greater demand for water
access, allocation, and services.
53 A The European Union (EU) is an economic and political union of 28
countries. It operates an internal (or single) market which allows free
movement of goods, capital, services and people between member
states.
54 C Under article 80 of the Constitution, the Council of States (Rajya
Sabha) is composed of not more than 250 members, of whom 12
are nominated by the President of India from amongst persons who
have special knowledge or practical experience in respect of such
matters as literature, science, art and social service.
55 D The Rajiv Gandhi Khel Ratna (RGKR) is India's highest honour
given for achievement in sports. The words "Khel Ratna" literally
mean "sports gem" in Hindi.
56 C Cloves are the aromatic flower buds of a tree in the family
Myrtaceae, Syzygium aromaticum. They are native to the Maluku
Islands in Indonesia, and are commonly used as a spice.
57 D Sunderbans.
58 B Dr. B R Ambedkar
59 B Taiwan wants to become a member of the United Nations. However,
it lost its seat in that body in 1971, when the U.N. chose to recognize
the People's Republic of China instead. There can be only one China

138
in the world, the U.N. decided, and the government in Beijing runs
it.
60 D Telecom equipments and machinery
61 B Tropic of Cancer is a line parallel to the Equator at 23-1/2 degrees
North. The significance of this line is that ‘Sun rays till this point
will almost be vertical and after this point they are mostly inclined
rays’. Hence this line divides India into tropical part (southern region
which will be hot) and sub-tropical part (northern region).
62 A
63 C The Tunisian Revolution, also known as the Jasmine Revolution,
was an intensive campaign of civil resistance, including a series of
street demonstrations taking place in Tunisia, and led to the ousting
of longtime president Zine El Abidine Ben Ali in January 2011.
64 B US is ex officio member
65 A He entered politics and was elected to the Madras assembly in 1952
and to Kerala assembly in 1957 and 1960. He was minister for law,
irrigation, power, prison and social welfare in the EMS
Namboodiripad government. He lost the election in 1965 from
Thalassery and thereafter concentrated on his law career. He was the
Supreme Court judge from 1973 to 1980.
66 B Air bags are not inflated from some compressed gas source but
rather from the products of a chemical reaction. The chemical at the
heart of the air bag reaction is called sodium azide, or NaN3.
67 D By the time it reaches the surface of the dun, the photosphere, it has
reached temperature of 5800 K which is perceived as white light.
68 C
69 B
70 A

2015

Question Answer Explanation


number
36. C The Assembly on Friday passed the Rajasthan Panchayati Raj
(amendment) Bill, 2015, which makes Class VIII pass mandatory for
the post of sarpanch — except in tribal reserved areas, where the
minimum qualification is Class V — and Class X for Zila Parishad
or Panchayat Samiti elections
37. B China
38. C On March 27, Modi had officially launched the 'Give-it-
Up' campaign, urging the well-off to surrender their LPG subsidy
39. C Kangana ranaut
40. B Romania
41. B Shashi Kapoor

139
42. A Six years after Bandung, the Movement of Non-Aligned Countries
was founded on a wider geographical basis at the First
Summit Conference
43. C Hepatitis B
44. A In an effort to reduce China’s harmful and plentiful greenhouse gas
emissions, Chinesecompany Sifang (a subsidiary of China South
Rail Corporation) has developed the world’s firsthydrogen powered
tram. The tram took two years of research and development to
complete.
45. D Founded by the first non-European Nobel Laureate Rabindranath
Thākur(popularly known as Tagore) in 1921, Visva-Bharati was
declared to be a central university and an institution of national
importance by an Act of Parliament in 1951. The President of India
is the Paridarsaka (Visitor) of the University, the Governor of West
Bengal is the Pradhana (Rector), and the Prime Minister of India acts
as the Acharya (Chancellor). The President of India appoints the
Upacharya (Vice-chancellor) of the University.
46. A Let Girls Learn is a United States government initiative to ensure
adolescent girls get the education they deserve
47. B Likud
48. C The real effective exchange rate (REER) is the weighted average of
a country's currency relative to an index or basket of other major
currencies, adjusted for the effects of inflation. The weights are
determined by comparing the relative trade balance of a country's
currency against each country within the index.
49. B As adults, humans can regenerate some organs, such as the liver. If
part of theliver is lost by disease or injury, the liver grows back to its
original size, though not its original shape. And our skin is
constantly being renewed and repaired
50. C Kayakulp
51. B
52. C
53. A The Mysuru royal family has a new heir in Yaduveer Gopal Raj
Urs to continue the Dasara traditions of the Wadiyar dynasty
54. A Pawan Chamling was chosen as the most dedicated Chief
Minster towards the environment and sustainable development
during the nationwide survey conducted by the Centre for Science
and Environment in 1998. He was voted as the Greenest Chief
Minister.
55. C The Fertile Crescent is the region in the Middle East which curves,
like a quarter-moon shape, from the Persian Gulf, through modern-
day southern Iraq, Syria, Lebanon, Jordan, Israel and northern
Egypt.
56. B
57. B

140
58. A This year, to mark David Beckham’s tenth year as a UNICEF
Goodwill Ambassador, UNICEF and David Beckham are
launching 7: The David Beckham UNICEF Fund, which aims to
help protect millions of children from danger.

Over the past decade as a UNICEF Goodwill Ambassador, David


Beckham has played a crucial role in raising awareness of
UNICEF’s work to save and change children's lives around the
world.

59. C Maharashtra
60. A India
61. D Madhya Pradesh
62. B Kerala Finance Minister KM Mani has been appointed as the
chairman of the Empowered Committee of State Finance Ministers
on Goods and Services Tax (GST).
63. B OPERATION DECISIVE STORM
Saudi-led air operations against the Houthis began after Yemeni
president Abdu Rabu Mansour Hadi's March 24 request for military
intervention "based on the principle of self-defense in Article 51 of
the Charter of the United Nations," as well as "the Charter of the
Arab League and the treaty of joint Arab defense."

64. D
65. C Nazim Zaidi
66. A Viswanathan Anand has a minor planet named after him. A minor
planet discovered in 1988 has been officially named "(4538)
Vishyanand".
67. D
68. A
69. C
70. A

2016

36 C The colourful Kullu folk dance (Nati), which saw


participation of 20,000 dancers during KulluDussehra
festival on October 26 last year, has entered the Guinness
book of world records.In the Nati, 10,000 men and women
attired in traditional Kullu dress performed at Dhalpur

141
Ground during KulluDussehra festival on October 26 last
year.The jamboree of over 20,000 people danced to spread
the message of 'BetiHaiAnmol' (Girl child is precious).

37 A Thomson Reuters 2015, Top 100 Global Innovators is based


on an analysis of overall patent volume, patent-grant success
rates, global reach and invention influence as evidenced by
citations. Amazon joined the ranks of Top 100 innovators for
the first time, generating handfuls of new inventions each
month in broad areas like data centers, devices and electronic
methods and systems.

38 B The Bretton Woods twins refers to the two multilateral


organizations created at the Bretton Woods Conference in
1944. They are:
 The World Bank
 The International Monetary Fund

39 C In economics and political science, fiscal policy is the use of


government revenue collection (mainly taxes)
and expenditure (spending) to influence the
economy.According to Keynesian economics, when the
government changes the levels of taxation and governments
spending, it influences aggregate demand and the level of
economic activity. Fiscal policy can be used to stabilize the
economy over the course of thebusiness cycle.

40 D Lakdawala Committee of 1993 was set such poverty line that


anyone above them would be able to afford 2400 and 2100
calories worth of consumption in rural and urban areas
respectively in addition to clothing and shelter.

41 C Prime Minister Narendra Modi described the landmark


climate change deal reached in Paris as the victory of
“climate justice” and said there were no winners or losers in
the outcome. He also said the deliberations showed collective
wisdom of the world leaders to mitigate climate change. This
deal was concluded in December 2015.

42 A A resourceful astromech droid, R2-D2 served Padmé


Amidala, Anakin Skywalker and Luke Skywalker in turn,
showing great bravery in rescuing his masters and their
friends from many perils. A skilled starship mechanic and
fighter pilot's assistant, he formed an unlikely but enduring
friendship with the fussy protocol droid C-3PO.

43 D All four statements are part of the quarter.

44 B Termed as UAN linked with Adhaar& PAN card.

45 C Cochin International Airport Ltd (CIAL) of Kerala has

142
become the first airport in the world to fully operate on solar
power. It achieved this feat after Kerala Chief Minister
Oommen Chand officially commissioned the 12 MWp solar
power plant near the cargo complex of airport.

46 C Two Years

47 D All Four statemnts are true in regard to PAN Card

48 B 11

49 A 100 Percent.

50 D Godavari-Krishna

51 B Court is a 2014 Indian courtroom drama film, written and


directed by ChaitanyaTamhane in his directorial debut.
Featuring a cast of newcomers, the film examines the Indian
legal system through the trial of an ageing folk singer at
a Sessions Court in Mumbai.

52 C A sidereal day is the time it takes for the Earth to rotate about
its axis so that the distant stars appear in the same position in
the sky. A solar day is the time it takes for the Earth to rotate
about its axis so that the Sun appears in the same position in
the sky. The sidereal day is ~4 minutes shorter than the solar
day.

53 B The Protocol was adopted at the third session of the


Conference of the Parties to the 1992 United Nations
Framework Convention on Climate Change (“the
Convention”), held at Kyoto (Japan) from 1 to 11 December
1997. This protocol was signed by USA on 12 Nov 1998 but
did not ratify.

54 B India is hosting the 8th BRICS Summit during its


Chairmanship which is scheduled to take place on 15-16
October 2016 in Goa. The theme of India’s BRICS
Chairmanship is Building Responsive, Inclusive and
Collective Solutions.

55 A Foreign portfolio investment (FPI) consists of securities and


other financial assets passively held by foreign investors.
Foreign portfolio investment (FPI) does not provide
the investor with direct ownership of financial assets, and
thus no direct management of a company. This type of
investment is relatively liquid, depending on the volatility of
the market invested in. It is most commonly used by
investors who do not want to manage a firm abroad.

56 A Tata Zica (Similar to Zika Virus) has been renamed as Tata


Tiago moniker

143
57 D Airtel 4G

58 C Afghanistan has become the 164th WTO member and the


36th least developed country (LDC) to join the global trade
body.The World Trade Organisation (WTO) on Thursday
formally approved Afghanistan’s membership at its 10th
ministerial conference in the Kenyan capital Nairobi.

59 D VHF

60 D 5-15 Percent

61 D Liquid crystals and their unusual properties have found a


wide range of commercial applications. They are used, for
example, in the liquid crystal displays (LCDs) in digital
watches, calculators, and computer and video displays.

62 D Saudi Arabia

63 A Firm without any 'real' existence other than a brass nameplate


(on a panel with other nameplates) at its registered (legal)
address. Accounting and legal firms (especially in tax
havens) rent their street addresses as the registered offices of
brassplate firms.

64 B Subway

65 C SaniaMirza (Tennis)

66 D France

67 A The amount has been further raised to 1,50,000.00 (One lac


Fifty thousand) from the earlier 1,00,000.00 (One Lac) in the
year AY 2015-16

68 C FTII (Pune)

69 D 50 days

70 B He for She Campaign on gender equality

144

Das könnte Ihnen auch gefallen